Tạp chí Epsilon số 9

205 98 0
Tạp chí Epsilon số 9

Đang tải... (xem toàn văn)

Tài liệu hạn chế xem trước, để xem đầy đủ mời bạn chọn Tải xuống

Thông tin tài liệu

, n − 1 phép cộng điểm P , cho đến khi tổng bằng Q , tuy nhiên có một số thuật toán tối ưu hơn để tìm n nhưng vẫn không thể giải được bài toán này trong thời gian đa thức vì thế dựa vào [r]

(1)

HÀM MOEBIUS VÀ ĐỊNH LÝ PHẦN DƯ TRUNG HOA Phùng Hồ Hải

ÍCH GÌ,

TỐN HỌC? Hà Huy Khối

DẪN NHẬP VỀ HÀM ZETA RIEMANN VÀ PHÉP BIẾN ĐỔI MELLIN

Ngô Bảo Châu

GIẢI NOBEL CỦA EINSTEIN HAY LÀ

SÓNG ĐIỆN THOẠI CÓ GÂY UNG THƯ HAY KHÔNG Đàm Thanh Sơn

VÀ CÁC

CHUYÊN MỤC KHÁC

tháng - 2016 NO

09

“Tích hai số nguyên, số là tổng hai bình phương, tổng hai bình phương.

DIOPHANTUS Khơng có gần với

thực tiễn lí thuyết đẹp”

(2)

Trần Nam Dũng

BIÊN TẬP VIÊN:

Võ Quốc Bá Cẩn Trần Quang Hùng Nguyễn Văn Huyện

Nguyễn Tiến Lâm Lê Phúc Lữ Ngô Quang Dương

Nguyễn Tất Thu Đặng Nguyễn Đức Tiến

tháng - 2016 NO

(3)

LỜI NGỎ CHO EPSILON SỐ 9

Ban Biên tập Epsilon

Epsilon số 9, mắt vào tháng 6, 2016 kỷ niệm chặng đường năm rưỡi đầy nỗ lực Ban Biên tập, tác giả cộng tác viên đầy ân tình từ phía độc giả ủng hộ chúng tơi suốt từ buổi ban đầu

Số tháng đến với bạn thời điểm mùa hè bắt đầu rộn rã Những kỳ nghỉ vẫy gọi hoa phượng giăng đầy sân trường, lớp học, sắc đỏ lan khắp nắng, khắp đường khiến vơ tình lướt ngang qua man mác hồi niệm Khơng khí tháng sáu làm sống lại chúng tôi, người thực Epsilon, nhiều ký ức đẹp đẽ Vì chúng tơi muốn dành lần mắt để giới thiệu chủ đề Trung học Cơ sở

Bên cạnh chuyên mục định kỳ, đề tài kiến thức toán trung học sở trình bày số tạp chí mùa hè

Ban biên tập cộng tác viên hy vọng rằng, Epsilon số góp thêm niềm vui học tập vào mùa hè tràn ngập niềm vui em học sinh, làm sống lại ký ức thuở bảng đen, mực tím với độc giả qua tuổi hoa niên

Trân trọng

(4)

Ban Biên tập Epsilon

Lời ngỏ cho Epsilon số

Hà Huy Khoái

Ích gì, tốn học?

Phùng Hồ Hải

Hàm Moebius định lý phần dư Trung Hoa 13

Ngô Bảo Châu

Dẫn nhập hàm zeta Riemann phép biến đổi Mellin 15

Đàm Thanh Sơn

Giải Nobel Einstein sóng điện thoại có gây ung thư hay khơng 21

Đặng Minh Tuấn

Hệ mật mã khóa cơng khai dựa đường cong Elliptic - Tổng quan hệ mật mã khóa cơng khai 23

Đặng Nguyễn Đức Tiến

Nghịch lý trai gái 43

Job Bouwman

Phép biến đổi Fourier có ý nghĩa vật lý gì? 47

Kiều Đình Minh

Tập hợp trù mật ứng dụng 53

Đinh Trung Hòa

About means of non-negative numbers and positive definite matrices 67

Trịnh Huy Vũ

(5)

Tạp chí Epsilon, Số 09, 06/2016

Trần Quang Hùng, Dương Ánh Ngọc

Định lý Sawayama Thébault tốn hình học thi Olympic 91

Lê Phúc Lữ

Về toán tam giác 80-80-20 115

Nguyễn Tài Chung

Sử dụng tổng tích phân để tính giới hạn dãy số 121

Nguyễn Ngọc Giang

Sáng tạo với tốn hình học Trung học sở 133

Trung Dũng

“Nếu bạn không nuôi dưỡng, đam mê từ bỏ bạn" - Trò chuyện đường đến với CMU Phạm Hy Hiếu 149

Võ Quốc Bá Cẩn

Về phong trào Olympic Toán Saudi Arabia 159

Trần Nam Dũng

Bài toán hay lời giải đẹp 163

Trần Nam Dũng

Đồng thức Brahmagupta-Fibonacci ứng dụng 165

Ban biên tập

Một số toán đề thi vào trường chuyên 173

Ban biên tập

Tuyển chọn đề thi Olympic năm 2016 dành cho học sinh THCS 183

Ban biên tập

Lời giải đề thi Toán quốc tế Formula of Unity - The Third Millennium 189

Trần Nam Dũng

(6)(7)

ÍCH GÌ, TỐN HỌC?

(Hà Huy Khối)

Bài giảng đại chúng, kỷ niệm5năm Chương trình quốc gia phát triển Tốn học thành lập Viện nghiên cứu cao cấp Toán (VIASM),20/12/2015

1 Câu hỏi “Ích gì?”

Trong dịp kỷ niệm5năm,10năm, tổ chức đó, người ta thường liệt kê việc làm, kết đạt Thực chất cố gắng “chứng minh” rằng, việc thành lập tổ chức cần thiết, có ích Vậy nên câu hỏi “Ích gì, Chương trình quốc gia phát triển Tốn học?”, “Ích gì, VIASM?”, khơng phát biểu cách cơng khai, chắn lởn vởn đầu khơng người, đặt lên bàn nhà hoạch định sách, Bộ Tài chính, Bộ Giáo dục Đào tạo

Vậy nên, đề nghị làm “bài giảng đại chúng”, tơi chọn đề tài “Ích gì, Tốn học?” Mà người “mách nước” cho tơi chọn đề tài lại khơng nhà tốn học, mà Chế Lan Viên! Hình ơng trăn trở với câu hỏi “Ích Thơ ca? Ích gì, Nghệ thuật?”

Ích gì?

Chế Lan Viên – Di cảo

Khéo giống bò sữa, hoạ mi, ngựa đua, gà chọi Khó gì? Ta khơng giữ, khơng ni mất

Giống nhà thơ vậy Tuyết non cao không thấy, Giống nàng tiên, ông Bụt mơ Mà chả cần giết

Chỉ thơi u chết

Chỉ cần bâng quơ, vu vơ đặt câu hỏi Trịnh trọng bâng quơ, vu vơ ấy Hỏi rằng: Ích họa mi?

Ích bị sữa? Ích xạ hương? Ích thi sĩ? Ích hơn? Ích giấc mơ? Ơng Bụt ích gì?

(8)

2 Đối tượng toán học: Tìm cội nguồn

Lo lắng Chế Lan Viên phải, lảng tránh câu hỏi “Ích gì”? Nhất Tốn học, nhìn sang “bên cạnh”, chưa có đặt câu hỏi: “Ích gì, Vật lý? Ích gì, Sinh học?”

Ích gì, Vật lý?Dễ trả lời thơi, vật lý học nghiên cứu vật chất chuyển động chúng khơng gian thời gian Có lại khơng cần kiến thức đó?

Ích gì, Sinh học?Dễ trả lời thơi, sinh học nghiên cứu thể sống tương tác chúng với mơi trường Có lại khơng cần kiến thức đó?

Nhưng “Ích gì, Tốn học? Tốn học nghiên cứu gì?” lại câu hỏi khơng dễ trả lời Để hiểu đối tượng Toán học, phải tìm cội nguồn Tức phải tìm đến “Cơ sở” Euclid Trước “Cơ sở” đời (khoảng300năm trước Cơng ngun), Tốn học chưa phải khoa học độc lập Nó “lẫn” vào Triết học Thiên văn học

Bắt đầu với “định nghĩa bản” đối tượng Toán học, Cơ sở - I, Euclid đưa ra23định nghĩa Xin nhắc lại ba số đó, định nghĩa thứ1, 2và15 :

α΄v Σημεῖόν ἐστιν, οὗ μέρος οὐθέν β΄v Γραμμὴ δὲ μῆκος ἀπλατές

ιε΄v Κύκλος ἐστὶ σχῆμα ἐπίπεδον ὑπὸ μιᾶς γραμμῆς περιεχόμενον [ἣ καλεῖται περιφέρεια], πρὸς ἣν ἀφ΄ ἑνὸς σημείου τῶν ἐντὸς τοῦ σχήματος κειμένων πᾶσαι αἱ προσπίπτουσαι εὐθεῖαι [πρὸς τὴν τοῦ κύκλου περιφέρειαν] ἴσαι ἀλλήλαις εἰσίν

Dịch nghĩa

1 Điểm khơng có kích thước.

2 Đường có chiều dài, khơng có chiều rộng.

15 Đường trịn hình phẳng gồm đường (gọi chu vi), (sao cho) mọi đường thẳng xuất phát (đến chu vi) từ điểm nằm bên hình nhau.

Như vậy, Tốn học nghiên cứu vật khơng tồn thực tế! Khơng thể tìm “vật thể” khơng có kích thước, khơng thể tìm khơng có chiều rộng Và hiển nhiên, đường trịn “lý tưởng” Tốn học khơng thể tồn tại, người ta nhìn thấy “vành nón trịn, mặt trời trịn”, hay “khn trăng đầy đặn” Th Vân trịn Mặt trăng! Vậy thì, ích gì, khoa học nghiên cứu vật khơng tồn tại? Tìm cội nguồn lại khơng cho ta câu trả lời, mà ngược lại, làm ta bối rối thêm Nhưng, phải khơng tồn thực tế vơ ích, cần phải biến sau câu hỏi “Ích gì?” Ta thử tìm Pablo Picasso, hoạ sĩ vĩ đại kỷ XX Người ta nhìn thấy Picaso khơng tranh ông để lại, mà vật dụng hàng ngày Ông làm thay đổi quan niệm đẹp Và điều kỳ diệu Picasso làm được, đưa ta tận chất vật

(9)

Tạp chí Epsilon, Số 09, 06/2016

Nhưng hiển nhiên chưa phải “con bị Picaso” hồn tồn giống bị ta nhìn thấy đấu trường Con bò tiếng Picaso “đơn giản”, “xấu” nhiều:

(10)

Như vậy, Picaso từ “con bị tót” đến “khái niệm bò”! Con bò “khái niệm” Picasso với sừng đáng sợ, với óc nhỏ “điểm” Euclid, thể đầy đủ sức mạnh ngu muội chủ nghĩa phát xít năm30.Hơn hai ngàn năm trước, Euclid cách từ “mặt trời tròn, vành trăng tròn” đến “đường trịn” tốn học

Cái khơng có giới thực lại mô tả chân thực giới thực tại, đưa ta với chất Phải lý giải thích việc lý thuyết tốn học cho ta cơng cụ mơ tả xác giới tự nhiên Nói Galilei, Thượng đế viết nên tự nhiên ngôn ngữ toán học

3 Nghề làm Toán

Nhiều người hỏi bác Tơm (René Thom, nhà tốn học Pháp, giải thưởng Fields) nghề làm Tốn Thấy khó nói q, bác kể chuyện săn rồng Chuyện rằng, xưa bên Trung Quốc, có anh chàng học nghề săn Anh chẳng chịu học săn hổ, săn lợn, mà lại học nghề săn Rồng! Nghề khó lắm, phải thực tập nhiều Bởi nên thạo nghề gian chẳng lấy Rồng nào! Có người hỏi: Bây sống nghề gì? Đáp: Đi dạy nghề săn Rồng! Bác Tơm nói: Làm Tốn tức dạy nghề săn Rồng vậy! (thảo chẳng có Rồng dám bén mảng đến nhà bác Tơm!)

Thế thì, làng nước đâu có cần anh săn Rồng Có cịn Rồng đâu mà học nghề săn rồng? Ấy chết, đừng vội nói Rồng chẳng cịn, có phải học nghề săn Rồng Nếu anh học nghề săn lợn bắn hổ? Mà học nghề săn hổ bắn voi? Nhưng thạo nghề săn Rồng hổ, báo, sư tử, voi, chắn săn tuốt! Này nhé, Rồng có thân cá sấu, móng vuốt hổ, đầu sư tử, ẩn trăn, mà cịn khơng tay anh săn Rồng, chẳng nói đến hổ, báo, voi, trăn, mà sau có “nhân bản” nữa, chẳng sợ! Thành ra, định học nghề săn học nghề săn Rồng!

Từ cá sấu, hổ, sư tử, trăn, người xưa “trừu tượng hóa” thành Rồng Cũng thế, từ thực tiễn, người ta trừu tượng hóa thành Tốn học Câu chuyện đơn giản bác Tơm mà thâu tóm mạnh, yếu, Toán học

Khi trừu tượng hố để tìm đến chất, Tốn học khơng phải dễ dàng trở với thực tại, vốn nơi xuất phát Thậm chí, người ta cịn nghi ngờ khả quay với thực

(11)

Tạp chí Epsilon, Số 09, 06/2016

Các phương trình, lý thuyết Tốn học đèn bác Tơm Có nó, người ta “làm Tốn” được, tức tìm quy luật vật Muốn trở với thực tiễn trước tiên phải biết rời xa thực tiễn, để khơng cịn bị che lấp rườm rà, không chất đời thường

Ba trăm năm trước bác Tơm, Newton nói: “Khơng có gần với thực tiễn lí thuyết đẹp!”

4 Ứng dụng Toán học

Nhưng câu hỏi “Ích gì, Tốn học?” lởn vởn đâu đây, nhìn nhà tốn học hàng đầu nghiên cứu thứ hoàn toàn “xa rời thực tế” , mà thân họ chưa biết đến đâu

Người ta thường hỏi nhà Tốn học: Lí thuyết anh ứng dụng vào đâu? Khơng phải lúc có câu trả lời Vào kỉ thứ III trước Công nguyên, hỏi Apolonius nghiên cứu đường cơnic (nhận cách cắt mặt nón mặt phẳng) để làm gì, Apolonius khơng trả lời Ơng ta nghiên cứu đường cơnic thấy chúng “đẹp” Khơng Apolonius khơng thể trả lời, mà chục kỉ sau không trả lời Phải chờ đến Kepler Newton, tức 20 kỉ sau, người ta biết ông già Apolonius làm trò chơi với quỹ đạo chuyển động hành tinh! Chính bị ám ảnh đường cônic từ thuở ấu thơ mà Kepler nghi ngờ kết luận người trước quỹ đạo tròn, đưa giả thuyết quỹ đạo đường ellip, với hai tiêu cự gần Giả thuyết Newton chứng minh, với định luật vạn vật hấp dẫn “Cái đẹp”, từ chỗ khơng biết để làm gì, tìm thấy ứng dụng vào loại vĩ đại lịch sử, sau hai ngàn năm

Bác Tơm có lần nói: Đối với người mở đường, đừng hỏi họ đâu, người ta biết đâu, người ta không xa “quand on sait òu va, on va pas loin” Thật thế, anh định đến Thành phố Hồ Chí Minh anh đến Cà Mau Ngay anh Armstrong, biết đến Mặt trăng đến thơi, Cịn bác Tơm chẳng biết đâu, nên bác xa hơn, đến tận Hỏa, hay miền đất khoa học Và chúng ta, dù không xa bác Tôm, muốn ngày mai có bát cơm ngon, đừng q sốt ruột hơm chưa “ra ngơ, khoai” gì! Cịn muốn “ra ngơ, khoai” có đời biết ăn ngô, ăn khoai!

Vậy nhưng, lý thuyết Toán học phải cần đến2000năm sau có ứng dụng, câu hỏi “Ích gì, Tốn học?” dễ nhận câu trả lời “Vơ ích”! Dù “nhìn xa” đến mấy, người ta khó nhìn đến tận 2000năm sau!

Nhưng Toán học vào thực tiễn với đường khác Có khi2000năm, có hai năm, chí cần hai tháng! Ví dụ tiếng hệ mật mã khố cơng khai, hệ mã RSA hay hệ mã dùng đường cong elliptic Từ trang giấy nhà nghiên cứu toán học đến ứng dụng vào điện thoại thơng minh hay thẻ tín dụng gần tức thời

(12)

người hỏi Rodin: “Làm mà ơng tạc nên tượng tuyệt vời đến vậy?” Rodin trả lời: “Đơn giản thôi, lấy khối đá, thấy thừa đẽo đi!”

Nhưng, sau tất điều nói, tồn dai dẳng câu hỏi: ”Ích gì, Tốn học”? Người ta hàng ngày dùng điện thoại di động để nói đủ thứ chuyện, đơi để nói vơ ích Tốn học Người ta hàng ngày dùng thẻ tín dụng để chuyển tiền, rút tiền Nhưng khơng có điện thoại thơng minh, khơng có thẻ tín dụng khơng có mật mã khố cơng khai, khơng có Tốn học Vậy người ta ngại dùng tiền đầu tư cho Tốn học, “Ích gì, Tốn học?” Khi dùng điện thoại, rút tiền, khơng thấy “tích phân, vi phân, tổ hợp hay số học” Nói khác đi, Toán học đến mức “trong suốt” người sử dụng (tất nhiên dùng cho tất người)

Xem ra, “trong suốt” lại che khuất tầm nhìn núi!

Năm1674Mayow tìm thấy khí chất giúp cho sống Năm1773Karl Scheele lần lập chất khí Antoine Lavoisier lặp lại thí nghiệm Scheele gọi “oxygen” Như vậy, oxy “tìm ra” muộn Tại sao? Vì suốt Người ta khơng nhận thấy cần đến oxy Và khơng chịu bỏ tiền “cho nó” Phải đến người nhìn thấy hình ảnh sau đây:

Đó khơng khí Bắc Kinh Nó khơng cịn suốt Và người ta buộc phải nhìn thấy nó, buộc phải họp Rio de Janeiro, Kyoto, Paris để bàn tìm cách bỏ tiền làm cho suốt trở lại Giá người ta nhìn thấy cần thiết từ cịn suốt!

5 “Tính” “Tốn”

(13)

Tạp chí Epsilon, Số 09, 06/2016

trị Tốn học phát triển cơng nghệ, tơi nghĩ rằng, đến lúc công nghệ phát triển để giải phóng người, cho họ thời gian quay với thơ ca, âm nhạc Toán học” Phải chăng, Hirzebruch muốn ám rằng, Tốn học có hai phần: “Tính” “tốn”

Nếu tính cần thiết cho cơng nghệ, Tốn, ngồi chức phát triển phần tính ra, cịn góp phần làm nên Con Người, giống âm nhạc, nghệ thuật thơ ca

Nhưng có thể5năm nữa, dịp kỷ niệm10năm VIASM, lại phải bàn câu hỏi: “Ích gì, Tốn học?”

(14)(15)

HÀM MOEBIUS VÀ ĐỊNH LÝ PHẦN DƯ TRUNG HOA

Phùng Hồ Hải (Viện Toán học Việt Nam)

Câu lạc (CLB) Toán học Viện toán học thành lập cách đây5năm Đầu tiên CLB sinh hoạt sáng chủ nhật tuần với giảng toán GS, TS, thầy giáo chuyên toán dành cho học sinh THPT Sau để tạo điều kiện cho học sinh tỉnh xa, CLB tổ chức đợt giảng dài ngày hình thức trường đơng, trường hè, trường xuân Gần CLB toán học Viện toán học tổ chức thêm lớp học dành cho học sinh THCS với mục tiêu định hướng tạo nguồn Epsilon xin giới thiệu với bạn đọc viết GS Phùng Hồ Hải kể lại buổi dạy ngẫu hứng vào chiều thứ ba7/6/2016vừa qua

Bài viết lấy từ trang facebook cá nhân giáo sư Phùng Hồ Hải

Để hiểu nội dung viết, ta cần đến định nghĩa hàm Moebius Hàm Moebiusµ(n)được xác định tập hợp số nguyên dương nhận giá trị tập hợp{−1,0,1}.Ta cóµ(1) =

Khin >1, µ(n)sẽ bằng0nếuncó ước phương lớn hơn1.Trong trường hợp

nkhơng có ước phương (lớn hơn1) vớin =p1p2· · ·prlà tích số ngun tố

phân biệt ta cóµ(n) = (−1)r.Ví dụ ta có bảng giá trị hàm Moebius vớin 615 n 10 11 12 13 14 15

µ(n) −1 −1 −1 −1 0 −1 −1 1

Lâu không report Math Circle Hơm tơi có buổi dạy ngẫu hứng Hàm Moebius nhắc tới vài lần Hôm muốn thuyết phục bạn lớp hàm bằng0tại “phần lớn” số tự nhiên Nói phải có sách, mách phải có chứng Ta thử liệt kê xem hàm triệt tiêu giá trị

Ta biết hàm triệt tiêu tạinnếunchia hết cho bình phương số nguyên tố Như trong10số tự nhiên đầu tiên, hàm triệt tiêu tại4, 8, 9- ba số

Trong10số tự nhiên hàm triệt tiêu tại12,16,18,20- bốn số Tình hình sau chút:32, 36, 40.Rồi lại lến:44, 45,48, 49, 50

Dù bạn lớp thấy tình hình có chiều hướng tốt lên, nghĩa ngày tỷ lệ số mà hàm triệt tiêu tăng Đặc biệt ta thấy xuất ba số liên tiếp mà hàm triệt tiêu:48, 49, 50

- Nếu tiếp tìm thấy10số tự nhiên liên tiếp mà hàm Moebius triệt tiêu Thầy tuyên bố

Khẳng định làm cho bạn học sinh ngạc nhiên - Chúng ta thấy có ba số liên tiếp Bạn chứng minh tồn bốn số liên tiếp mà hàm triệt tiêu?

Câu hỏi khó

(16)

Ngay có lời giải: xét hai sốa2−1vàa2,hàm hiển nhiên triệt tiêu tạia2,chỉ cần chọna

lẻ thìa2−1chia hết cho4

- Thế ba số sao, liệu có tồn vơ hạn ba số tự nhiên liên tiếp mà hàm Moebius triệt tiêu đó?

Sau lúc trao đổi bạn đề xuất lời giải này: xét sốa3−1, a3, a3+ 1.Hàm Moebius tất nhiên triệt tiêu tạia3.Ta tìmasao choa3−1chia hết cho4còna3+ 1chia hết cho9.Tuy

nhiên lời giải chưa chặt chẽ

Thầy giải thích cho bạn rằnga3 −1chia hết cho4khi khia−1chia hết cho4còn

a3+ 1chia hết cho9khi khia+ 1chia hết cho3.Vậy toán sốanào thỏa

mãn điều kiện này?

Đây toán dễ bạn, câu trả lời có ngay:achia12dư5, haya= 12k+

- Bài tốn tìmađể ađồng dư với3mod4và đồng dư với2mod3chính tốn “giải hệ phương trình đồng dư bậc nhất”, thầy giải thích cho bạn Mọi người phấn khích

- Bây ta xem nguyên tắc giải hệ nào: Trước hết ta tìm “nghiệm riêng”, số5,từ ta cần cộng thêm “nghiệm tổng quát”, số12- bội chung nhỏ của3và4

Quá dễ hiểu bạn học sinh

- Bây ta xét hệ phương trình xem Chẳng hạn:ađồng dư với1mod3,đồng dư với2mod4và đồng dư với4mod5

a= 34 + 60k,chỉ vài phút bạn có câu trả lời

- Chúng ta giải hệ “dần dần”, xét hai phương trình ta có nghiệm:ađồng dư với10mod12,sau ta lại xét phương trình với phương trình thứ ba,ađồng dư với

4mod5,để suy nghiệm phương pháp tương tự

- Bằng cách ta chứng minh Định lý phần dư Trung Hoa: Mọi hệ phương trình đồng dư bậc với modun đôi nguyên tố có nghiệm Chỉ cần xét dần dần, lần hai phương trình

- Bạn biết hệ hai phương trình đồng dư bậc với modun ngun tố ln có nghiệm?

- Sử dụng hệ thức Bezout, bạn trả lời

Thế định lý phần dư Trung Hoa chứng minh, “về nguyên tắc” Trở lại toán ban đầu, phương pháp sử dụnga3 rồi thêm bớt1không thể cho ta lời giải cho câu hỏi: Tìm10số

(17)

DẪN NHẬP VỀ HÀM ZETA RIEMANN VÀ PHÉP BIẾN ĐỔI MELLIN

Ngô Bảo Châu (Đại học Chicago)

Đọc lịch sử toán học ta thấy trước thư tín phương tiện trao đổi thơng tin tốn học quan trọng Rất nhiều ý tưởng, giả thuyết, dự đoán chứng minh hay phản ví dụ trình bày lần thư khơng phải tạp chí sách Newton, Leibniz, Fermat, Euler, Gauss, Jacobi, Bernoulli, Mersenne, có “cơng bố” dạng thư tín

Ngày nay, với phương tiện giao tiếp đại blog, trang wordpress, archiv ta có nhiều cách để “cơng bố” ý tưởng hay tiền ấn phẩm Tuy nhiên, trao đổi học thuật thư tín, email hình thức thông dụng hiệu

Thông tin qua email khơng hình thức chặt chẽ báo, sách, có nhiều ý tưởng cịn thơ ráp, nơm na, điều làm độc giả dễ đọc hơn, gần gũi hơn, dễ hiểu

Chúng xin giới thiệu với bạn đọc thư GS Ngô Bảo Châu gửi cho GS Phùng Hồ Hải nhân việc GS Hải muốn tổ chức khóa học hàm zeta phép biến đổi Mellin để chuẩn bị cho chuỗi giảng lý thuyết biểu diễn GS Ngô Bảo Châu GS Phạm Hữu Tiệp vào tháng8/2016

Dịch Nguyễn Vũ Duy Linh Trần Nam Dũng hiệu đính

From:

Đại Học CHICAGO Khoa Toán

5734 University Avenue, Chicago IL 60637 Ngô Bảo Châu

To:

GS Phùng Hồ Hải Viện Toán học Việt Nam 18 Hoàng Quốc Việt, Hà Nội

Ngày5tháng5năm2016

Hải thân mến,

Tổ chức seminar học tập hàm zeta ý tưởng tốt

Tơi thiết nghĩ nên mở đầu vài vấn đề lý thuyết số giải tích Chúng ta xem xét hàm số học tức đơn dãy sốa1, a2, định nghĩa theo kiểu

số học Đây khái niệm trừu tượng người ta làm mức độ trừu tượng Trên phương diện khác, ta có nhiều ví dụ cụ thể:

(18)

2 an =µ(n)là hàm Moebius,

3 an =d(n)là số ước số củan,

4 an =r(n)là số cách biểu diễnnnhư tổng hai bình phương,

5 an = 1nếunnguyên tố bằng0nếu ngược lại,

6 an = log(p)nếunlà lũy thừa củapvà0nếu ngược lại,

Mặc dầu dãy số bất thường, ngoại trừ dãy đầu tiên, tổng Cesaroa1+a2+· · ·+an

có tính tiệm cận Chẳng hạn xác định tính tiệm cận dãy thứ5,chúng ta thử tính số lượng số nguyên tố nhỏ số

Ước lượng tổng Cesaro củad(n)vàr(n)là vấn đề sơ cấp cổ điển lý thuyết số Đối vớir(n),đó tốn hình trịn Gauss: Có điểm ngun nằm hình trịn bán kính tiến về∞.Bằng lý luận sơ cấp Gauss làm, có

X

n<X

r(n) =πX +O(X1/2) (0.1)

Seminar bắt đầu với tốn người ta khơng cần có kiến thức để hiểu cách giải Nó giải thích kỹ lưỡng chương6của “Nhập mơn lý thuyết số giải tích” Chandrasekharan Tại chỗ này, ta cần đề cập sai sốO(X1/2)có thể cải thiện đáng kể nhờ cơng cụ giải tích tốt Chẳng hạn, nhờ cơng thức lấy tổng Poisson, người ta đạt đượcO(X1/3).

Cách truyền thống để hiểu tính tiệm cận tổng Cesaro tạo chuỗi Dirichlet

D(s, a) = X n

ann−s,

với biến phứcs.Bằng cách áp đặt điều kiện tăng trưởng choan,chuỗi Dirichlet hội tụ

nửa mặt phẳng<(s)> αnào Chẳng hạn chuỗi Dirichlet tương ứng với dãyan= 1chính

Riemannς-functionς(s, a) = P

nn

−s hội tụ miền<(s)>1.

Có vài thao tác mà nên làm quen làm việc với chuỗi Dirichlet Chẳng hạn, bốn ví dụ đầu tiên,ancó tính chất nhân tính:amn =amanvới(m;n) = 1.Chuỗi Dirichlet

tương ứng phân tích thành nhân tử thành tích Euler tức tích thừa số đánh số số nguyên tố Chẳng hạn như:

ς(s) =Y p

(1−p−s)−1 (0.2)

Một nhận xét khác theo lối tương tự chuỗi Dirichlet tương ứng với tích chập hàm số học Chẳng hạn

X

n

d(n)ann−s =ς(s)2, (0.3)

X

n

µ(n)ann−s =ς(s)

−1

(19)

Tạp chí Epsilon, Số 09, 06/2016

Những đẳng thức hiển nhiên xem xét chuỗi Dirichlet hình thức Như hàm phức, chúng chứng minh dễ dàng miền mà chuỗi Dirichlet hội tụ tuyệt đối Tất điều giải thích hồn hảo chương2của “Nhập mơn lý thuyết số giải tích” Apostol

Một hướng lý thuyết số giải tích cổ điển dẫn ước lượng tiệm cận tổng Cesaro từ cực chuỗi Dirichlet Đó nội dung định lý Tauberian Một vấn đề khó chịu có nhiều phiên định lý Tauberian Phiên đơn giản định lý Wiener-Ikehara’s giải thích chương11của Chandrasekharan chẳng hạn Trong trường hợp nào, định lý Tauberian cho ta biết điều sau: Giả sử

D(s, a) = P

nann−s thác triển phân hình vượt qua đường thẳng<(s) = 1−ε.Giả sử

rằng có cực cấp tại2và cực cấp1tại1.Khi biểu thức tiệm cận có dạng:

X

n<X

an =c2,2X2log(X) +c2,1X2 +c1,1X+· · · (0.5)

trong đóc2,2, c2,1 hệ số Laurent khai triểnD(a, s)thành chuỗi Laurent tạis= .Ở

số mũ2trongX2tương ứng với vị trí cực tại2,số hạnglog(X)xuất cực tại2có cấp2

Mất nhiều thời gian cho định lý Tauberian khơng hay phương diện vấn đề kỹ thuật, phương diện khác phần giải tích ỏi cần dùng để chứng minh định lý Tauberian khơng thích đáng xét quan điểm lý thuyết số

Nếu tơi khơng lầm có cách tiếp cận tốt sử dụng gọi “tổng trơn” Người ta đọc post Emmanuel Kowalski blog ông ta để giới thiệu tổng trơn:

https://blogs.ethz.ch/kowalski/smoothing-sums-wiki-page/

Cách tốt để khởi đầu với tổng trơn quay với tốn hình trịn Gauss Chúng ta phát biểu lại tốn hình trịn Gauss tổng qt sau: Cho hàmφ :R2 →C,chẳng hạn hàm

đặc trưng đĩa đơn vị, muốn biết tiệm cận

X

n∈Z2

φ(tn) (0.6)

khit→0.Nếuφlà hàm đặc trưng đĩa đơn vị, X

n∈Z2

φ(tn)sẽ số điểm nguyên

bên đĩa bán kínht−1

Điểm yếu chiến lược “tổng trơn” nhận thay làm việc với hàm thử “thô”, hàm đặc trưng đĩa đơn vị có bước nhảy thơ chu vi đường tròn, tốt làm việc với hàm thử trơn với giả thiết compact Đối với hàm trơn, tiệm cận củaP

n∈Z2φ(tn)là hệ trực tiếp công thức lấy tổng Poisson: X

n∈Z2

φ(tn) =t−1 X

n∈Z2

ˆ

φ(t−1n) (0.7)

trong đóφˆlà biến đổi Fourier củaφBiến đổi Fourierφˆkhơng cịn hỗ trợ compact nữa, hàm giảm nhanh không gian Schwartz Ở vế phải, khit−1 → ∞,mọi số hạng,

ngoại trừn = 0,giảm nhanh Do ta có

X

n∈Z2

(20)

Số hạngφˆtương ứng với diện tích đĩa, nơi xuất sốπtrong tốn hình trịn Gauss Mặc dù vậy, áp dụng trực tiếp công thức lấy tổng Poisson cho hàm đặc trưng đĩa đơn vị, gặp rắc rối to điểm kỳ dị lớn củaφsẽ dẫn tới tăng nhanh biến đổi Fourierφ.ˆ Tốt xấp xỉ hàm đặc trưng đĩa hàm trơn cố gắng khống chế sai số Làm theo đường lối thông minh, nhận sai sốO(t−1/3).Tôi nhớ thấy lối giải thích đẹp tơi khơng tìm thấy nữa.

Nhưng tập thật tốt cho tham dự seminar anh

Một học rút từ phép giải tốn hình trịn Gauss qn hàm thử thơ thay vào làm việc hàm trơn Phần lý thuyết hồn tồn nói hàm thử trơn Đối với toán cụ thể, người ta phải cất công xấp xỉ hàm thử thô hàm trơn Điều cho ta hình dung tốt hàm số học chuỗi Dirichlet theo nghĩa phân bố Mỗi dãyantăng vừa phải xem phân bố

φ7→a(φ) =X n

anφ(n) (0.9)

Tích chập hàm số học chuyển thành tích chập phân bố Đồng thời chuỗi Dirichlet

P

nann−s biến đổi Mellin phân bốφ → a(φ).Chẳng hạn biến đổi Mellin

phân bốφ7→P

nφ(n)là hàm Riemann zetaς(s).Ta cần phải quen với phân bố

hiểu rõ biến đổi Mellin phân bố Cách làm có phần thưởng lớn: Người ta tước bỏ ước lượng trung gian cồng kềnh giải tích Fourier xử lý thứ tính tốn đại số

Người ta định nghĩa biến đổi Mellin phân bố loạianhư sau Đối với hàm Schwartzφ,tích chậpa∗φlại hàm Schwartz Cả hai biến đổi Mellin củaφvàa∗φvẫn hàm trơn với tiệm cận vừa phải Khi định nghĩa biến đổi MellinM(φ)

vàM(a∗φ)rồi choM(a) =M(a∗φ)/M(φ)

Ý tưởng nhiều người biết, không nhớ rõ giải thích tài liệu Ít giải thích thư ngắn tơi gởi cho anh trước hàm ς Nếu không anh đọc phép biến đổi Mellin giảng Igusa

trong TIFR:http://www.math.tifr.res.in/publ/ln/tifr59.pdf.Zagiercó

viết giới thiệu sơ cấp hơn:http://people.mpim-bonn.mpg.de/zagier/files/ tex/MellinTransform/fulltext.pdf

Bây người ta tiếp cận hàm Riemann zeta theo lối phân bố Cơng thức lấy tổng Poisson phát biểu sau: phân bố lược Diracφ 7→ σZ(φ) =

P

n∈Z

φ(n),là phép biến đổi Fourier Bây tính biến đổi Mellin biến đổi Fourier củaσZ ta có phương trình

hàm hàm Riemann zeta Trong q trình tính tốn, người ta phải chứng minh rằngς có thác triển phân hình dựa tác động lẫn phép biến đổi Mellin, tiệm cận tổng Cesaro công thức lấy tổng Poisson

Một tập tốt thử tổng quát hóa lý thuyết hàm zeta Riemann giải thích mở rộng bậc haiF củaQ.Chẳng hạn trường hợp tách đượcF =Q×Q,

chúng ta cần phải quay trở lại dãyan=d(n).Nếu nhưF =Q[i]là trường hữu tỉ Gauss, chúng

ta cần phải quay trở lại tốn hình trịn Gauss

(21)

Tạp chí Epsilon, Số 09, 06/2016

trả lời phép lấy tổng Poisson làm việc với tổng có số ngun khơng thể làm với tổng có số số nguyên tố Chẳng hạn phép lấy tổng Poisson khơng thể làm việc với dãy (0.5) (0.6) Nói cách khác phép lấy tổng Poisson khơng thể chứng minh định lý số ngun tố

Đối với định lý số nguyên tố, nói chung cần phải xử lý sau: Chúng ta bắt đầu với dãy (0.1), tạo chuỗi Dirichlet hàm zeta Riemann Chúng ta dùng cơng thức lấy tổng Poisson để chứng minh rằngς có thác triển phân hình phương trình hàm Chúng ta suy rằngdlogς có thác triển phân hình phương trình hàm Giờ đâydlogς chuỗi Dirichlet dãy (0.6) Chúng ta hồn tất cơng việc biết cực củadlogς.Nhưng cực củadlogς khơng điểm củaς,đó lý muốn định vị không điểm củaς

Chẳng hạn, định lý số nguyên tố dựa kiện làς khơng điểm miền mở chứa<(s)>1,điều chứng minh Hadamard trăm năm trước

Nếu anh có đủ thời gian can đảm, anh nghiên cứu lại tất thứ định lý Dirichlet số nguyên tố cấp số cộng Đó chỗ mà cần luật thuận nghịch Nhưng anh khơng cần bắt đầu với luật thuận nghịch thật khơng phải hướng phát triển ban đầu hàm zeta, theo ý kiến Tất nhiên sau luật thuận nghịch trộn lẫn nhiều vào lý thuyết hàm zeta chương trình Langlands, lúc học thứ dễ dàng

(22)(23)

GIẢI NOBEL CỦA EINSTEIN HAY LÀ SÓNG ĐIỆN THOẠI CÓ GÂY UNG THƯ HAY KHÔNG

(Đàm Thanh Sơn - ĐH Chicago)

Bài viết tham khảo từblogcủa giáo sư Đàm Thanh Sơn

Albert Einstein có lẽ nhà vật lý tiếng từ trước đến Công chúng thường biết đến ông ta người khám phá thuyết tương đối, làm thay đổi quan niệm không gian thời gian Thuyết tương đối bao gồm thuyết tương đối hẹp, Einstein tìm năm

1905và thuyết tương đối rộng ơng ta tìm ra10năm sau Tuy nhiên khơng phải biết giải thưởng Nobel vật lý năm1921được trao cho Einstein khám phá khác ơng: Hiệu ứng quang điện Đây cơng trình Einstein viết vào năm1905,cùng năm với cơng trình thuyết tương đối hẹp cơng trình chuyển động Brown Hiệu ứng quang điện đóng góp lớn Einstein vào thuyết lượng tử, lý thuyết mà sau Bohr, Heisenberg, Schrăodinger v nhiu ngi khỏc phỏt trin lờn lại bị Einstein nghi ngờ đến cuối đời

(24)

Einstein giải thích điều cách áp dụng mở rộng giả thuyết lượng tử Planck Einstein giả thuyết ánh sáng bao gồm hạt photon, hạt mang lượng tỉ lệ thuận với tần số ánh sáng

E =hv

Ở đâyElà lượng hạt photon,vlà tần số ánh sáng, vàhlà số Planck Công thức có tên cơng thức Planck, cơng thức mà theo phải tiếng công thứcE =mc2

Hiệu ứng quang điện trình hạt photon truyền lượng cho hạt điện tử Để bứt điện tử khỏi mảnh kim loại ta cần lượng tối thiểu định, ta gọi là∆.Như khiv > ∆h ánh sáng bứt điện tử khỏi khối kim loại Nếuv < ∆h nguồn sáng có mạnh khơng có photon đủ lượng để gây hiệu ứng quang điện Bạn hỏi liệu có hai hạt photon, nhiều hơn, hợp sức để bứt điện tử hay không Điều nguyên tắc xảy ra, xác suất thấp, bỏ qua Hiệu ứng quang điện có liên quan trực tiếp đến câu hỏi hay đặt nay: Điện thoại di động có gây tác hại cho sức khoẻ hay khơng? Một điều làm nhiều người lo lắng khả gây ung thư sóng điện thoại (ví dụ xembài này) Nhiều người cịn nói sóng điện từ lị vi sóng gây ung thư

Nếu ta nhớ lại công thứcE =hvcủa Einstein ta thấy lo lắng khơng có sở Đó tần số sóng thiết bị điện tử thấp để gây biến đổi phân tử ADN Tần số sóng lị vi sóng là2500MHz, tần số điện thoại di động là800MHz hay1900MHz Hằng số Planck là4×10−9 eV/MHz, vậy2500MHz tương đương với năng

lượng10phần triệu eV, q trình hố học hay sinh hố cần lượng cỡ

0.1eV, khơng phải là1eV Sự chênh lệch đến10−100nghìn lần hai cỡ lượng làm cho lị vi sóng hay điện thoại di động làm biến đổi gien miếng thịt để lò hay thể (Tia cực tím lại khác, tần số tia cực tím cao tần số điện thoại di động đến triệu lần, nên có đủ lượng để gây tác hại cho tế bào)

(25)

HỆ MẬT MÃ KHĨA CƠNG KHAI DỰA TRÊN ĐƯỜNG CONG ELLIPTIC - TỔNG QUAN VỀ

HỆ MẬT MÃ KHĨA CƠNG KHAI

Đặng Minh Tuấn (Vietkey)

Trong số này, Epsilon trân trọng gửi đến độc giả phần chuyên đề hệ mật mã khóa cơng khai dựa đường cong Elliptic tác giả Đặng Minh Tuấn Phần sau chuyên đề chuyển tải tạp chí số 10

Sau toàn văn lời mở đầu chương loạt chuyên đề

LỜI MỞ ĐẦU

Tháng năm 2016, Bộ Ngoại Giao Hoa Kỳ, đứng đầu trưởng John Kerry, dẫn đồn đại biểu tới nước ASEAN có Việt Nam để thảo luận phát triển Fintech đặc biệt công nghệ Blockchain1.Tháng năm 2015, Ủy

ban giao dịch hàng hoá tương lai Mỹ cơng bố, Bitcoin thức đưa vào danh sách hàng hóa phép giao dịch Mỹ2.Cơng nghệ Blockchain Bitcoin công nghệ tiền số đời năm 2009 ngày có nhiều quốc gia tổ chức, doanh nghiệp cho phép lưu hành tốn loại tiền số khơng gian mạng Internet toàn cầu Tháng 4-2016, giá trị thương mại Bitcoin lên đến 6.5 tỷ USD3 Nền tảng sở Bitcoin lý thuyết mật mã mà cụ thể

ở hàm băm lý thuyết chữ ký số dựa Hệ mật đường cong Elliptic (ECC)

Bên cạnh việc sử dụng tiền số Bitcoin4, ECC ứng dụng rất

nhiều thực tiễn ngành Công nghệ thông tin [1] Các trang Web bảo mật https (http-secure) thường dùng toán điện tử hay ứng dụng riêng tư gmail sử dụng giao thức TLS (Transport Layer Security) mà trước

1B Cohen (April 12, 2016) U.S State Department Recommends Development of Blockchain and Dis-tributed Ledgers to International Partners [Online] Available: http://www.nasdaq.com/article/us-state-department-recommendsdevelopment-of-blockchain-and-distributed-ledgers-to-international-partnerscm605334

2(21/09/2015) Bitcoin thức Mỹ cơng nhận hàng hoá [Online] Available: http://vnreview.vn/tin-tuc-kinh-doanh/-/view-content/content/1654484/bitcoin-chinh-thuc-duoc-My-cong-nhan-la-hang-hoa

3(15/4/2016) https://markets.blockchain.info/

4Địa ví Bitcoin tính dựa khóa cơng khai ECC với hàm băm bảo mật có độ dài 256-bit SHA256 thuật toán Base58Encode dùng để chuyển số thành dạng 56 ký tự, RIPEMD (RACE Integrity Primitives Evaluation Message Digest) họ hàm băm bảo mật khác:

Version= 1(byte)

KeyHash=Version+RIPEMD(SHA256(PublicKeyEC)) CheckSum=SHA256(SHA256(KeyHash))

(26)

SSL (Secure Socket Layer) Trong giao thức ECC sử dụng để trao đổi khóa phiên Các giao dịch remote access sử dụng nhiều giới Unix, Linux SSH (Secure SHell) sử dụng ECC để trao đổi khóa Ưu điểm hệ mật sử dụng đường cong Elliptic (ECC) có độ dài khóa nhỏ (160 bit tương đương với khóa độ dài 1024 Bit hệ mật RSA), sử dụng độ dài khóa nhỏ nên tài nguyên phục vụ cho ECC thường nhỏ nhiều, bên cạnh hiệu tính tốn nâng cao rõ rệt Hiện ECC xu để thay RSA

Cơ sở toán học hệ mật ECC nhóm giao hốn Abel điểm nằm đường cong Elliptic Ngoài việc đường cong Elliptic sở cho hệ mật ECC, hệ mật ID-Based, đường cong Elliptic (EC) cịn cơng cụ hữu hiệu để phân tích số nguyên thừa cố nguyên tố [2,3,4], dùng để kiểm tra tính nguyên tố số nguyên [3] EC sở để chứng minh định lý Fermat tiếng tồn nhiều trăm năm qua

Đường cong Elliptic trường hợp đặc biệt phương trình Diophant Lý thuyết đường cong Elliptic (EC) phong phú đồ sộ Trong [5] tác giả Serge Lang phát biểu phương diện học thuật: “Có thể viết vơ tận đường cong Elliptic” Các lý thuyết khái niệm liên quan tới EC liệt kê số đây:

• Lý thuyết nhóm, vành, trường đại số trừu tượng [3,6];

• Đa tạp Affine, đa tạp Jacobian đa tạp xạ ảnh hình học đại số [7,8];

• Điểm Torsion, Divisor, cặp song tuyến tính Weil, Tate-Lichtenbaum [3];

• Lý thuyết trường Galois, tự đồng cấu-ánh xạ Frobenius [3,9];

• Lý thuyết Baker-Feldman, Baker-Tijdeman lý thuyết Kummer [5];

• Số p-adic, Isogenies, hàm Sigma hàm Zeta [5,7,10,3,4];

• Nhóm đối đồng điều, đối đồng điều Galois đối đồng điều phi giao hốn (Topo đại số) [8,4];

• Nhóm Mordell–Weil, Selmer nhóm Shafarevich–Tate [8,11];

• Phương pháp hình học Tựa tuyến tính (Quasilinear) [12]

Với ý nghĩa to lớn thực tiễn học thuật, EC tảng toán học quan trọng đại số đại lý thuyết mật mã đại EC tảng quan trọng phủ điện tử thương mại điện tử Chính điều mà Chun đề “Hệ mật mã khóa cơng khai dựa đường cong Elliptic” lựa chọn để trình bày báo cáo

(27)

Tạp chí Epsilon, Số 09, 06/2016

Phạm vi chuyên đề giới hạn với khái niệm lý thuyết đủ cho ứng dụng EC, phát triển EC thành hệ mật ID-Based, ứng dụng chữ ký số tập thể, chữ ký số nhóm, chữ ký ngưỡng, ký ủy nhiệm, ký số mù không đề cập đến khuôn khổ báo cáo

Báo cáo chuyên đề kết cấu thành 02 chương, chương trình bày khái niệm, định nghĩa đường cong Elliptic (Phương trình EC, nhóm cộng Abel điểm đường cong, chứng minh định lý nhóm ) Chương trình bày Hệ mật dựa đường cong Elliptic số ứng dụng mã hóa, xác thực chữ ký số, trao đổi khóa dự tốn khó Logarithm rời rạc

Ký hiệu

N tập hợp số tự nhiên

N∗ tập hợp số tự nhiên khác0

Z trường số nguyên

Q trường số hữu tỉ

char(K) đặc số trườngK

≡ dấu đồng dư

∞ dương vô (tương đương với+∞)

gcd ước số chung lớn

deg bậc đa thức

det định thức

RSA Hệ mã cơng khai dựa tốn phân tích thừa số nguyên tố Rivest-Shamir-Adleman phát triển

EC Đường cong Elliptic (Elliptic Curve)

ECC Hệ mật dựa đường cong Elliptic (Elliptic Curve Cryptography) ECDLP Elliptic Curve Logarithm Problem

ECDH Thuật toán Elliptic Curve Diffie–Hellman ECDSA The Elliptic Curve Digital Signature Algorithm ECIES The Elliptic Curve Integrated Encryption System ECMQV Elliptic Curve Menezes–Qu–Vanstone protocol

1. Tổng quan đường cong Elliptic

Năm 250 sau Công ngun, Diophant giải tốn tìm số tầng tháp cầu mà trải mặt đất xếp thành hình vng dẫn đến giải phương trình (ylà số cầu cạnh hình vng;xlà số tầng tháp):

y2 = 12+ 22+ 32+· · ·+x2 = x(x+ 1)(2x+ 1)

Phương trìnhy2 =x(x+ 1)(2x+ 1)/6là dạng đường cong Elliptic

Năm 1637, nhà toán học vật lý học người Pháp Pierre de Fermat công bố định lý Fermat cuối viết lề copy công trình Diophant: Phương trình sau vơ nghiệm:

(28)

Hơn ba kỷ, có nhiều nhà toán học cố gắng chứng minh định lý xong thất bại, năm 1994, Andrew Wiles, giáo sư trường Princeton gây tiếng vang lớn cộng đồng toán học giới vào thời điểm sử dụng đường cong Elliptic có dạng

y2 =x(x−an)(x+bn)cùng với lý thuyết Modul để chứng minh định lý Fermat cuối cùng.

Năm 1987, Trong [14], Lenstra đề xuất thuật tốn phân tích số nguyên thừa số nguyên tố sử dụng đường cong Elliptic, thuật tốn tương đối nhanh, chạy với thời gian hàm mũ thuật toán nhanh thứ việc phân tích thừa số nguyên tố, sau phương pháp sàng đa thức toàn phương phương pháp sàng trường số tổng quát

Trong lĩnh vực mật mã, vào năm 1985, Victor S Miller công bố báo ứng dụng đường cong EC mật mã “Use of Elliptic Curves in Cryptography” [15] sau Neal Koblitz với “Elliptic curve cryptosystem” [16] vào năm 1987 Từ có nhiều cơng bố nghiên cứu EC lý thuyết thực tiễn ngày ứng dụng ECC sử dụng rộng rãi, đưa thành tiêu chuẩn

Một số tiêu chuẩn liên quan đến đường cong Elliptic:

IEEE 1363: Tiêu chuẩn bao gồm gần tất thuật tốn hệ khóa cơng khai có ECDH, ECDSA, ECMQV ECIES Trong phần phụ lục có thuật tốn lý thuyết số liên quan đến hệ mật khóa cơng khai

ANSI X9.62 X9.63: Các chuẩn tập trung vào đường cong Elliptic cụ thể ECDSA X9.62 ECDH, ECMQV ECIES X9.63 Các chuẩn xác định khuôn dạng liệu danh mục đường cong khuyến cáo sử dụng

FIPS 186.2: Tiêu chuẩn NIST cho chữ ký số, mơ tả chi tiết thuật tốn DSA algorithm

SECG: Là tiêu chuẩn biên soạn nhóm doanh

nghiệp dẫn dắt công ty Certicom, gần ánh xạ chuẩn ANSI tiếp cận môi trường Web từ Website http://www.secg.org/

ISO 15946-2: Tiêu chuẩn mô tả ECDSA ECIES (còn gọi ECIES-KEM)

RFC 3278: “Use of Elliptic Curve Cryptography (ECC)

Algo-rithms in Cryptographic Message Syntax (CMS)” khuyến nghị sử dụng thuật toán ECC mã hóa thơng điệp văn

2. Phương trình Weierstraß của đường cong Elliptic

Trong tài liệu này, đa phần số đường cong Elliptic nghiên cứu dạng sau:

y2 =x3+Ax+B, (2.1)

Trong đóAvàB số Các giá trị củax, y, A, B thường giá trị trường đó, ví dụ nhưR(số thực),Q(số hữu tỷ),C(số phức), trường hữu hạnFq,vớiq=pn

(29)

Tạp chí Epsilon, Số 09, 06/2016

được gọi điểmK–Hữu tỷ Dạng tổng quát phương trình Weierstrass đường cong Elliptic biểu diễn dạng:

y2+a1xy+a3y=x3+a2x2+a4x+a6, (2.2)

Trong đóa1,· · · , a6là số Dạng (2.2) thường sử dụng với trườngK có đặc số

chap(K)bằng KhiK có chap(K)khác biến đổi (2.2) thành dạng sau:

y+ a1x +

a3

2

=x3+

a2 +

a21

4

x2+

a4 +

a1a3

x+

a23

4 +a6

,

Có thể viết lại sau:

y12 =x3+a02x2+a04x+a06,

Vớiy1 =y+a1x/2 +a3/2và với sốa02, a

0

4, a

0

6 KhiKcó chap(K)khác dùng

phép thếx1 =x+a02/3và ta có:

y12 =x31+Ax+B,

Trong đóA, Blà số Đường cong (2.1) có định thức∆ =−16(4A3+ 27B).

Đường cong suy biến đủ nghiệm phân biệt khi∆ = 0, tài liệu xét đường cong có∆6=

3. Cộng điểm đường cong Elliptic

Xét hai điểmP1 = (x1, y1)vàP2 = (x2, y2)trên đường cong EllipticE :y2+a1xy+a3y =

x3+a

2x2+a4x+a6 Phép cộng hai điểm đường congE định nghĩa sau:

P3(x3, y3) = P1(x1, y1) +P2(x2, y2) (3.1)

Trong đóP3(x3, y3) =−P30(x3, y30), điểmP

0

3(x3, y03) giao điểm đường congEvà đường

thẳng qua P1 vàP2 Vì điểmP3(x3, y3)và−P30(x3, y30)đều nằm đường congE nên (x3, y3)và(x3, y30)phải thỏa mãn phương trình (2.2) Cơng thức để tính giá trị(x3, y3)sẽ

được chứng minh

Trong các tài liệu nâng cao tham chiếu nhiều đường cong Elliptic [3,7,8] người viết chưa thỏa mãn với dẫn dắt chứng minh công thức tổng quát cho giá trị(x3, y3), cơng thức chứng minh chi tiết tài liệu Đường

thẳng qua điểmP1 vàP2 có phương trình là:

y=λx+µ (3.2)

Trong đóλlà hệ số góc đường thẳng quaP1, P2 Ta có:

y1 =λx1+µ (3.3)

y2 =λx2+µ (3.4)

(30)

Hình 6.1: Phép cộng đường cong Elliptic

3.1 Trường hợp điểm không trùng nhau P1 6= P2

Từ (3.3) (3.4) suy ra:y1−y2 =λ(x1−x2), khiP1 6=P2, nghĩa làx1 6=x2 ta có cơng thức:

λ= y1−y2

x1−x2

(3.6)

µ=y1−λx1 =y1−

y1 −y2

x1 −x2

×x1 =

x1y2−x2y1

x1−x2

(3.7)

Tiếp theo thayyở (3.2) vào phương trình (2.2) ta có:

(λx+µ)2+ (a1x+a3)(λx+µ) =x3+a2x2+a4x+a6 (3.8)

Từ dẫn đến phương trìnhr(x) = 0với:

(31)

Tạp chí Epsilon, Số 09, 06/2016

Biết rằngr(x)có nghiệm phân biệt nên viết:

r(x) = (x−x1)(x−x2)(x−x3)

= (x2−(x1+x2)x+x1x2)(x−x3)

=x3−(x1+x2)x2+x1x2x−x3x2 + (x1+x2)x3x−x1x2x3 =x3−(x1+x2+x3)x2+ (x1x2+x1x3+x2x3)x−x1x2x3

(3.10)

Đồng hệ số x2 của r(x) ở phương trình (3.9) (3.10) ta có: x

1 +x2 + x3 = −(a2−λ2−a1λ)từ tính đượcx3 theo cơng thức sau:

x3 =λ2+a1λ−a2−x1−x2 (3.11)

Đến cần phải tính tiếp giá trịy3, lúc nàyx3 tính xong nên coi số,

viết lại (2.2) thành dạng sau:

y2+ (a1x3+a3)y−(x33 +a2x23 +a4x3 +a6) = (3.12)

Phương trình bậc có nghiệm là:

y3, y03 =

−(a1x3+a3)± √

2×1 (3.13)

Cộng nghiệm ta có:y30 +y3 =−a1x3−a3, mặt khác doy30 nằm đường thẳngP1, P2

nêny30 =λx3+µ Từ tính đượcy3 theo cơng thức5:

y3 =−λx3−µ−a1x3−a3 (3.14)

Thayµtừ (3.7) ta tínhy3dưới dạng sau:

y3 =λ(x1−x3)−y1−a1x3−a3 (3.15)

3.2 Trường hợp điểm trùng nhau P1 = P2

Khi nàyx1 =x2vày1 =y2 cơng thức tínhλở (3.7) khơng sử dụng xuất phép

chia số0 Trong trường hợp nàyλchính hệ số góc đường thẳng tiếp tuyến đường congE

tạiP1hayP2 Hệ số góc tiếp tuyến củaE đạo hàm dydx, sử dụng quy tắc lấy đạo

hàm tích, đạo hàm hàm số hợp lấy đạo hàm vế phương trình (2.2) theodxta có:

d(y2+a1xy+a3y)

dx =

d(x3+a2x2+a4x+a6)

dx d(y2)

dx +

d(a1xy)

dx +

d(a3y)

dx = 3x

2+ 2a

2x+a4

d(y2)

dy × dy dx+a1(

d(xy)

dx ) +a3 dy dx = 3x

2+ 2a

2x+a4 2ydy

dx+a1(y+x dy dx) +a3

dy dx = 3x

2+ 2a

2x+a4 (2y+a1x+a3)

dy dx = 3x

2+ 2a

2x+a4−a1y

dy dx =

3x2+ 2a2x+a4−a1y 2y+a1x+a3

5Ghi chú: Các tác giả H Cohen G Frey “Handbook of Elliptic and Hyperelliptic Curve

Cryptog-raphy” [7] trình bày diễn giải cách tínhy3dựa đối xứng qua trụcxlà khơng xác thiếu rõ ràng

(32)

Như với điểmP1(x1, y1)ta có:

λ= 3x

1+ 2a2x1+a4−a1y1 2y1+a1x1+a3

(3.16)

Trong tất trường hợp điểmP3 tổng điểmP1, P2 điểm có tọa độ là:

P3(x3, y3) = (λ2+a1λ−a2−x1−x2, λ(x1−x3)−y1−a1x3−a3) (3.17)

Với đường congEdạng (2.1), đóa1 =a3 =a2 = 0vàP3 tính theo cơng thức:

P3(x3, y3) = (λ2−x1−x2, λ(x1−x3)−y1) (3.18)

Trong trường hợpP1 =P2, (3.16) biến đổi thành:

λ= 3x 1+a4 2y1

(3.19)

4. Nhân vô hướng điểm đường cong Elliptic

Vớin ∈N\ {0}định nghĩa phép nhân vô hướng điểmP nằm đường congE phép cộngnlần thân điểmP:

P 7→nP =P +P +· · ·+P | {z }

nlần

=Q

Để tối ưu phép nhân vơ hướng, sử dụng phương phápNhân đôi-và-cộng, biểu diễn sốndưới dạng: n = n0 + 2n1 + 22n2 +· · ·+ 2mnm với[n0 nm]∈ {0,1}, sau áp

dụng thuật tốn:

Algorithm 1Phương pháp Nhân đơi-và-cộng

1: Q←0

2: fori= 0tomdo

3: ifni = 1then

4: Q←CộngĐiểm(Q,P)

5: end if

6: P ←NhânĐôi(P)

7: end for

8: return Q

Ngồi phương phápNhân đơi-và-cộng, sử dụng phương phápTrượt-cửa-sổ Các phương pháp cho phép nhân vô hướng cách tối ưu

Lưu ý người viết:

• Khơng tồn phép nhân điểm đường congE, có nghĩa khơng tồn tạiP ×Qvới

P, Q∈E

(33)

Tạp chí Epsilon, Số 09, 06/2016

Hình 6.2: Ví dụ tính chất kết hợp đường cong Elliptic

5. Nhóm (+) của điểm đường cong Elliptic

Xét đường cong EllipticE định nghĩa phương trìnhy2 =x3+Ax+B Xét điểm

(34)

Để điểm đường cong Elliptic tạo thành nhóm(+), “điểm vơ cùng” (∞) thêm vào đường cong, kí hiệu làO, điểm nằm trụcy Một thuộc tính quan trọng đường cong Elliptic tồn nhóm điểm với phép cộng nằm đường cong

Định lý 5.1 Phép cộng với điểmP, P1, P2, P3 trên đường congE thỏa mãn tính chất của nhóm:

1 (Giao hoán):P1+P2 =P2+P1; 2 (Điểm đơn vị):P +∞=P;

3 (Điểm nghịch đảo): Tồn tạiP0 củaP sao choP +P0 =∞; 4 (Kết hợp):(P1+P2) +P3 =P1+ (P2+P3)

Chứng minh. (1 Tính chất giao hốn) phép cộng điểmP1, P2là hiển nhiên từ cơng thức

tính tọa độ điểm tổng, giá trị có giao hốn giá trị tính cơng thức khơng thay đổi, mặt hình học đường thẳng quaP1, P2 dù có xuất phát từP1hayP2thì

như cắt đường congEtại điểm chung

(2 Điểm đơn vị) khơng cần phải chứng minh xuất phát từ định nghĩa Có thể lý giải rõ điểm (∞) cách định nghĩa phép cộng trênE (theo quan điểm người viết) sau: Khi đường congE khơng suy biến, cắt đường thẳng định nghĩa phương trình (3.2) điểm, theo phép biến đổi mục 3.1 (trang 28) phương trình (3.9)r(x)sẽ có nghiệm phân biệt Mặt khácEđối xứng qua trụcxdo phương trình (2.1) có thành phầny2

nên ln tồn hai giá trịy,−ythỏa mãn (2.1), tính đối xứng nên đường congE cắt đường thẳng song song với với trụcyở điểm, cắt thêm điểm phải cắt thành điểm tính đối xứng, điều mẫu thuẫn phương trình bậc có tối đa nghiệm Ở trường hợp cộng điểm nằm đường song song trụcysẽ khơng tìm điểm thứ vậyP1+P2sẽ khơng tồn Chính để nhóm điểm trênE có tính đóng bắt

buộc phải định nghĩa thêm điểm∞coi điểm thứ nằm đường congE, nằm vơ cực đầu trụcy

Tiếp theo, lý giải (của người viết6) phép định nghĩa phép cộng điểm trênE sau Để thỏa mãn tính chất tồn điểm đơn vị theo định nghĩa nhómGvới giá trịa∈Gtồn tạie ∈Gsao cho

a•e=e•a=a (5.1)

Xét điểmP đường congE, cần tínhP +∞, dễ thấy điểm chắn phải nằm đường thẳng song song trụcyvì khơng cắtE điểm với∞tạo thành điểm điều phi lý Nếu nằm đường song songythì cắtEở điểm đối xứng qua trụcx, coi điểm cắt tổngP +∞thì tồn tạiP +∞=P0 điều kiện (5.1) không thỏa mãn Vì phải định nghĩa điểm tổng khơng phải giao

6

Khi bắt đầu nghiên cứu đường cong Elliptic, ln có câu hỏi mà người viết khơng thể tìm thấy nhiều tài liệu kể kinh điển Elliptic:

1 Tại phải chọn∞làm điểm trung hòa

2 Tại saoP1+P2=P3màP3không nằm đường thẳng quaP1, P2mà phải điểm đối xứng giao

điểm qua trụcx

(35)

Tạp chí Epsilon, Số 09, 06/2016

trực tiếp vớiEmà phải lấy điểm đối xứng để đối xứng điểm đối xứng quay lại chínhP

và ta cóP +∞=P

(3.Điểm nghịch đảo) Cũng từ nhận xét tồn điểmP, P0nằm đường thằng song song với trụcyvà cắt đường congE điểm∞, coi điểm nghịch đảo ln có:P +P0 =∞

(4 Tính chất kết hợp) Chứng minh(P1+P2) +P3 =P1+ (P2 +P3)khác hẳn với điều kiện

khác nhóm, đặc biệt phức tạp Có cách chứng minh điều dùng phương pháp hình học đại số Có thể tham khảo chứng minh hình học qua tài liệu [3], [17] [4], nhiên chứng minh phương pháp hình học tương đối khó hiểu với số định lý khơng gian xạ ảnh Dưới số điểm sử dụng để chứng minh phương pháp đại số Trước tiên tính điểm tổng sau:

P12 =P1+P2

P32 =P3+P2

P123 = (P1+P2) +P3 =P12+P3

P321 = (P3+P2) +P1 =P32+P1

Từ công thức tính giá trị tọa độ(x, y)của điểm tổng, dễ dàng biến đổi tính điểm:

λ12=

(y2−y1) (x2−x1)

(5.2)

λ32=

(y2−y3) (x2−x3)

(5.3)

x12=λ212−x1−x2 =

(y2−y1)2 (x2−x1)2

−x1−x2 (5.4)

y12=λ12(x1−x12)−y1 =−λ312+ (2x1+x2)λ12−y1 (5.5) =−(y2−y1)

3 (x2−x1)3

+ (2x1 +x2)(y2−y1)

x2−x1

−y1

x32=λ232−x3−x2 =

(y2−y3)2 (x2−x3)2

−x3−x2 (5.6)

y32=λ32(x3−x32)−y3 =−λ332+ (2x3+x2)λ32−y3 (5.7) =−(y2−y3)

3 (x2−x3)3

+ (2x3 +x2)(y2−y3)

x2−x3

−y3

λ123=

(y12−y3) (x12−x3)

(5.8)

λ321=

(y32−y1) (x32−x1)

(5.9)

(36)

được biểu diễn dạng công thức từ (5.10) đến (5.13)

x123=

−(y2 −y1) (x2 −x1)3

+ (2x1+x2)(y2−y1)

x2−x1

−y1−y3

2

(y2−y1)2 (x2−x1)2

−x1 −x2 −x3

2 −

(y2−y1)2 (x2−x1)2

+x1+x2−x3

(5.10)

y123=

−(y2 −y1) (x2 −x1)3

+ (2x1+x2)(y2−y1)

x2−x1

−y1−y3

3

(y2−y1)2 (x2−x1)2

−x1 −x2 −x3

3 (5.11)

+

2(y2−y1)2 (x2−x1)2

−2x1−2x2+x3 −

(y2 −y1)3 (x2 −x1)3

+ (2x1+x2)(y2−y1)

x2−x1

−y1−y3

(y2−y1)2 (x2−x1)2

−x1 −x2 −x3 + (y2−y1)

3 (x2−x1)3

− (2x1+x2)(y2−y1)

x2−x1

+y1

x321=

−(y2 −y3) (x2 −x3)3

+ (2x3+x2)(y2−y3)

x2−x3

−y3−y1

2

(y2−y3)2 (x2−x3)2

−x3 −x2 −x1

2 −

(y2−y3)2 (x2−x3)2

+x3+x2−x1

(5.12)

y321=

−(y2 −y3) (x2 −x3)3

+ (2x3+x2)(y2−y3)

x2−x3

−y3−y1

3

(y2−y3)2 (x2−x3)2

−x3 −x2 −x1

3 (5.13)

+

2(y2−y3)2 (x2−x3)2

−2x3−2x2+x1 −

(y2 −y3)3 (x2 −x3)3

+ (2x3+x2)(y2−y3)

x2−x3

−y3−y1

(y2−y3)2 (x2−x3)2

−x3 −x2 −x1 + (y2−y3)

3 (x2−x3)3

− (2x3+x2)(y2−y3)

x2−x3

+y3

Cần phải chứng minh rằngP123 =P321điều có nghĩa cần phải chứng minh:

dx=x123−x321 = (5.14)

dy =y123−y321 = (5.15)

Triển khai vế trái (5.14), phân số mà tử số mẫu số bao gồm tổng cộng 1446 thành phần, tương tự vế trái (5.15) có tất 10081 thành phần có dạngnkxi11x

i2 x i3 3y j1 y j2 y j3 ,

(37)

Tạp chí Epsilon, Số 09, 06/2016

thức Có thể kiểm tra tính đắn (5.14) (5.15) phần mềm Maple với ràng buộc sau:

y12 =x31+Ax1+B (5.16)

y22 =x32+Ax2+B (5.17)

y32 =x33+Ax3+B (5.18)

6. Đường cong Elliptic trường hữu hạn Fq

6.1 Trường hữu hạn Fq

Các ứng dụng mật mã đường cong Elliptic đa số sử dụng đường cong trường hữu hạn

XétFqlà trường hữu hạn (hữu hạn số phần tử số nguyên dương): Fq={0,1,2 , q−1}

qlà số nguyên tố có dạngq =pm vớiplà số nguyên tố vàmlà số nguyên

dương Khi nàypđược gọi đặc sốchar(q) = pvàmlà bậc mở rộng củaFq

Trong thực tế đặc biệt thiết bị phần cứng [18], người ta thường sử dụng trường hữu hạnF2m Khi phép cộng trường đơn giản phép toán XOR (Exclusive OR)

Nhiều tài liệu cho thấy làm việc vớiF2m hiệu 40% so với làm việc với trườngFq Nhóm

thực Đề tài cấp Nhà nước KC01.18 người viết làm hủ nhiệm đề tài cài đặt tồn phép tốn đường cong Elliptic trườngF2m cho Chip Spartan Xilinx cho toán

xác thực trao đổi khóa phiên thiết bị VPN IPSec

TrườngF2mthường biểu diễn dạng tổ hợp tuyến tính vector gồmmphần tử

{α0, α1, , αm−1}, phần tửα ∈F2m biển diễn dạng:

α=a0α0+a0α0+ .+am−1αm−1, ∈ {0,1}

Có nhiều phương pháp để xây dựng sở củaF2m: đa thức sở sở chuẩn tắc Các thuật

toán để thực phép tốn EC tìm thấy [19]

6.1.1 Đa thức sở

Xét đa thứcf(x) =xm+f

m−1xm−1+ .+f2x2+f1x+f0 (vớifi ∈F2, i= 0, , m−1)

là đa thức bất khả quy bậcmtrên trườngF2, nghĩa khơng thể phân tíchf(x)thành đa thức thừa số khác có bậc nhỏ hơnm.f(x)gọi đa thức rút gọn Trường hữu hạnF2msẽ tập

tất đa thức trênF2 có bậc nhỏ bằngm

F2m ={am−1xm−1 + .+a2x2+a1x+a0 :ai ∈ {0,1}}

Các phần tử(am−1xm−1+ .+a2x2 +a1x+a0)thường biểu diễn dạng chuỗi bit (am−1 a1a0)có độ dài làm

Các phép tốn trườngF2m:

• Phép cộng:

(38)

• Phép nhân:

(rm−1 r1r0) = (am−1 a1a0).(bm−1 b1b0)

Trong đó(rm−1xm−1+ .+r1x+r0) = (am−1xm−1+ .+a1x+a0)×(bm−1xm−1+

.+b1x+b0) mod f(x)

Đa thức rút gọnf(x)thường có dạng sau:

• Trinomial basis (TPB):

f(x) = xm+xk+ 1, 16k 6m−1. • Pentanomial basis (PPB):

f(x) = xm+xk3 +xk2 +xk1 + 1, 16k

1 < k2 < k3 6m−1

6.1.2 Cơ sở chuẩn tắc

F2m sử dụng sở có dạng{β, β2

2

, , β2m−1

}vớiβ ∈F2m, phần tửa ∈F2mđều có

dạnga=Pm−1

i=0 aiβ2

i

, ∈ {0,1}và biểu diễn dạng chuỗi bit(a0a1 am−1)

có độ dài làm Với sở phép bình phương thực đơn giản cách quay bit Các phép toán trườngF2m:

• Phép cộng:

(c0c1 cm−1) = (a0a1 am−1) + (b0b1 bm−1), ci = (ai+bi) mod • Phép bình phương:

a2 = m−1

X

i=0

aiβ2

i

!2

= m−1

X

i=0

aiβ2

i+1

= m−1

X

i=0

ai−1β2

i

= (am−1a0a1 am−2) • Phép nhân:

Xétp=T m+ 1vàu∈Fp phần tử bậcT, định nghĩa chuỗiF(1), F(2), , F(p−1)

ta có:

F(2iuj mod p) = i, 06i6m−1,06j 6T −1

(c0c1 cm−1) = (a0a1 am−1).(b0b1 bm−1)

cl=

  

 

Pp=2

k=1aF(k+1)+lbF(p−k)+l, nếuT chẵn

Pm/2

k=1(ak+l−1bm/2+k+l−1+am/2+k+l−1bk+l−1) +Pp=2

k=1aF(k+1)+laF(k+1)+l, nếuT lẻ

6.2 Tổng số điểm đường cong Elliptic trường hữu hạn Fq

E đường cong Elliptic trườngFq, cặp(x, y)vớix, y ∈Fqlà hữu hạn nhóm

E(Fq)cũng nhóm hữu hạn Các giá trịx, y số nguyên, dễ dàng nhận thấy với giá trịxđều tìm giá trị ngunybởi khơng phải bao giờ√ycũng số nguyên dương Câu hỏi đặt số điểm của đường cong Elliptic trườngFqlà bao

nhiêu? Xác định số điểm đường congE nhằm xác định khơng gian khóa hệ mật Sau phần trình bày việc tính tổng số điểm đường cong Elliptic trường hữu hạn

(39)

Tạp chí Epsilon, Số 09, 06/2016

Bổ đề M N hai ma trận2×2.M =

m11 m12

m21 m22

, N =

n11 n12

n21 n22

, với số nguyên

a, bta có:

det(aM +bN) =a2det(M) +b2det(N) +ab(det(M +N)−det(M)−det(N)) (6.1)

Chứng minh. Theo định nghĩa định thức ta có:

det(M) = m11m22−m21m12, det(N) = n11n22−n21n12 det(aM +bN) = det

am11+bn11 am12+bn12

am21+bn21 am22+bn22

= (am11+bn11)(am22+bn22)−(am21+bn21)(am12+bn12) =a2(m11m22−m21m12) +b2(n11n22−n21n12)+

+ab(m11n22+n11m22−m21n12−n21m12)

=a2det(M) +b2det(N) +ab(m11n22+n11m22−m21n12−n21m12)

(6.2)

Khia =b = 1áp dụng cơng thức ta có:

det(M +N) = det(M) + det(N) + (m11n22+n11m22−m21n12−n21m12) (6.3)

Nhân vế (6.3) vớiab sau trừ vào (6.2) kết (6.1) điều cần phải chứng minh

Định nghĩa 6.1. Điểm n-xoắn (Torsion): Cho đường cong EllipticEđược định nghĩa trường

K, chonlà số nguyên dương, tập điểm Torsion E[n] tập điểm đường cong có tính chất sau:

E[n] =

P ∈E(K)|nP =∞ (6.4)

K đóng đại số củaK

Định nghĩa 6.2. Divisor: Cho đường cong EllipticE được định nghĩa trườngK, với mỗi điểmP ∈E(K)định nghĩa ký hiệu hình thức (formal symbol)[P], divisor D là:

D=X j

aj[Pj], aj ∈Z

f là hàm trênEmà khác0, divisor củaf sẽ là:

div(f) = X P∈E(K)

ordP(f)[P]∈Div(E)

f =ur

Pg, vớir∈Zg(P)6= 0,∞,u(P) = 0, định nghĩa bậc củaf tạiP là: ordP(f) =r

Định nghĩa 6.3. Cặp Weil: ánh xạ từ điểm nhóm điểm Torsion thành giá trị bậc thứncủa đơn vị:

(40)

Bổ đề Với tự đồng cấu bất khả táchαtrênE, với mọiS, S1, S2, T, T1, T2 ∈E[n]ta có:

en(α(S), α(T)) = en(S, T)deg(α)

en(T, T) =

en(S1+S2, T) = en(S1, T)en(S2, T)

en(S, T1+T2) = en(S, T1)en(S, T2)

Chứng minh xem [3]

Giả thiết{T1, T2}là sở củaE[n], phần tử trongE[n]đều biểu diễn dạng tổ

hợp tuyến tínhm1T1+m2T2.αlà tự đồng cấu trongE[n],nlà số nguyên không chia

hết bởichar(K) Tồn sốa, b, c, d∈Zsao cho:

α(T1) = aT1+cT2, α(T2) = bT1+dT2

Do tự đồng cấuαđều biểu diễn ma trận2×2:

αn=

a b c d

Bổ đề αlà tự đồng cấu trongE[n],nlà số ngun khơng chia hết bởichar(K)khi đódet(αn)≡deg(α) mod n.

Chứng minh. Đặtζ =en(T1, T2), theo bổ đề ta có:

ζdeg(α)=en(α(T1), α(T2)) =en(aT1+cT2, bT1+dT2) =en(T1, T1)aben(T1, T2)aden(T2, T1)cben(T2, T2)cd =ζad−bc =ζdet(αn)

Nếuα, β tự đồng cấu trênE, vàa, blà số nguyên tự đồng cấuaα+bβ định nghĩa sau:

(aα+bβ)(P) =aα(P) +bβ(P)

Bổ đề deg(aα+bβ) =a2degα+b2degβ+ab(deg(α+β)−degα−degβ)

Chứng minh. Biểu diễn tự đồng cấuα, βbằng ma trậnαn, βn(với số sở

E[n]), theo đóaα+bβ biểu diễn bằngaαn+bβn Áp dụng cơng thức (6.1) ta có: det(aαn+bβn) =a2det(αn) +b2det(βn) +ab(det(αn+βn)−det(αn)−det(βn))

Theo bổ đề có:

(41)

Tạp chí Epsilon, Số 09, 06/2016

Định lý 6.4 (Hasse) NếuE là đường cong Elliptic trườngFq, và#E(Fq)là tổng số điểm trên đường cong thì:

q+ 1−2√q6#E(Fq)6q+ + 2√q (6.5)

Chứng minh. Trước tiên xét ánh xạ Probenius định nghĩa sau:

φq :Fq −→Fq,

x7→xq

Có thể viết cách khác:

φq(x, y) = (xq, yq), φq(∞) =∞

Khi thay giá trịxq, yq vào phương trình (2.1) dễ thấy(x, y)cũng nằm đường congE.

Ánh xạφq tự đồng cấu biểu diễn hàm đa thức hữu tỷ có bậc làq Đạo hàm

củaxqlàqxq−1 sẽ bằng0bởi vìq = 0trong trường

Fq Do đạo hàm bằng0nênφq khả tách

(separable)

Bởi vìφq tự đồng cấu trongEdo đóφ2q =φq◦φqcũng tự đồng cấu vàφnq tự đồng

cấu trongE Phép nhân với−1cũng tự đồng cấu tổngφnq −1là đồng cấu trongE

φq khả tách (separable) nhưngφq−1sẽ bất khả tách bậc số phần tử

của hạchφq−1có nghĩa số điểm đường congE là: #E(Fq) =deg(φq−1)

Với số nguyênr, s, áp dụng bổ đề ta có:

deg(rφq−s) =r2deg(φq) +s2deg(−1) +rs(deg(φq−1)−deg(φq)−deg(−1))

Bởi vìdeg(−1) = 1vàdeg(φq) = qnên:

deg(rφq−s) = r2q+s2+rs(deg(φq−1)−q−1))

Đặt a = −(deg(φq − 1)− q − 1) = q + − #E(Fq), deg(rφq −s) > suy

r2q+s2+rsa>0hay với mọir, sta có:

qr s

2

−ar s

+ >0

Do đó∆ = a2−4q60hay là|a|62√qcũng có nghĩa là|q+ 1−#E(

Fq)|62 √

qvà điều phải chứng minh

Tham khảo thêm cách tính số điểm đường congE xem [20]

Tài liệu tham khảo

(42)

[2] J H Silverman and J T Tate, Rational Points on Elliptic Curves - Second Edition Springer, 2015

[3] L C Washington, Elliptic Curves Number Theory and Cryptography, Second Edition CRC Press, 2008

[4] J W S Cassels,Lectures on Elliptic Curves University of Cambridge, 1991

[5] S Lang,Elliptic Curves Diophantine Analysis Springer, 1978

[6] C Kenig, A Ranicki, and M Rockner,Elliptic Curves A Computational Approach Walter de Gruyter GmbH & Co., 2003

[7] H Cohen and G Frey, Handbook of Elliptic and Hyperelliptic Curve Cryptography Chapman Hall/CRC, 2006

[8] J H Silverman,The Arithmetic of Elliptic Curves Springer, 2009

[9] L Berger, G Bockle, L D M Dimitrov, T Dokchitser, and J Voight, Elliptic curves, Hilbert modular forms and Galois deformations Birkhauser, 2013

[10] I F Blake, G Seroussi, and N P Smart, Advances in Elliptic Curve Cryptography Cambridge University Press, 2005

[11] I Connell,Elliptic Curve Handbook McGill University, 1999

[12] T H Otway,Elliptic Hyperbolic Partial Differential Equations Springer, 2015

[13] Dang Minh Tuan, “Che tao thiet bi VPN IPSec bang phan cung dau tien o Vietnam,”Tap chi CNTT & TT, no 2, pp 41–45, 2014

[14] H Lenstra., “Factoring Integers with Elliptic Curves,”The Annals of Matematics, vol 126, no 3, pp 649–673, 1987

[15] V S Miller, “Use of elliptic curves in cryptography,”CRYPTO ’85, pp 417–428, 1985

[16] N Koblitz, “Elliptic curve cryptosystem,”Math.Comp, vol 48, no 16, pp 203–209, 1987

[17] A Enge, Elliptic Curves and Their Applications to Cryptography Kluwer Academic Publishers, 2001

[18] D Hankerson, J L Hernandez, and A Menezes, “Software Implementation of Elliptic Curve Cryptography over Binary Fields,”CHES2000, vol 1965, pp 243–267, 2000

[19] D Hankerson, A Menezes, and S Vanstone, Guide to Elliptic Curve Cryptography Springer-Verlag, 2004

[20] R Schoof, “Elliptic Curves Over Finite Fields and the Computation of Square Roots,”

Matematics of Computation, pp 483–495, 1985

[21] I Blake, G Seroussi, and N Smart,Elliptic Curves in Cryptography Cambridge Univer-sity Press, 1999

(43)

Tạp chí Epsilon, Số 09, 06/2016

[23] J M Pollard, “Monte Carlo Methods for Index Computations (mod p),”Mathematics of Computation, vol 32, no 143, pp 918–924, 1978

[24] S C Pohlig and M E Hellman, “An Improved Algorithm for Computing Logarithms over GF(p) and its Cryptographic Significance,”IEEE Transactions on Information Theory, vol 24, pp 106–110, 1978

[25] A J Menezes, T Okamoto, and S A Vanstone, “Reducing elliptic curve logarithms to logarithms in a finite field,”IEEE Trans Inform Theory, vol 39, no 5, pp 1639–1646, 1993

[26] C Research,Standards For Efficient Cryptography, SEC 1: Elliptic Curve Cryptography Certicom Corp, 2000

[27] L Gao, S Shrivastava, and G E Sobelman, “Elliptic Curve Scalar Multiplier Design Using FPGAs,”CHES’99, vol 1717, pp 257–268, 1999

[28] L Laurie, M Alfred, Q Minghua, S Jerry, and V Scott, “An Efficient Protocol for Authenticated Key Agreement,”Designs Codes and Cryptography, vol 28, no 2, 1998

[29] D Johnson, A Menezes, and S Vanstone, “The Elliptic Curve Digital Signature Algorithm (ECDSA),” 2001

[30] T E Gamal, “A Public Key Cryptosystem and a Signature Scheme Based on Discrete Logarithms,”CRYPTO ’84, vol 196, pp 10–18, 1985

[31] NIST,Digital Signature Standard (DSS) FIPS 186-4 National Institute of Standards and Technology, 2013

[32] J Massey and J Omura, “Method and apparatus for maintaining the privacy of digital messages conveyed by public transmission,” Jan 28 1986, US Patent 4,567,600 [Online]

(44)(45)

NGHỊCH LÝ TRAI GÁI

Đặng Nguyễn Đức Tiến (Đại học Trento, Italia)

LỜI GIỚI THIỆU

Tiếp nối chun mục Tốn học Giải trí số Epsilon trước, số giới thiệu với độc giả nghịch lý tiếng xác suất: Nghịch lý trai gái

Bài toán này, lại lần nhiều toán đố khác chun mục Tốn học Giải trí, xuất phát từ đại thụ Martin Gardner1với tên gọi toán hai người (Two Children Problem) đăng tờ Scientific American vào năm 1954:

"Một người có hai con, biết có trai, hỏi khả hai là con trai bao nhiêu?"

1 Nghịch lý trai gái toán ngày thứ ba

Chúng ta bắt đầu với việc phân tích thử lời giải cho tốn hai người Martin Để đơn giản, thống với giả thiết giới tính người độc lập (nghĩa giới tính người em khơng phụ thuộc vào giới tính anh chị người đó) khả người nam hay nữ

Trực quan dễ thấy, người trai, nên người lại trai gái với khả (theo giả thiết) Khả trai 1/2

Vậy đâu có mâu thuẫn hay nghịch lý! Nhưng dừng lại chút, thử phân tích khả có người chưa biết giới tính họ Có thể có trường hợp theo thứ tự người thứ người thứ là: nam - nam, nam - nữ, nữ - nam nữ - nữ, khả xảy Và ta biết người nam, nên trường hợp cuối nữ - nữ bị loại ra, cịn có trường hợp đầu số có trường hợp nam - nam ứng với yêu cầu câu hỏi Vì vậy, khả trai 1/3 Nghịch lý lý luận khả 1/2!

Nghịch lý thu hút nhiều người, đặc biệt có thời điểm người ta chia làm hai “trường phái" đối lập hẳn trường phái “nhị phân" trường phái "tam phân", ứng với lý luận giải thích Và sau 60 năm từ đời nay, mâu thuẫn hai trường phái, hay nhìn nhận mâu thuẫn trực quan lý luận tốn cịn xảy với nhiều người câu trả lời đâu đáp án thật đơn giản, nhìn, có lý

Trước vào phân tích đâu trường phái "chiến thắng" nghịch lý trai gái, giới thiệu tiếp với độc giả phiên lạ toán này, đề xuất Gary Foshee vào năm 2010 Bài toán sau:

1

(46)

"Một gia đình có hai con, biết có đứa trai sinh vào ngày thứ ba khả để hai trai bao nhiêu?"

Hẳn nhiều bạn đọc hỏi: "Thêm vào ngày thứ ba liên quan gì?" Và câu trả lời hẳn làm ngạc nhiên cho bạn xác suất để hai trai 1/2 hay 1/3 mà 13/27

2 Đi tìm lời giải

Trước tiên, hệ thống lại hai tốn sau:

Một gia đình có hai người còn, hỏi khả trai bao nhiêu, biết rằng: Xác suất nam nữ 1/2

2 Giới tính hai người khơng phụ thuộc vào Cho biết trước điều kiệnK

Và tổng hợp lại trường hợp khác vào bảng sau:

Trường hợp Điều kiệnK

TH1 Khơng có điều kiện

TH2 Người đầu trai TH3 Ít hai trai

TH4 Ít hai trai sinh vào ngày thứ ba

Bây giờ, xét trường hợp đơn giản trước TH1, không gian mẫu

Ω ={T T, T G, GT, GG}vớiT trai vàGlà gái Và vậy, dễ thấy xác suấtT T 1/4 Với TH2,Ω ={T T, T G}ứng với người đầu trai, vậy, xác suấtT T xảy 1/2 Với TH3,Ω ={T T, T G, GT}, nênT T xảy với xác suất 1/3, tốn đầu đề Với TH4, lời giải khó thơng tin ngày thứ dường khơng liên quan

Trước tiên, giả sử người thuộc hai nhóm đó, mà tần suất nhóm

P làpvà tần suất nhómQlàq, vớip+q =nvớinlà tổng khả có Ví dụ TH4, hai nhóm làP = "sinh vào ngày thứ ba" vàQ= "khơng sinh vào ngày thứ ba" cóp= 1và

q= Như vậy, khơng gian mẫuΩcó 16 trường hợp ứng với tần suất xảy sau:

Hàng/Cột C1 C2 C3 C4

H1 TPTP TPTQ TPGP TPGQ [p2] [pq] [p2] [pq]

H2 TQTP TQTQ TQGP TQGQ [pq] [p2] [pq] [q2]

H3 GPTP GPTQ GPGP GPGQ [p2] [pq] [p2] [pq]

H4 GQTP GQTQ GQGP GQGQ [pq] [q2] [pq] [q2]

Qua bảng trên, ta thấy tổng tất khả xảy làn4 và ứng với "khối"2×2, số khả

(47)

Tạp chí Epsilon, Số 09, 06/2016

khối2×2ở trái (là khối toànT) chia cho toàn khả bảng, sau loại bỏ khối2×2ở góc phải (tức khối toànG) nên xác suất xảy 3nn2 =

1

Quay lại toán ngày thứ ba, với ràng buộc khả năngP = "sinh vào ngày thứ ba" phải xảy ra, không gian mẫuΩcủa tốn tồn hàng H1 cột C1 bảng, nghĩa tồn có chứaTp Và u cầu tốn (H1, C1), (H1, C2) (H2, C1) Xác

suất (chúng tạm ký hiệu làρ), là:

ρ= p

2+pq+pq

p2+pq+p2+pq+pq+p2+pq =

p2+ 2pq

3p2+ 4pq =

p+ 2q

3p+ 4q =

2n−p

4n−p =

2− np 4− np

Khip/ntiến dần tới0,ρsẽ bằng1/2và khip/ntiến dần đến1,ρsẽ 1/3 Với tốn ngày thứ ba (TH4),p= 1vàn= 7, ta có xác suất bằngρ= 42−−11//77 = 13/27 Và vậy, yếu tố "ngày thứ ba", thật có ảnh hưởng đến kết cuối cùng!

Chúng ta thử xét điều kiện sau: "có người trai sinh vào ngày 13 tháng rơi vào thứ ba, xác suất để hai trai bao nhiêu?" Bỏ qua năm nhuận, cóρ= 42−−11//(7(7××365)365) ≈0.49995

3 Cuối có phải nghịch lý?

Như vậy, giải có kết câu hỏi: "Một gia đình có hai người con, biết có người trai, hỏi khả người trai bao nhiêu" với đáp số 1/3 Tuy nhiên, tốn vừa đời, có nhiều tranh cãi, thử khảo sát phiên khác toán với câu hỏi "xác suất để hai trai bao nhiêu":

"Trong tất gia đình có hai con, mà người trai, gia đình chọn ngẫu nhiên để hỏi." Đáp án câu hỏi 1/3, câu hỏi gốc Martin

"Trong tất gia đình có hai con, người số chọn ngẫu nhiên, người trai." Đáp án lúc lại 1/2

Nói cách khác, ngồi vấn đề kết khác với trực quan cách phát biểu toán dễ dẫn đến bối rối gây tranh cãi

Quay lại toán ngày thứ ba, với u cầu "có người trai sinh vào thứ ba" khả trai 13/27 Tuy nhiên, câu hỏi gốc Foshee lại là: "Trong toàn gia đình có hai mà có người trai sinh vào ngày thứ ba, cho biết tỉ lệ gia đình mà hai người trai." Và đáp án câu hỏi này, theo tác giả, 1/2

4 Lời kết

(48)

khó tiếp cận hơn, chất toán học giải trí Để có viết này, chúng tơi tổng hợp sử dụng nguồn tham khảo sau:

1 Wikipedia,Boy or Girl Paradox

2 Gardner, Martin, 1954: The Second Scientific American Book of Mathematical Puzzles and Diversions Simon & Schuster

3 Peter Lynch, The Two-Child Paradox: Dichotomy and Ambiguity, Irish Math Soc Bulletin 67 (2011), pp 67-73

(49)

PHÉP BIẾN ĐỔI FOURIER CĨ Ý NGHĨA VẬT LÝ GÌ?

Job Bouwman (Trần Nam Dũng dịch)

Hãy tưởng tượng bạn gõ phím đàn dương cầm, dây đàn rung lên tạo sóng âm có dạng hình sin ta biết lượng giác Bây tưởng tượng ba phím nhấn lúc, sóng âm khơng cịn hình sin nữa, mà hỗn độn phức tạp Nhưng ẩn sóng âm lộn xộn mơ hình đơn giản: Sau tất cả, hợp âm tạo ba phím đánh lúc, sóng âm lộn xộn kết tổng hợp ba nốt (hay ba sóng sin) Và ví dụ biến đổi Fourier

Trong số này, Epsilon tiếp tục giới thiệu với độc giả phần bình luận từ viết tác giả tác giả Job Bouwman đăng Epsilon số 7:Phép biến đổi Fourier có ý nghĩa vật lý gì?

1 Stephen Scholnik

Thực tơi sử dụng phép biến đổi Fourier nhiều

Đó cơng cụ tốn học để phân tích tín hiệu, khơng có tương tự xác mặt vật lý (IMHO) Nhưng phải nói rằng: Sự tương tự tốt mà tơi nghĩ cho ta biết giá trị tần số trung bình mẫu thử của, chẳng hạn như, ghi âm Như ghi âm nhạc, nói chung cho biết lượng âm trung bình (và pha) nốt nhạc toàn Nhưng điều khơng hẳn nốt nhạc có hịa âm phép biến đổi Fourier khơng thể phân biệt hịa âm nốt La nốt La quảng tám cao

Ngồi ra, tơi nói, phép biến đổi Fourier cho bạn biết pha trung bình tần số mà điều thơng thường lại khơng có minh họa vật lý (mặc dầu bạn nghe biến đổi pha động) Tệ là, nốt nhạc gõ mạnh (tấn công sắc bén rơi xuống), phổ tần số mờ Như có tần số phép biến đổi có lượng đáng kể lại không tương ứng với nốt nhạc Do tất lý đó, tương tự sai lầm (thêm vào sau đó)

(50)

2 Konstantinos Konstantinides, Kỹ sư điện làm việc trên lãnh vực xử lý tín hiệu mã hóa video

Một phép biến đổi Fourier (FT) khơng có nghĩa Đó hàm số Đầu phép biến đổi Fourier cho ta biễn tín hiệu miền tần số Có làm việc với miền thời gian tiện hơn, có làm việc với miền tần số tiện Chẳng hạn như, có bảo với bạn “lọc thơng thấp”, bạn hiểu lọc lọc bỏ tất tần số cao sau phép lọc tần, biểu diễn miền thời gian (đáp ứng xung) phức tạp

Trong xử lý tín hiệu, tích chập miền thời gian tương ứng với phép nhân miền tần số Điều giúp ích nhiều bạn phân tích đáp ứng tần số hệ thống Vậy phép biến đổi Fourier xem cơng cụ chuyển đổi qua lại miền thời gian miền tần số

3 Samuel O Ronsin, PhD, Toán ứng dụng

Thêm ý “ý nghĩa vật lý”, thật thú vị phép biến đổi Fourier (cụ thể dạng trực giao) “căn bậc hai” toán tử “đảo chiều”Φ :f 7→Φ(f) = (x7→f(−x))trong khơng gianL2 đó, vậyF ◦F = Φ

Điều có ý nghĩa gì? Vâng, bạn muốn phân tích tốn tử “đảo chiều” hàm thành hai tốn tử giống nhau, phép biến đổi Fourier bạn muốn tìm Dầu cịn có thêm vài ví dụ

4 Norman Corbett, PhD, Toán ứng dụng

Đối với câu trả lời có: Phép biến đổi Fourier xây dựng nhờ tần số điều hòa bội nguyên vài tần số Một điểm tách rời với âm nhạc chỗ nốt nhạc tuân theo thang hàm mũ (hãy nghĩ đàn piano bạn) Điều khơng có nghĩa phổ q trình vật lý bị ràng buộc theo kiểu

Bên cạnh đó, tính chất tốn học phép biến đổi Fourier đảm bảo xử lý liệu âm thanh, hình ảnh, vết đen mặt trời, ẩn tinh, làm lộ đặc điểm vật lý

Phải phép biến đổi Fourier có nguyên nhân phát sinh cụ thể? Vâng! Joseph Fourier khởi đầu tất nghiên cứu truyền nhiệt khí Ở phân tích tín hiệu tần số điều hòa thõa đáng (vâng cho mơ hình tuyến tính)

(51)

Tạp chí Epsilon, Số 09, 06/2016

5 Singer Karthik, Kỹ sư điện xử lý tín hiệu số điện tử số

Phép biến đổi Fourier (FT) mơ hình/trừu tượng hóa tốn học phân tích tín hiệu thành tín hiệu giống lăng kính nhận vào ánh sáng trắn tách thành tần số ánh sáng khác Khơng có tương tự tốt Trước hết, tín hiệu có thành phần tần số khơng phải đặc điểm miền thời gian Đa số khơng phải kỹ sư gặp khó khăn với khái niệm tần số Hiểu khái niệm trước phép biến đổi Fourier trở nên dễ hiểu

Nói tóm lại, phép biến đổi Fourier cho phép bạn phân tích tín hiệu miền xác định khác giúp ta hiểu thấu thành phần tần số tạo thành tín hiệu mà ta nghiên cứu Nếu bạn thật muốn phân tích nội dung tín hiệu bạn phải xem xét miền tần số Đồ thị biên độ tần số cho ta biết tần số chủ đạo Từ bạn xác định chẳng hạn tần số xác định chứa nhiễu thiết kế lọc để lọc nhiễu Hoặc giả bạn muốn làm suy yếu tần số xác định khuếch đại tần số cịn lại Nó giúp bạn tạo hình tín hiệu mong muốn

6 Jonathan Roberts

Phép biến đổi Fourier phép biến đổi từ miền thời gian vào miền tần số Nếu bạn áp dụng phép biến đổi Fourier cho sóng cosine bạn có đoạn thẳng đứng đồ thị Mọi sóng phức tuần hồn liên tục tạo thành từ tổng chuỗi hình sin trực giao Biến đổi sóng gián đoạn phát sinh lỗi gọi tượng Gibbs Vài hình ảnh hữu ích

(52)

7 Henk Mulder

Phép biến đổi Fourier đo tương quan tín hiệu tuần hồn chuỗi hàm điều hịa hình sin Đây cách mà tốn học làm theo nghĩa đen: tính tương quan hàm điều hịa tín hiệu nhập vào

Điều cụ thể có nghĩa phép biến đổi Fourier đo xem tín hiệu tuần hồn giống hàm điều hịa tới mức Đối vói hàm điều hịa chủ đạo, bạn nhận Nhưng có nhiều hàm điều hịa điều khó khăn Đó lý bạn dùng phép biến đổi Fourier để có tranh xác diện tất hàm điều hịa tín hiệu nhận vào

Cần nói thêm thật thú vị người chị em phép biến đổi Fourier, phép biến đổi Laplace thực xác cách làm với hàm mũ Nó tính tương quan hàm mũ thành phần hàm đầu vào biểu diễn hàm dạng hàm mũ Vì hàm mũ có đạo hàm nó, điều cho phép bạn xử lý dễ dàng nhiều phương trình vi phân

8 Michael Kownacki

Làm việc xí nghiệp làm giàu uranium, thường xuyên dùng phép biến đổi Fourier để lấy giá trị định lượng việc xử lý cố máy móc đồ sộ sử dụng đầu gia tốc kế Mỗi vấn đề tần số đặc biệt, shaft alignment, rung chong chóng turbine, Bằng cách lấy tích phân bình phương phép biến đổi Fourier vùng tần số đặc biệt, tơi tính lượng vấn đề cụ thể

Tôi xác định chuyển động rối làm giảm hiệu trình Tất thứ bạn cần làm lắc nhẹ van đầu vào để thay đổi phân bố áp suất làm cho chuyển động rối biến Một rung vùng rối biến mất, bạn biết van bố trí Một vấn đề nghiêm trọng xảy với turbine áp suất không chung quanh chu vi (thường gọi đột biến thứ cấp) Nó làm cho chong chóng turbine chết máy chúng quay, tạo thành rung chong chóng làm gãy chong chóng Đối với máy lớn có tốc độ cao, điều khiến cho tồn máy nổ tung theo nghĩa đen

9 Mil Ford

Tơi nghĩ ví dụ đề tài tơi, phần xác định độ cao thủy triều bổ ích cho bạn

Thủy triều xuất lực hấp dẫn mặt trời, mặt trăng,

Vì mặt trăng quay chung quanh đất đất quay chung quanh mặt trời, chiều cao thủy triều có dạng hàmsin/cos

Tơi dùng phương pháp gọi Phân Tích Fourier Rời Rạc tạo thành trước Phân Tích Fourier dựa điểm liệu thay hàm

(53)

Tạp chí Epsilon, Số 09, 06/2016

Kết là, chẳng hạn tần số nửa ngày, biết hiệu ứng thủy triều quay trái đất, biết tần số dễ dàng tìm biên độCn

Từ bạn viết thành hàm dài chẳng hạn nhưC1·sin(· · ·) +C2·sin(· · ·)Vẽ hàm

này giúp ta dự đoán thủy triều tương lai

Tiến hành phép biến đổi bạn làm chuyển từ cõi thời gian sang không gian tần số

10 David Tung, nhà doanh nghiệp

Phép biến đổi Fourier kết hợp hai khái niệm vật lý: Tuần hồn sở (đó tín hiệu tốt phân tích thành thành phần độc lập thành phần kết hợp lại tạo thành nguyên tín hiệu ban đầu)

Nhưng phép biến đổi Fourier tạo khái niệm tần số, phần hiểu biết giới vật lý, nói “tìm tần số tương ứng với tin hiệu ý nghĩa vật lý phép biến đổi Fourier”, quên vấn đề con-gà-và-quả-trứng

11 Rod Schmidt

Trong tai có ốc tai cuộn xoắn lại Tưởng tượng khơng xoắn trải dài Nó hình nón dài hẹp Khi âm vào tai, tần số thấp (bước sóng dài nhất) tạo thành cộng hưởng rung gần đầu lớn mở hình nón Những tần số cao (bước sóng ngắn nhất) gây cộng hưởng rung tương tự gần đỉnh hình nón Như tần số phân loại Mỗi tần số tương ứng với khoảng cách dọc theo hình nón

Đối với phép biến đổi Fourier ngược hình dung điều đó, có điều phim chiếu ngược lại

12 Lionel Chiron

(54)(55)

TẬP HỢP TRÙ MẬT VÀ ỨNG DỤNG

Kiều Đình Minh, Nguyễn Quang Khải (THPT chuyên Hùng Vương, Phú Thọ)

Trong viết này, tập hợp hiểu tập củaR

1 Tập hợp trù mật

Định nghĩa Cho hai tập hợpA, X vớiA ⊂X.Ta nói rằngAtrù mật trongXnếu với mọi

x ∈ X và mọi ε > tồn tại x ∈ A sao cho |x−x| < ε Một cách tương đương, với mọi

x, y ∈X, x < ytồn tạix∈Asao chox < x < y

Nói cách nơm na rằng, tậpAtrù mật trongX phần tử trongXđều bị kiểm soát phần tử trongAvới khoảng cách bé tuỳ ý

Định lý 1.1 Tập hợpAtrù mật tập hợpXkhi với mọix∈X, tồn dãy(xn) thoả mãnxn∈A,∀n∈N lim

n→+∞xn=x A=X

Chứng minh Thật vậy, nếux∈Athì chọnxn =x,∀n Nếux /∈A, theo định nghĩa ta có: Với

mọix∈Xvà mọiε >0, tồn tạix∈Asao cho|x−x|< ε Mặt khác dễ thấy tồn dãy(εn)

thoả mãn0< εn < ε,∀n ∈Nsao cho mỗin ∈N, tồn tạixn ∈Athoả mãn|xn−x|< εn

Suy

|xn−x|=|xn−x+x−x|6|xn−x|+|x−x|< εn+ε <2ε,

do lim

n→+∞xn=x Ngược lại hiển nhiên

Nhận xét Nếu Atrù mật trongX thì với mọi x ∈ X, tồn hai dãy(xn1),(xn2) ∈ Athoả

mãn:

(i) xn1 < x < xn2,∀n1, n2 ∈N

(ii) lim

n1→+∞xn1 =n2→lim+∞xn2 =x

Định lý 1.2 Tập số hữu tỷQlà trù mật tập số thựcR.

Chứng minh Ta có nguyên lý Archimet “vớix, y ∈Rsao chox >0, y > 0thì ln tồn tại

n ∈ Nsao choy 6 nx” Xétx, y ∈ R, x < y Đặtd =y−x > Theo nguyên lý Archimet tồn tạin ∈ N∗ sao chond >1, suy ra

n < d Gọim = [nx] + 1thìm−16nx < msuy

nx < m6nx+ 1,suy tiếpx < mn 6x+n1 < x+d=y

Chứng minh Ta cần ra∀a, b ∈ R,∃m

n (m∈Z, n∈Z

+)sao choa < m

n < b Thật vậy,

chọnn ∈Z+sao cho đoạn[na, nb]có độ dài lớn hơn1haynb−na >1tứcn >

b−a Khi

(56)

Chứng minh Khơng tổng quát, ta cố địnhbvà đặtb−a=ε.Xét tậpA={x∈Q\x < b} Theo định nghĩa Supremum:∀ε >0,∃x0sao chob−ε < x0 < b Như ta cần chứng

minh SupA=b Gọi SupA=s Xétn∈N∗ ta chứng minh

s−

n 6b 6s+

1

n

Bất đẳng thức bên trái hiển nhiên vì∃x∗ chos− n < x

∗ < b Giả sửs+

n < b Nếu

s∈Qthìs+

n ∈Avàs+

n > s(vơ lý) Nếus /∈Qthì

w= [(n+ 1)s]

n+ +

n+ ∈Q,

và thoả mãns <w < s+n1 Do w < bhay w ∈ A(vô lý) Suy ra|b−s| 6

n suy tiếp lim|b−s|= 0tứcb=s

Chứng minh Kí hiệu[a]là phần ngun củaa Ta có

[a]6a <[a] + 1⇒ [a]

a 61<

[a] +

a (a >0)

Theo định lý giới hạn kẹp ta lim a→+∞

[a]

a = = lima→−∞ [a]+1

a Choxlà số thực dương, với

n ∈N∗, đặtx

n = [10

nx]

10n vàx

0

n = [10

nx]+1

10n Hai dãy(xn)và(x0n)là giá trị hữu tỉ gần

dưới gần củaxvới độ xác10−n.Dễ thấyxn, x0n∈Qvà(xn)tăng cịn(x0n)

giảm Ngồi lim

n→+∞xn=xvàn→lim+∞x

n=x Nếux <0thì hai dãy số lập tiến tới −x Bởi hai dãy số(−xn)và(−x0n)sẽ tiến tớix

Chứng minh Xét số thựcxnào Ta chứng minh rằng∀ε > 0,∃r ∈Q :|r−x| < ε Thật vậy, nếux∈Qthì ta cần chọnr=x Ngược lại,x /∈Q, xét biểu diễn thập phân vơ hạn khơng tuần hồn củax, x=a0a1a2· · ·an· · ·a∞ Ta thấy∀ε >0,∃n ∈N:ε >10−n Khi đó,

chọnr=a0a1a2· · ·an+1 ∈Qta thấy ngay|r−x|<10−n < ε

Chú ý Tập số vô tỉ trù mật tập số thựcR Thật vậy, với mọi a, b ∈ Q, a < b Đặt

e=b−a >0, ln tìm số tự nhiênnđủ lớn chone > Khi đón(b−a)>2, suy

nb >2 +na Chọnq=√2 +nalà số vô tỉ, do√2là số vô tỉ vànalà số hữu tỉ Ta có

na < √2 +na <2 +na < nb⇒na < q < nb⇒a < q n < b

Vậy tồn tạim= qn là số vô tỉ thoả mãn.

Định lý 1.3 (Kronecker) Cho θ là số vơ tỉ Kí hiệu {x} = x −[x] Khi tập hợp {{nθ} |n∈Z+}là trù mật khoảng(0; 1).

Chứng minh Xét khoảng∆ = (a;b)⊂(0; 1), ta chứng minh∃n∈Z+ :{nθ} ∈(a;b).

Thật vậy, ta chia(0; 1)thành số hữu hạn nửa khoảng∆1,∆2, ,∆n0 cho độ dài

của nửa khoảng nhỏ số thực dươngεmà ta lựa chọn sau Khi đó, ta tìm vơ hạn giá trịn, msao chonθ+m∈[0; 1]mà lại có hữu hạn khoảng nên theo nguyên lý Dirichlet tồn số nguyên dươngn1, n2(n1 > n2)vàm1, m2sao cho

(57)

Tạp chí Epsilon, Số 09, 06/2016

Bây ta chọnεthoả mãnε < b−2a Ta có

b−a

(n1−n2)θ+ (m1−m2)

> b−a ε >2

Do tồn số nguyênknằm (n a

1−n2)θ+(m1−m2)

b

(n1−n2)θ+(m1−m2),tức a < k(n1−n2)θ+ (m1−m2)< b

Bây chọnN =k(n1−n2)thì số{N θ} ∈(a;b).Vậy ta kết luận{{nθ} |n∈Z+}trù mật

trong(0; 1) Định lý chứng minh xong

Định lý 1.4 (Thác triển hàm liên tục) Cho hai hàm số f, g : X → X, đó f liên tục,

g liên tục đơn điệu vàA là tập trù mật trongX Khi nếu f(x) = g(x),∀x ∈ Athì

f(x) = g(x),∀x∈X.

Chứng minh Ta xét hai trường hợp:

a) g hàm liên tục: Theo định lý ∀x ∈ X,∃(xn)n>0 ∈ A : lim

n→+∞xn = x Ta có f(xn) =g(xn),∀n ∈Nsuy lim

n→+∞f(xn) = limn→+∞g(xn)⇒f(x) = g(x)

b) glà hàm đơn điệu: Không tổng quát, giả sửglà đơn điệu tăng.∀x∈X,∃xn1, xn2 ∈A

sao choxn1 < x < xn2 lim

n2→+∞

xn1 = lim

n2→+∞

xn2 =x Ta có f(xn1) =g(xn1)< g(x)< g(xn2) = f(xn2)

Chuyển qua giới hạn ta đượcf(x) =g(x) Trong trường hợp định lý chứng minh

Nhận xét Từ định lý định lý ta có nghiệm phương trình hàm trênRkhi biết hàm đó trênQvà tính liên tục đơn điệu trênR Tiêu biểu lớp phương trình hàm Cauchy.

2 Một số ví dụ minh họa

Ví dụ Cho dãy số thực dương (an), tăng không bị chặn Chứng minh tập hợp

S1 =

n

m

an|m, n∈N

∗o

trù mật trongR+

Chứng minh. Nhận xét với sốα >0và dãy(an)có tính chất lim

n→+∞an= +∞thì

có lim

n→+∞αan = +∞ Xétp, q ∈R

+ thoả mãnp > q Khi ấy lim

n→+∞(p−q)an = +∞ Do đó,

tồn tạin0 ∈N∗ cho

(p−q)an0 >2⇒pan0 >[pan0 −1]> qan0

Chọnm0 = [pan0]−1, ta cóp >

m0

an0 > q VậyS1 trù mật trongR

+. Nhận xét Từ ví dụ suy tập2mn |m, n∈N

∗ , an

m|m, n∈N

trù mật trong

(58)

Ví dụ Giả sửN∗ được chia thành hai tập conAB, tập chứa vô hạn phần tử Đặt

S3 =

a

b |a∈A, b∈B Chứng minh tậpS3trù mật trong[0; +∞)

Chứng minh. DoB có vơ hạn phần tử nên∀k ∈N,∀ε >0,∃b∈B chob > k

2ε.DoAcó vơ

hạn phần tử nên

b(x+ε)> b(x−ε) +k,∀x∈[0; +∞)⇒ ∃a∈A:b(x+ε)> a > b(x−ε),

suy

x+ε > a

b > x−ε⇒ x− a b < ε

VậyS3 trù mật trong[0; +∞)

Nhận xét NếuAtrù mật trongR+ thìB =1r|r∈A cũng trù mật trongR +. Ví dụ Cho(an)n>1là dãy số nguyên dương thoả mãn0< an+1−an<

an,∀n ∈N∗.Đặt

S4 =

n

ak

al |k, l∈N

∗, k > lo.Chứng minh tậpS

4 là trù mật trong(0; 1) Chứng minh. Ta chứng minh nhận định sau:

∀ 0< x < y <1,∃k, l∈N∗, k > l :x < ak al

< y

Thật vậy, từ giả thiết suy ra(an)là dãy số nguyên dương tăng nên lim

n→+∞an= +∞ Suy

∀x, y >0,∃l ∈N∗sao cho a1

al < xvà

1

al < y−x Xét phần tử dãy số hữu hạn

a1

al

< a2 al

<· · ·< al−1 al

< al al

=

Khi với∀k = 1,2, , l−1, ta có

ak+1

al

− ak

al < √ ak al < √ al al

= √1

al

< y−x

Hơn a1

al < xnên phải tồn tạiksao chox <

ak

al < y

Ví dụ Cho dãy số dương(an)tăng thoả mãn lim n→+∞

an

n = k, k ∈ R

+ và dãy số dương(b n) tăng, không bị chặn Chứng minh tậpS5 =

n

am

bn |m, n∈N

∗o

trù mật trongR+. Chứng minh. Xétp, q ∈ R+, p > q Do lim

n→+∞

an

n =knên chọn ε > 0sao cho p−q

p+q ·k > ε Ta

có, tồn tạiN cho

an

n −k <

p−q

p+q ·k−ε,∀n>N ⇒

2q

p+q ·k+ε < an

n <

2p

p+q ·k−ε,∀n >N

Đặtα= 2qq p+qk+ε

< 2qp p+qk−ε

=β Theo ví dụ 1, tồn vơ số bộ(m, n)sao choα < bn

m < β Chú

ý theo thí dụ có tồn hữu hạn, tồn dẫn đến tồn vơ hạn ta chọn

α < α1 <· · ·< αn < βthì giữa(αi, αj)tồn số, chon→+∞thì tồn vơ số số Do

đó, chọnn0 > N vàm0sao cho

α < n0 bm0

(59)

Tạp chí Epsilon, Số 09, 06/2016

2q

p+qk+ε < an0

n0

< 2p

p+qk−ε (2.2)

Nhân vế theo vế (2.1) (2.2) ta cóq < an0

bm0 < q VậyS5trù mật trongR

+. Nhận xét Ta thấy rằng:

1) Tậpm!sinn1

|m, n∈N∗ trù mật trongR+

2) Nếu lim n→+∞

an

n = +∞thì tốn khơng cịn Thật vậy, chọnan = n, b

m = 2m thìS5 khơng trù mật trongR+

3) Nếu lim n→+∞

an

n = thì tốn khơng cịn Thật vậy, chọnan =

k nếu22k

6

n622k+1−1b

m = 2mthìS5 khơng trù mật trongR+

Ví dụ Chứng minh tậpS6 ={sinn|n ∈N}trù mật trong[−1; 1]

Chứng minh. Trước hết ta chứng minh nếuαlà số vơ tỉ dương, tập{n−mα|m, n∈N}

là trù mật trongR+(có thể xem ví dụ 6) Để làm điều đó, lấy khoảng(a;b),0< a < bta khoảng chứa phần tử tập cho Đặtε=b−a >0, tồn phần tửRnsao cho

0< Rn−α <

q2 n

(2.3)

Thực vậy, lấy sốnlẻ ý rằng(qnxn+1+qn−1)qn > qn2 Vì lim

n→+∞qn = +∞, vớinđủ

lớn ta có q1

n < ε Từ điều (2.3) suy ra0< pn−αqn<

1

qn < εvớinđủ lớn, tồn

n0 ∈Nsao chon0(pn−αqn)∈(a;b)

Chot∈[−1; 1], tồn số thựcxsao chot= sinx Từ nhận xét suy tồn dãy số nguyên dương{mn} và{kn}vớix = lim

n→+∞ (mn−2πkn), Sử dụng tính liên tục hàm

sinxta

t= sinx= sin

lim

n→+∞ (mn−2πkn)

= lim

n→+∞sinmn

Vậy số thuộc khoảng[−1; 1]đều điểm giới hạn tậpS6 ={sinn|n ∈N} Ví dụ Chou, v ∈R+.Chứng minh tập hợp{au+bv|a, b∈

Z}trù mật trongRkhi và chỉ khi uv là số vô tỷ.

Chứng minh. Ta có{au+bv|a, b∈Z}trù mật Rnên v au

v +b

|a, b∈Z trù mật trongRdo auv +b|a, b∈Z trù mật trongRsuy raauv |a∈Z trù mật trong(0; 1)

vì uv số vơ tỷ

Nhận xét Vớiu6= 0, v 6= 0thì tốn đúng.

Ví dụ Cho số thực dươnga, bthoả mãnb[an] = a[bn], ∀n ∈ Z+.Chứng minh rằng

(60)

Chứng minh. Giả sửa6=bvà cảa, bđều vơ tỷ, ta có

b[an] =a[bn]⇔ban−b{an}=abn−a{bn} ⇔b{an}=a{bn} (2.4)

Khơng tổng qt, giả sửa > b, ta có

b a =

{bn}

{an},∀n∈Z + ⇒ b

a >{bn},∀n∈Z

+,

điều vô lý doblà số vô tỷ nên{{bn} |n∈Z+}là trù mật trong(0; 1) Do hai

sốa, blà hữu tỷ Giả sửa ∈Q, tồn tạin0 ∈Z+sao choan0 ∈Z+ ⇒ {an0}= Do {bn0}= 0⇒b ∈Q.Đặta = pq, (p, q) = 1vàb= st, (s, t) = Trong (2.4) chon=qcó

0 =a{bq} ⇒ {bq}= 0⇒ {bq} ∈Z+ ⇒q .t.

Tương tự, ta cót q.Vậyq =t,do (2.4) tương đương với

s p qn =p s qn

,∀n∈Z+.

Vớiq >1, ta chọnn ≡

s (mod q), suy

sn q = q =

q ⇒s p qn =p q = p q Đặt

pn=qk+r⇒s

pn q

=s· r

q ⇒ p q =s

r

q ⇒p=s·r⇒p

s

Tương tự cós p.Do đóp=s,vơ lý doa6= b,do vậyq= 1vàa, b∈ Z.Tóm lại hoặca =b

hoặca, b∈Z

Ví dụ Cho a, blà số thực dương thoả mãn [2an+b]chia hết cho2với số nguyên dươngn Chứng minh rằngalà số nguyên dương.

Chứng minh. Nếualà số vơ tỷ ta có

[2an+b] = [2{an}+b] + [an] 2,∀n ∈Z+⇒[2{an}+b] .2,∀n∈

Z+ (2.5)

Lại do{{an} |n∈Z+}là trù mật trong(0; 1) Suy tồn số nguyên dươngnmà[b+ 2{an}]

không chia hết cho2.Thật vậy:

Nếu2k+ 1> b>2kthì chọnnsao cho2k+ >2{an}+b >2k+

Nếu2k+ > b> 2k+ 1thì chọnn cho2k+ >2{an}+b > 2k+ 1.Điều mâu thuẫn với (2.5) nên suy raaphải số hữu tỷ Nếuakhơng ngun đặta = pq, (p, q) = 1, q∈Z+, q >1 Khi viếtn=qt+r, 06r 6q−1thì

[2an+b] =

2

pt+ pr

q

+b

(61)

Tạp chí Epsilon, Số 09, 06/2016

Chọnr = 0thì[2pt+b] 2⇒[b] Do

[2an+b] = 2pt+ [b] +

2pr q +{b}

.2,∀t∈Z+,

tương đương với

2pr q +{b}

.2,∀t∈Z+,

hay pr q

+{b}

.2,∀t∈Z+.

Mà3>2nprq o+{b}>0suy rah2nprq o+{b}i ∈ {0; 2}.Ta xét hai trường hợp sau:

• Nếu tồn tạir0 6= 0sao cho

h

2npr0

q

o

+{b}i = 0,thì

1>2

pr0

q

+{b}>0⇒ > pr0 q ⇒ p2r0

q

=

2pr0

q = pr0 q , p2r0

q

+{b}=

pr0

q

+{b}>2

pr0

q

+{b}

Cứ làm suy tồn số nguyên dươngimà (vì lần thực giá trị biểu thức tăng lên)

      

p·2i ·r0

q

+{b}<1

2

p·2i+1·r

q

+{b}>1

Khi

2

p·2i+1·r

q

+{b}=

p·2i·r

q

+{b}<2

2

p·2i·r

q

+{b}

<2

Do

2

p·2i+1·r

q

+{b}

=

Gọir1 số dư2i+1·r0 chia choq,suy vơ lý • Nếu không tồn tạir0 6= 0sao cho

h n pr0 q o

+{b}i= 0,thì với mọir0 6= 0ta có

pr0 q

+{b}

= 2⇒3>2

pr0

q

+{b}>2,

1>{b}>0⇒2

pr0

q

>1⇒

pr0 q > ⇒ p·2r0

q

=

2· pr0

(62)

Cứ làm vậy, suy tồn số nguyên dươngisao cho (vì lần thực giá trị biểu thức giảm xuống)

      

pr0·2i

q

+{b}>2

2

pr0·2i+1

q

+{b}<2

Do2npr0·q2io+{b}>2,nên

2

pr0·2i+1

q

+{b}=

2

pr0·2i

q

−1

+{b}=

pr0·2i

q

+{b}−2>2

pr0·2i

q

>1

Suy rah2npr0·q2i+1o+{b}i= 1.Chọnr1 số dư chiar02i+1 choqsuy điều vơ lý

Tóm lạia∈Z+.

Ví dụ ĐặtA=23mn |m, n∈Z

+,2m >3n .Chứng minh inf(A) = 1.

Chứng minh. Ta đặtB ={log3a|a∈A}.Khi ta phải chứng minhinf (B) = log31 = Ta thấyb > 0,∀b ∈Bsuy inf(B)>0.Lại thấy với mỗib ∈B, bcó dạngnlog32−m Với số nguyên dươngnmàn·log32>1, xétm = [nlog32]∈Z+ Khi đó,b ={nlog

32} Lại thấy log32là số vô tỷ dương nên theo định lý Kronecker thì{{nlog32} |n∈Z+}trù mật trong(0; 1).

Do tồn dãy số nguyên dươngniđểbi ={nilog32} →0, khii→+∞

Do vậyinf (B) = 0, suy inf(A) =

Ví dụ 10 (Putnam 1995) ChoA(x) ={[nx]|n ∈Z+}.Chứng minh nếua, b, c∈

R+ thì

A(a), A(b), A(c),không thể phân hoạch củaZ+

Chứng minh. Giả sử phản chứng nếua, b, c∈R+ thìA(a), A(b), A(c)là phân hoạch của Z+.Ta xét trường hợp sau:

• Trong sốa, b, ccó số hữu tỷ, giả sử làa.Đặta= pq, (p, q) = 1, q∈Z+.Ta

chứng minh tồn số nguyên dươngm, nsao cho

[ma] = [nb] (2.6)

Nếub∈Qthì tồn tạim, n∈Z+mà

(

ma, nb ∈Z+

ma=nb

Khi ta có điều phải chứng minh Nếu b 6∈ Q chọn m = qt, t ∈ Z+. Khi đó [ma] = hqtpqi = pt Do b /∈ Qnên theo định lý Kronecker, tồn n0 ∈ Z+ cho {n0b}< 1p.Khi

[(pn0)b] = [p[n0b] +p{n0b}] =p[n0b] + [p{n0b}] =p[n0b]

Chọnt= [n0b]thì ta có điều phải chứng minh Vậy (2.6) đóA(a)∩A(b)6=∅,

(63)

Tạp chí Epsilon, Số 09, 06/2016

• Cả ba sốa, b, cđều vơ tỷ Xétn ∈Z+bất kỳ, số số thuộcA(a)mà nhỏ hơn

hay bằngnlà số sốm∈Z+thoả mãn

[am]6n ⇔am−1< n⇔m < n+

a ⇔m

n+

a

Do cón+1a số thuộcA(a)và nhỏ hay bằngn.Tương tự vớib, c.Khi (do giả sử)

X

a,b,c

n+

a

=n,∀n ∈Z+,

suy X a,b,c n+1 a n+ =

n n+

Chon →+∞có P

a,b,c

a = 1,suy ba sốa, b, cnhỏ hơn3,giả sử làa

Ta nhắc lại định lý Beatty sau:

Choα, β ∈I+ : α +

1

β = Khi đóA(α), A(β)là phân hoạch củaZ +.”

Trở lại toán, doa ∈ I+, a >1nên a a−1 ∈ I

+và a a−1 >

3

2 Khi đóA(a), A a a−1

là phân hoạch củaZ+ Khi đóA(b)∪A(c) = A a

a−1

Nhận xét Choα ∈I, α > 32 β ∈R+sao choA(β)⊆A(α),khi đó β α ∈Z

+. Chứng minh.DoA(β)⊆A(α)nên với mỗim ∈Z+ thì tồn tạin∈

Z+thoả mãn [mβ] = [nα]⇒nα−1<[mβ]< nα⇒ [mβ]

α < n <

[mβ]

α +

1

α (2.7)

Ta thấy tồn tạim ∈Z+mà0<n[mβ] α

o

<1−

α khơng tồn tạin∈Z

+thoả mãn

(2.7),

[mβ]

α

>1−1

α,∀m ∈Z

+⇔

mβ − {mβ}

α

>1−1

α ⇔

mx− {mβ}

α

>1−1

α,

x= β

α ⇔

{mx} − {mβ}

α

>1−

α,∀m∈Z

+. (2.8)

Nếux∈I, với mỗim∈Z+có

 

{mx} − {mβ}

α >1−

1

α

− {mx}+{mβ}

α >1−

1 α ⇒   

{mx}>1−

α +

{mβ}

α >1−

1

α

{mβ}

α +

1

α −1>{mx}

 

{mx}>1−

α

2

α −1>{mx}

1) Nếu α2 −160thì ta phải có{mx}>1−

α,∀m ∈Z

+,trái với định lý Kronecker.

2) Nếu α2 −1 > 0thì ta có 1− α >

2

α −1 α >

2, suy với mọim ∈ Z + thì {mx}∈/

α −1; 1− α

(64)

Do đóx /∈Isuy rax∈Q

Nếux /∈Z+, đặtx= p

q, (p, q) = 1, q >1 Xétm=qk+ 1, k ∈Z

+, đó

{mx} − {mβ}

α

=

(qk+ 1)p

q

− {(qk+ 1)β}

α = p q

−{k·qβ+β}

α

=

{x} − {kp·α+β}

α

(2.9)

Vì α ∈ I nên pα ∈ I, {{k(pα)} |k ∈Z+} trù mật trên (0; 1), từ suy ra {{k(pα) +β} |k∈Z+},trù mật trên(0; 1),dẫn đếnn{k(pα)+β}

α |k∈Z

+otrù mật trên 0;1α.Suy tồn tạik0 ∈Z+sao cho

1>{x}> {k0pα+β}

α >{x}+

1

α−1

do1>{x}>{x}+

α −1;

1

α >{x}+

1

α −1

Khi đó, từ (2.7) xétm=qk0+ 1, ta có

{mx} − {mβ}

α

=

{x} − {k0pα+β}

α

={x} −{k0pα+β}

α <{x} −

{x}+

α −1

= 1−

α,

trái với (2.9) Do đóxphải thuộcZ+, hay β α ∈Z

+và nhận xét chứng minh.

Trở lại với toán,

A(b)∪A(c) = A

a a−1

⇒A(b), A(c)⊂A

a a−1

,

mà a−a1 > 32 nên tồn tạib0 ∈Z+sao chob= a−a1 ·b0 vàc0 ∈Z+sao choc= a−a1 ·c0.Đặt

d= a−a1 ·b0c0

A(d)⊂A(b), A(d)⊂A(c),

điều vơ lý doA(b)∩A(c) =∅

Vậy điều giả sử ban đầu sai, ta có điều phải chứng minh

Nhận xét Cho ∈ R+(i= 1, 2, , n)đơi phân biệt, nếu A(a1), A(a2), ,

A(an)là phân hoạch củaZ+thìn ∈ {1; 2}

Tiếp theo nghiên cứu số toán mà cách giải dùng phương pháp tập trù mật

Ví dụ 11 (France TST2005) ChoA ⊂ N∗, A 6=∅sao cho ∀x ∈ Athì4x,[√x] ∈ A.Chứng

(65)

Tạp chí Epsilon, Số 09, 06/2016

Chứng minh. Ta gọia0 số nguyên dương bé thuộcA.Nếu a0 >

√ a0

∈ Avà

√ a0

6√a0 < a0,mâu thuẫn với cách chọna0 vậya0 = 1.Từ giả thiết suy

4n ∈A,∀n⇒h√4ni= 2n ∈A,∀n⇒h(2n)21mi∈A,∀m, n. (2.10)

Ta có nhận xét sau

∀k ∈N∗,∃m, n∈N∗ :k2m 62n<(k+ 1)2m

Thật vậy, chọnm∈N∗ để2m[log

2(k+ 1)−log2k]>2,

log2(k+ 1)2m −log2k2m >2⇒ ∃n ∈N∗ : log2k2m 6n <log2(k+ 1)2m,

suy

k2m 62n<(k+ 1)2m ⇒k =

h

(2n)21m i

Từ (2.10) suy rak∈AnênN∗ ⊂A Do đóA=N∗

Ví dụ 12 (VMO1997, bảng A) Cho số tự nhiênn >1không chia hết cho1997 Xét hai dãy số (ai)(bj)được xác định sau:

ai =i+

ni

1997, i= 1,2, ,1996, bj =j+ 1997j

n , j = 1,2, , n−1

Xét tất số hai dãy thứ tự không giảm ta đượcc1 6c2 6· · · 6c1995+n Chứng minh rằngck+1−ck<2,∀k= 1,2, ,1994 +n

Chứng minh. Thay1997bởi sốmkhông ước củan.Xét hai dãy số

ai =i

1 + n

m

, i= 0,1, , m, bj =j

1 + m

n

, j = 0,1, , n

Ta có

a0 = < a1 <· · ·< am =m+n, b0 = 0< b1 <· · ·< bn=m+n

Nếun < mthìai+1−ai = + mn <2.Với mỗikmà16k 6m+n−2thì tồn nhấtj

đểaj 6ck< aj+1với06j 6n−1 Khi đóck+1 6aj+1

ck+1−ck 6aj+1−aj <2

Nếum < nthìbj+1−bj = +mn <2 Với mỗikmà16k 6m+n−2thì tồn nhấtj

đểbj 6ck < bj+1với06j 6n−1.Khi đóck+1 6bj+1và

ck+1−ck 6bj+1−bj <2

Ta có điều phải chứng minh

Ví dụ 13 Vớiα >1là số thực cho∀m, n∈ N∗, m . n ⇒[mα] . [nα] Chứng minh α∈N∗.

Chứng minh. Giả sử α = t + r, t ∈ N∗,0

6 r 6 Từ giả thiết ∀k, n ∈ N∗ sao cho

[knα] [nα],ta có

[knα] = [knt+knr] =k[nt] + [knr]≡[knr] (mod nt),

suy ra∀k, n ∈ N∗.Lại có[knr] . [nα].Cố địnhk, chon → +∞cónt → +∞mà [knr]bị

(66)

Ví dụ 14 (IMO 1997) Chox1, x2, , xn∈Rthoả mãn n X i=1 xi

= 1, |xi|6

n+

2 ,∀16i6n

Chứng minh tồn hoán vị(y1, y2, , yn)của(x1, x2, , xn)sao cho

n X i=1 iyi

6 n+

Chứng minh. Với hoán vịΠ = (y1, y2, , yn)của(x1, x2, , xn), ta kí hiệuS(Π)là giá

trị tổngy1+ 2y2+· · ·+nyn Đặtr= n+12 Ta cần phải chứng minh tồn tạiΠnào để |S(Π)|6r GọiΠ0là hốn vị đồng nhấtΠ0 = (x1, x2, , xn)và gọiΠlà hoán vị đảo ngược Π = (xn, xn−1, , x1) Nếu|S(Π0)|6rhoặc

S Π6r, toán coi giải xong,

do vậy, ta giả sử|S(Π0)|> rvà

S Π> r Để ý

S(Π0) +S Π

= (x1+ 2x2+· · ·+nxn) + (xn+ 2xn−1+· · ·+nx1) = (n+ 1) (x1+x2+· · ·+xn)

Ta suy raS(Π) +S Π

=n+ = 2r Nhưng vì|S(Π0)| > rvà

S Π

> r nên ta phải

cóS(Π0)vàS Π

trái dấu nhau, nghĩa số lớn hơnr, cịn số nhỏ hơn−r Từ

Π0, ta thu đượcΠm = Πbằng cách chuyển hai phần tử kề Nói cách khác, tồn

một dãy hoán vịΠ0,Π1, ,Πm choΠm = Πvà với mỗii(i= 0,1, , m−1), hốn

vịΠi+1có từΠi cách hoán chuyển hai số hạng liên tiếp Điều có nghĩa Πi = (y1, y2, , yn),Πi+1 = (z1, z2, , zn)thì tồn sốk ∈ {1,2, , n−1}sao

cho

zk=yk+1, zk+1 =yk, zj =yj, j 6=k, k+

Do giả thiết|xi|6r,∀i= 1,2, , nnên ta có

|S(Πi+1)−S(Πi)|=|kzk+ (k+ 1)zk+1−kyk−(k+ 1)yk+1|=|yk−yk+1|62r

Điều chứng tỏ khoảng cách hai số liên tiếp dãy sốS(Π0), , S(Πm)

không lớn hơn2r Mặt khác, xemS(Π0), S(Πm)là điểm nằm đường thẳng

thực, nằm ngồi đoạn[−r;r], từ đó, suy sốS(Πi)phải rơi vào đoạn

đó; nói cách khác; tồn tạiΠi đểS(Πi)6r

Trên chúng tơi nói khái niệm tập hợp trù mật số ứng dụng giải tốn sơ cấp nhằm phục vụ cho kỳ thi Olympic tốn phổ thơng Việc tiếp tục nghiên cứu vấn đề nói trương trình tốn bậc đại học bậc học

3 Bài tập tự luyện

Bài tập ChoX, Y là hai tập củaRsao choXtrù mật trongRX ⊂Y.Chứng minh rằngY trù mật trongR

(67)

Tạp chí Epsilon, Số 09, 06/2016

Bài tập Chứng minh tồn vô số(m, n)∈Z2sao cho

n√2 = 2m. Bài tập (Isarel – Hungarian1998) Tìm tất cảα∈Rthoả mãn

∀n ∈N∗,∃m∈Z:

α−

m n

<

1 3n

Bài tập Cho a > blà hai số nguyên dương thoả mãn ab > Chứng minh tồn số nguyên dươnglsao chob < ϕ 2l

= 2l−1 <2l < a. Bài tập Cho tập hợpS=

n√

a−√b|a, b∈No Chứng minh tậpStrù mật trongR.

Bài tập Cho dãy số(an)n>0(bm)m>0thoả mãn đồng thời điều kiện sau: lim

n→+∞an =mlim→+∞bm = +∞,

lim

m→+∞(bm+1−bm) =

Chứng minh tậpS ={an−bm|m, n∈N∗}là trù mật trongR

Bài tập Cho(an)(bn)là hai dãy số dương, tăng ngặt không bị chặn Đặc biệt, tập hợp {|an+1−an| |n ∈N∗}bị chặn.

Chứng minh tập hợpS=nam

bn |m, n∈N

∗otrù mật trong

R+ Bài tập Cho hàmf :R+→

R+thoả mãn điều kiện sau: (i) f khả vi trênR+

(ii) lim

x→+∞f(x) = +∞

(iii) |f0(x)|< M,∀x∈R+\(0; 1).

Cho(bn)là dãy số dương, tăng không bị chặn Chứng minh tậpS =

n

f(m)

bn |m, n∈N

∗o

trù mật trongR+ Do tậpn

n

2m |m, n∈N

∗otrù mật trong

R+ Bài tập 10 Cho a, b ∈ Z+ sao cho s(an) = s(bn),∀n ∈

Z+.Chứng minh rằnglogab ∈ Z Trong đó,s(x)là tổng chữ số củaxviết hệ nhị phân.

Tài liệu tham khảo

[1] Titu Andreescu, Gabriel Dospinescu,Problems from the book

(68)(69)

ABOUT MEANS OF NON-NEGATIVE NUMBERS AND POSITIVE DEFINITE

MATRICES

Đinh Trung Hòa

(Department of Fundamental Sciences, Ho Chi Minh University of Food Industry)

Trong viết ngắn này, tác giả Đinh Trung Hòa giới thiệu lý thuyết trung bình số khơng âm ma trận xác định dương Khởi đầu từ khái niệm quen thuộc trung bình cộng, trung bình nhân, trung bình điều hịa hai số, tác giả đến khái qt hóa khái niệm trung bình Tiếp theo viết giới thiệu tính chất đặc trưng hàm đơn điệu liên quan đến trung bình Cuối cùng, tác giả giới thiệu định nghĩa dạng trung bình hai ma trận Từ định nghĩa bất đẳng thức số hàm, vấn đề tương ứng với ma trận đặt cách tự nhiên (và tác giả với số tác giả khác thu số kết theo hướng này)

Bài viết dựa nói chuyện TS Đinh Trung Hịa Đà Nẵng (cho nhóm học sinh THPT chun Lê Quý Đôn Đà Nẵng chuẩn bị dự thi HSG quốc gia năm2014) buổi seminar dành cho học sinh sinh viên trung tâm Titan Education (năm2015)

Đây viết ngắn, văn phong dễ đọc nên để nguyên tiếng Anh Để độc giả tiện theo dõi, xin chuyển ngữ giải thích số thuật ngữ tiếng Anh

Positive definite matrices = ma trận xác định dương Positive semidefinite = nửa xác định dương

Successive iterations = bước lặp liên tiếp

Arithmetic, geometric, harmonic means = trung bình cộng, trung bình nhân, trung bình điều hịa

Two-variable function = hàm2biến Monotone increasing = đơn điệu tăng One-to-one map = ánh xạ1−1

Reverse Cauchy inequality = bất đẳng thức Cauchy ngược Convex function = hàm lồi

Classification = phân loại

Quantum information theory = Lý thuyết thông tin lượng tử Matrix optimization = Tối ưu hóa ma trận

(70)

Eigenvalues = giá trị riêng Eigenspaces = không gian riêng

Orthogonal projections = hình chiếu vng góc Axiomatic approach = cách tiếp cận tiên đề Binary operation = phép tốn2ngơi

Riemannian manifold = đa tạp Riemann

Weighted Heron mean = trung bình Heron có trọng Norht and south poles = bắc cực nam cực

Mathematical aspects = khía cạnh tốn học

Theory of operator and matrix = lý thuyết toán tử ma trận

In this short we introduce the theory of means for non-negative numbers and positive definite matrices

1 Some information about means for scalars 1.1 Some well-known means

We list some (not all) definitions of the geometric mean as follows:

• The unique solution ofxa−1x=b;

• The common list of the successive iterations of harmonic and arithmetic means:

ab= liman= limbn,

wherea0 =a, b0 =b, an+1 = a2na+nbbnn, bn+1 = an+2bn • The maximum among realxfor which

a x x b

is positive semidefinite, or its determi-nant is nonnegative:x2 6ab.

• The unique midpoint for the metricδonR+given byδ(x, y) =

log

x y

: δ(a, b) = 2δ(a,√ab) = 2δ(b,√ab);

• If x(t) is the solution of x0 = b −a−1x with initial condition x(0) = x

0 > 0, then limt→∞x(t) =

ab

It is well-known that for two nonnegative numbersa, bands∈[0,1]

(a

−1+b−1

2 )

−1

6√ab6 a

sb1−s+a1−sbs

2

a+b

2 , (1.1)

where(a−1+2b−1)−1 is the harmonic mean,asb1−s+a1−sbs

2 is the Heinz mean and a+b

2 is the

(71)

Tạp chí Epsilon, Số 09, 06/2016

The AM-GM inequality states that only the square has the smallest perimeter amongst all rectangles of equal area At the same time, we have the following geometric interpretation: The geometric mean and the arithmetic mean are the midpoints of curvesasb1−sandsa+ (1−s)b,

respectively, connecting aand b And the Heinz mean is the curve connecting the geometric mean and the arithmetic mean whensruns from0to1

Now, let us formulate a general approach of means A meanM of nonnegative numbers is a map fromR+×

R+toR+such that:

1) M(x, x) = xfor everyx∈R+;

2) M(x, y) =M(y, x)for everyx, y ∈R+;

3) Ifx < y, thenx < M(x, y)< y;

4) Ifx < x0andy < y0,thenM(x, y)< M(x0, y0);

5) M(x, y) is continuous;

6) M(tx, ty) = tM(x, y) (t, x, y∈R+.

A two-variable function M(x, y) satisfying condition (6) can be reduced to a one-variable function

f(x) :=M(1, x) (1.2)

Namely,M(x, y)is recovered fromf asM(x, y) = xf(x−1y) Mention that the corresponding

toM functionf is monotone increasing onR+ And that relation (1.2) forms a one-to-one map between means and monotone increasing functions onR+.

It is obvious that:

• The arithmetic mean is corresponding to 1+2t;

• The geometric mean is corresponding to√t;

• The harmonic mean is corresponding to 1+2tt;

• Thelog-meanL(x, y) = logxx−−ylogy is corresponding to t−1 logt

1.2 Monotone functions and means

Letf be a monotone increasing function on[0,∞), then for any nonnegative numbera, b,

f(√ab)6f

a+b

2

(1.3)

It is natural to ask that if inequality (1.3) holds for any pair of positive number a, bwill the function f be monotone increasing on[0,∞)? The answer is positive, and follows from the elementary fact that for any positive numbersa6bthere exist positive numberx, y such thata

is arithmetic mean andbis geometric mean ofx, y

An interesting and useful reverse Cauchy inequality is as follows: For any positive numberx, y,

x+y

2

xy+

2|x−y|, (1.4)

or

(72)

Proposition A function f on [0,∞) is monotone increasing if and only if the following inequality

f(x+y )6f

xy+1

2|x−y|

, (1.5)

hold for any pair of positive numberx, y.

Chứng minh. Firstly, we show that for06a6b 6√2athere exist positive numberx, ysuch that

a= x+y

2 , b = √

xy+

2|x−y|

We can assume thatx6a It is easy to see that for06a6b 6√2athe equation

b =px(2a−x) +a−x,

or,

2x2+ 2(b−2a)x+ (b−a)2 =

has a positive solution Consequently, if we have (1.5), then

f(a) = x+y 6f

xy+1

2|x−y|

=f(b)

For arbitrarya6b, it is obvious that there exist numbersa1, a2, , am such that

a =a0 6a1 6· · ·6am =b and 6ai+1 √

2ai

Apply above arguments, we can getf(a)6f(a1)6f(a2)6· · ·6f(an) =f(b)

Of course we can get new characterization of monotone increasing functions by using other inequalities in (1.1)

1.3 Some classes of convex functions with means

LetM, N be arbitrary means andf be continuous onR+ We callf to beM N-convex if for any nonnegativex, y,

f(M(x, y))6N(f(x), f(y)) (1.6) If M(x, y) = N(x, y) = sx+ (1 −s)y, then we have the class of convex functions, and inequality (1.6) is no thing but Jensen inequality for convex functions Such functions have many applications in optimization, in information theory etc

IfM(x, y) =sx+ (1−s)y, N(x, y) = xsy1−s, then we have the class oflog-convex functions.

IfM(x, y) = N(x, y) = xsy1−s, then we have the class of geometrically convex functions.

(73)

Tạp chí Epsilon, Số 09, 06/2016

2 A little about matrix means

In this section we talk about the matrix means that have many applications in quantum information theory, matrix optimizations etc

LetMnbe the space ofn×ncomplex matrices ForA, B ∈Mn, the notationA 6B means that

B−A∈M+

n (i.e.,<(B −A)x, x >>0for any vectorx) The spectrum of a matrixA∈Mn

is denoted byσ(A) For a real-valued functionf of a real variable and a matrixA ∈ Mh n, the

value f(A) is understood by means of the functional calculus for Hermitian matrices, i.e if

A=P

λiPi (whereλi are eigenvalues ofA,Pi are orthogonal projections on the eigenspaces of

λi), then the matrixf(A)is defined as

P

f(λi)Pi We can understandf(A)in another way: Let

A=U∗diag(λ1, λ2, , λn)U (whereλiare the eigenvalues ofA), then

f(A) = U∗diag(f(λ1), , f(λn))U

Taking an axiomatic approach, Kubo and Ando [6] introduced the notions of connection and mean A binary operationσdefined on the set of positive definite matrices is called aconnection

if

(i) A6C, B 6DimplyAσB 6BσD;

(ii) C∗(AσB)C6(C∗AC)σ(C∗BC);

(iii) An↓AandBn↓B implyAnσBn ↓AσBIfIσI =I,thenσis called amean

ForA, B >0, thet-geometric meanof positive definite matricesA, B is

A]tB :=A1/2 A−1/2BA−1/2

t

A1/2 (2.1)

When t = 1/2, A]B := A]1/2B is called the geometric mean of A and B, and it was first

introduced in [8] and is often denoted byA]B in the literature (the problem of defining the geometric mean of two positive definite matrices was standing for more than 25 years because we could not just take the square root of product of two positive definite matrices AB!!) The harmonic A!B and arithmetic A∇B means are defined A!B = 2(A−1 +B−1)−1 and

A∇B = A+B

2 , respectively A meanσis called to be symmetric ifAσB =BσAfor any pair

of positive definite matricesA, B

It is well-known that the arithmetic mean∇is the biggest among symmetric means From the general theory of matrix means we know that∇>σ andτ >! And we still have the following inequalities

A!B 6A]B 6A∇B (2.2)

The proof of (2.2) is not very difficult and based on the scalar case

Thet-geometric mean is interesting from the point of view of Riemannian geometry since the set

Pnof positive semidefinite matrices is a Riemannian manifold with the Riemannian metric [9]:

δ(A, B) = n

X

i=1

log2λi(A−1B)

!1/2

, A, B ∈Pn,

which is the distance betweenAandB so that the curveβ(t) =A]tB,06t61, is the unique

geodesic joiningAand B in Pn The picture is more interesting if we consider the weighted

(74)

3 Concluding comment

Many properties formulated in Subsection 1.1 have been studied for matrices [10] Some in-teresting information related to the last part of Subsection 1.1 an be found in [1] (Petz is one of the most active mathematicians in studying mathematical aspects of quantum information theory) where the author discusses about means of more than positive semidefinite matrices Matrix analogs of results in Subsection 1.2 recently was studied by Ando and Hiai [11] and by the author [12] At that time, problems of characterization of matrix convex functions in sense of Subsection 1.3 are still in research interests of many mathematicians Nowadays, the theory of operator and matrix means is one of the most studied in matrix analysis with many applications in machine learning and quantum theory etc

Tài liệu tham khảo

[1] D.Petz.Means of positive matrices: Geometry and a conjecture.Annales Mathematicae et Informaticae.32(2005) 129-139

[2] F Topse Basic Concepts, Identities and Inequalities theToolkit of Information Theory.

Entropy,3(2001) 162

[3] K M R.Audenaert, J Calsamiglia, LI Masanes, R Munoz-Tapia, A Acin, E Bagan and F Verstraete, Discriminating States: The Quantum Chernoff Bound, Phys Rev Lett.98

(2007) 160501

[4] D Bini, B Meini, F Poloni,An effective matrix geometric mean satisfying the Ando-Li-Mathias properties, Math Comp.79(2010) 437-452

[5] S Furuichi, K Yanagi, and K Kuriyama,Fundamental properties of Tsallis relative entropy, J Math Phys.45(12) (2004) 4868-4877

[6] F Kubo and T Ando,Means of positive linear operators, Math Ann.246(3) (1980) 205-224

[7] H Lee, Y Lim, T Yamazaki,Multi-variable weighted geometric means of positive definite matrices, Linear Algebra Appl.435(2) (2011) 307-322

[8] W Pusz, S.L Woronowicz, Functional calculus for sesquilinear forms and the purification map, Rep Math Phys.,8(1975), 159-170

[9] R Bhatia,Positive Definite Matrices, Princeton University Press, New Jersey, 2007

[10] J D Lawson, Y Lim, The geometric mean, matrices, metrics, and more,Amer Math Monthly108(2001), 797-812

[11] T Ando, F Hiai, Log majorization and complementary Golden-Thompson type inequalities, Linear Algebra Appl.,197/198(1994), 113–131

(75)

ĐIỂM KOSNITA VÀ MỘT SỐ ĐƯỜNG THẲNG ĐI QUA NĨ

Trịnh Huy Vũ (Lớp 12 A1 Tốn, trường THPT Chuyên KHTN, Hà Nội)

Bài viết nêu định nghĩa số kết điểm Kosnita Đồng thời báo đưa chứng minh túy cho số đường thẳng qua điểm Kosnita

1 Định nghĩa

Định nghĩa ĐiểmKosnitaK của 4ABC bất kì xác định điểm đồng quy ba đường thẳng nối ba đỉnhA, B, Ctương ứng với tâm ngoại tiếp của4OBC,4OCA,4OABvới

Olà tâm ngoại tiếp của4ABC ĐiểmKosnitaKimberlingcenterX(54)trong Encyclopedia of Triangle Center (ETC) [1].

A

B C

O

D

E F

K

Hình 11.1: Điểm Kosnita

2 Một số kết quen thuộc

(76)

Kết 4ABC có tâm đường trịn EulerN và điểm KosnitaK Khi đó,N K đẳng giác trong4ABC.

Định nghĩa Cho tam giácABCvới tâm nội tiếpI Đường thẳngEulercủa4IBC,4ICA,

4IAB,4ABC đồng quy điểm Điểm đồng quy gọi điểm Schiffler của 4ABC.

Kết GọiD, E, F lần lượt trung điểm cung nhỏBC, CA, AB của 4ABC. Khi đó, điểmKosnitacủa4DEF cũng điểmSchifflercủa4ABC.

Chứng minh. X trung điểmBC Vẽ đường kínhDJ Lấy điểmD0, E0, F0 đối xứngD, E, F quaBC, CA, AB GọiN, U, V, W tương ứng trung điểm củaIO, ID0, IE0, IF0 Ta có kết quen thuộc:D, E, F tương ứng tâm ngoại tiếp tam giácIBC , ICA , IAB;I, N trực tâm tâm đường trịnEulercủa4DEF

Ta có DI2 = DB2 = DC2 = DX.DJ (theo hệ thức lượng tam giác vuông) Từ ta đượcDI2 = DX.DJ = 2DX.DO = DD0.DO Suy ra DO

DI = DI

DD0 Do đó,4DOI ∼

4DID0(c.g.c) Mà hai tam giác có hai trung tuyến tương ứng DN DU,

∠N DI =∠U DO Nói cách khác,DN vàDU đẳng giác trong∠IDO Mặt khác, doI, Otương úng trực tâm tâm ngoại tiếp của4DEF nênDH, DOđẳng giác trong∠EDF Từ ta thu đượcDN vàDU đẳng giác trong∠EDF Mặt khác ta lại cóU tâm đường trịnEuler

của4IBC nênDU đường thẳngEulercủa4IBC VậyDN đường thẳngEuler

của 4IBC đẳng giác 4DEF Chứng minh tương tự ta EN, đường thẳng

Eulercủa 4ICA vàF N, đường thẳng Eulercủa 4IAB hai cặp đường thẳng đẳng giác trong4DEF Do điểmSchifflerSđẳng giác vớiN qua4DEF TheoKết 1, điểm

SchifflerScủa4ABC điểm Kosnita của4DEF

A

B C

O

D

E

F

I

D0 J

N U

X

(77)

Tạp chí Epsilon, Số 09, 06/2016

Nhận xét Kết 2cịn có cách chứng minh khác Telv Cohltrong [4] Từ kết thu hệ điểmKosnitacủa4DEF nằm đường thẳngEulercủa4ABC Hơn nữa, dựa trùng điểmSchifflervà điểm Kosnitacủa4ABC và4DEF, ta thu chứng minh kết sau:

Kết GọiD, E, F lần lượt trung điểm cung nhỏBC, CA, AB của đường tròn (O)ngoại tiếp4ABC.Hlà trực tâm của4ABC K là điểmKosnitacủa4DEF Khi đó,

OK OH =

R

2r+ 3R vớiR, r tương ứng bán kính đường tròn ngoại tiếp nội tiếp tam giác ABC.

A

B C

O

D

E

F

I H

Ha

G

K L

X

Hình 11.3: Tỉ sốOK/OH

Trước hết ta nêu bổ đề quen thuộc, tác giả xin phép không nêu chứng minh bổ đề

Bổ đề Cho 4ABC với trực tâm H, tâm ngoại tiếp O M là trung điểm BC Khi đo,

AH = 2OM.

Chứng minh. L hình chiếu củaI lên AC.X trung điểm BC GọiHa trực tâm 4IBC DoK nằm đường thẳngEulercủa4IBCnênK ∈DHa ĐặtDHacắtAHtạiG

Do4ALI ∼ 4BXD(g.g) nên DI

AI = DB

AI = XD

IL = XD

r Từ áp dụng định lý Thales, ta

suy IHa

AG = DI DA =

XD

XD+r Theo bổ đề

2DX AG =

XD

(78)

suy HG = AH +AG = 2OX + 2(XD+r) = 2(r +R) Theo định lý Thales suy

OK KH =

OD HG =

R

2(r+R) hay

OK OH =

R

2r+ 3R

3 Một số đường thẳng qua điểm Kosnita

Trong mục tác giả phát biểu số đường thẳng qua điểm Kosnita Hầu hết đường thẳng liệt kê ETC [1] nên tác giả phát biểu lại đồng thời đưa chứng minh túy hình học cho chúng

3.1 Đường thẳng qua ảnh nghịch đảo trực tâm H qua đường tròn ngoại tiếp (O) và tâm đường tròn Taylor (X(186)X(389))

Định nghĩa 4ABC với A0, B0, C0 tương ứng chân ba đường cao. AB, AC lần lượt là hình chiếu củaA0 trênAB, AC Xác đinh điểmBC, BA, CA, CB tương tự Khi đó, điểm

AB, AC, BC, BA, CA, CB cùng nằm đường tròn Đường tròn gọi đường trònTaylor[5] Tâm đường trònTaylortrong ETC làKimberlingcenterX(389)[1].

A

B A0 C

B0

C0 AB

AC

CB BC

CA

BA

Hình 11.4: Đường tròn Taylor

Chứng minh đường tròn Taylor. Ta thực phép biến đổi góc sau:

∠AABAC =∠AA0AC (A, AB, A0, AC nằm đường tròn(AA0)) =∠ACB (cùng phụ∠A0AC)

=∠AC0B0(B, C, B0, C0 nằm đường tròn(BC)) =∠ACABA(B0, C0, BA, CAnằm đường tròn(B0C0))

Do điểmAB, AC, BA, CAcùng thuộc đường tròn Chứng minh tương tự thìBABCCBAB

vàCACBBCAC tứ giác nội tiếp đường tròn Giả sử ba đường trịn khơng trùng

nhau ta thấyBCCB, CAAC, ABBAlần lượt trục đẳng phương đường

tròn{(ABACBACA); (BABCCBAB); (CACBBCAC)}, tức làBCCB, CAAC, ABBAphải đồng

quy (mâu thuẫn) Do vậy, đường tròn trùng điểmAB, AC, BC, BA, CA, CBcùng

(79)

Tạp chí Epsilon, Số 09, 06/2016

Các chứng minh khác cho đường tròn Taylor tham khảo thêm [6], [7]

Định nghĩa Cho4ABC vớiD, E, F lần lượt trung điểm củaBC, CA, AB Khi đó, tâm nội tiếp của4DEF được gọi điểmSpiekercủa4ABC.

Sau tác giả trình bày thêm tính chất liên quan đến tâm đường trònTaylorX(389)và điểmSpieker

Bổ đề 4ABC nhọn vớiA0, B0, C0 lần lượt ba chân đường cao Khi đó, tâm đường trònTaylorcủa4ABClà điểmSpiekercủa4A0B0C0.

A

B A0 C

B0

C0 AB

AC

CB BC

CA

BA

D

E

F S

Hình 11.5

Chứng minh. S tâm đường trònTaylorcủa4ABC GọiD, E, F trung điểm

B0C0, C0A0, A0B0 Trước hết ta chứng minhABAC quaE, F Thật vậy, doElà trung điểm

C0A0nên∠EABA0 =∠C0A0AB = 90◦−∠BC0A0 = 90◦−∠ACB =∠A0AAC =∠ACABA0

Do đó, E ∈ ABAC Tương tự với F suy ABAC qua E, F Chứng minh tương tự

trên ta đượcBABC quaF, D vàCACB quaD, E Mặt khác dễ thấyBACA k CBBC nên

BACABCCBlà hình thang cân, màD=BABC ∩CACB suy ra4DBCCB tam giác cân

vàDSlà phân giác của∠BCDCB ≡∠EDF Tương tự,ESlà phân giác của∠DEF VậyS

tâm nội tiếp của4DEF điểmSpiekercủa4A0B0C0

Nhận xét. Kết 4ABC tam giác nhọn Khi 4ABC tù

A, B, C tâm đường tròn Taylorcủa 4ABC tâm bàng tiếp tương ứng với đỉnh

D, E, F tam giác trung bìnhDEF của4A0B0C0

Tác giả xin tiếp tục nêu thêm kết tiếng liên quan đến điểmSpieker:

Bổ đề 4ABC với tâm nội tiếpI, trọng tâmGvà điểmSpiekerSp và điểmNagelNa Khi đó,I, G, Sp, Nacùng nằm đường thẳng vàINa = 2ISp = 3IG.

Đây định lý tiếng, bạn đọc tham khảo chứng minh từ trang 7-12 [6]

(80)

A B C O H K D E F H∗ X Z Y S N J L M R P G Q Ho

Hình 11.6: Định lý 3.1

Trong đó, ảnh nghịch đảo H∗ H (O)là Kimberling center X(186) ETC [1] Trong viết tác giả nêu chứng minh định lý 1 trường hợp 4ABC nhọn (các trường hợp lại chứng minh tương tự) Khi đó, tâm đường trịnTaylorchính điểm

Spiekercủa tam giácDEF Do vậy, ta quy toán việc chứng minh đường thẳng qua điểmSpiekercủa4DEF ảnh nghịch đảo củaHqua(O)đi qua điểmKosnitaK

Chứng minh định lý. N tâm đường tròn Eulercủa 4ABC vàP trung điểm N O

X, Y, Z, J, L, M tương ứng trung điểm củaEF, F D, DE, HA, HB, HC Dễ thấyHlà tâm nội tiếp của4DEF (kết tiếng) GọiHo, Glần lượt trực tâm trọng tâm của4DEF

Ta Ho, G, N thẳng hàng đường thẳng Euler 4DEF H, G, S thẳng hàng,

HG = 2GS Gọi Qlà trung điểm HK.r, Rlần lượt bán kính đường trịn nội tiếp ngoại tiếp của4DEF

Ta cóOA2 = OH.OH∗ suy ra OH OH∗ =

AH AH∗ = OH OA

= N H

R2 Mặt khác, áp dụng

công thức Euler ý H, N tâm nội tiếp tâm ngoại tiếp 4DEF, N H2 = Rr2 −2Rr = R(R −2r) Do đó, OH

OH∗ =

R−2r

R Vậy

OP OH∗ =

R−2r

4R hay P H∗

OH∗ =

3R+ 2r

4R

Xét phép vị tự tâmHtỉ số

2,A 7→J;B 7→L;C7→M;K 7→Q, đóQlà điểmKosnitacủa 4J LM Mặt khác, ta có kết tiếngJ, L, M trung điểm cungEF, F D, DE

(81)

Tạp chí Epsilon, Số 09, 06/2016

N Q N Ho

= R

2r+ 3R Suy N Q N G =

3R

2r+ 3R Mặt khác, theo định lýThalesta có,OK = 2N Q

vàSP = 3N G

2 Do vậy,

P S OK =

3N G

2 2N Q =

3N G

4N Q =

3

2r+ 3R

3R =

2r+ 3R

4R

Vậy P H

∗ OH∗ =

P S

OK Suy ra4P H

∗S ∼ 4OH∗K(c.g.c) nên

∠P H∗S =∠OH∗K VậyH∗, S, K

thẳng hàng

3.2 Đường thẳng qua Trung điểmHG và tâm đường tròn Euler của tam giác tạo ba đường tiếp tuyến qua A, B, C(X(156)X(381))

Định lý Cho 4ABC với trực tâmH, trọng tâmGvà đường tròn ngoại tiếp(O) Các tiếp tuyến tạiA, B, Ccủa đường tròn(O)cắt tạo thành4DEF.I là trung điểm củaHG.L

là tâm đường trịnEulercủa4DEF Khi đó,ILđi qua điểmKosnitaK của4ABC

Trong đó, trung điểm I củaHG làKimberling centerX(381) của4ABC, tâm đường tròn

EulerLcủa tam giácDEF làKimberlingcenterX(156)của4ABCtrong ETC [1] Để chứng minhđịnh lý 2, tác giả xin phát biểu không chứng minh bổ đề quen thuộc sau:

Bổ đề Cho 4ABC với tâm ngoại tiếp O, đường tròn nội tiếp(I) Tiếp điểm của(I)với

BC, CA, ABtương ứng làD, E, F.Hlà trực tâm4DEF.R, rlần lượt bán kính đường trịn ngoại tiếp nội tiếp của4ABC Khi đó,OItrùng với đường thẳngEulerIHcủa4DEF

IH

OI = r R.

Chứng minh định lý. GọiH∗, O∗, S∗lần lượt trực tâm, tâm ngoại tiếp điểmSchifflercủa

4DEF.X, Y, Z tương ứng trung điểm cungEF, F D, DE đường tròn(O∗)ngoại tiếp4DEF Theokết 2thìS∗ điểmKosnitacủa4XY Z ĐặtR, R∗ bán kính đường trịn ngoại tiếp tam giácABC vàDEF

Do vng góc vớiODnênBC kY Z Tương tự ta cóCAkZX;ABkXY Do

4ABC và4XY Z vị tự nhau, màO, O∗tương ứng tâm ngoại tiếp của4ABC và4XY Z;

K, S∗ tương ứng điểmKosnitacủa4ABC và4XY Z Từ suy raOK kO∗S∗ ≡O∗H∗

và OK

O∗S∗ = OA O∗X =

R

R∗ Mặt khác, áp dụng kết 3 suy O∗S∗ O∗H∗ =

R∗

2R+ 3R∗ hay O∗S∗

O∗L =

2R∗

2R+ 3R∗ Vì vậy, ta được: OK

O∗L = OK O∗S∗ ·

O∗S∗ O∗L =

R R∗ ·

2R∗

2R+ 3R∗ =

2R

2R+ 3R∗

Mặt khác dễ thấy(O)cũng đường trịn nội tiếp của4DEF Theo bổ đề thìOO∗

trùng với đường thẳng Euler của4ABC, nói cách khác làO∗ ∈OH OH

OO∗ = R

R∗ MàI

trung điểm củaHGnênHI =IG=GO hay OI

OH =

2

3 Do đó,

OI OO∗ =

OI OH ·

OH OO∗ =

2 ·

R R∗ =

2R

3R∗ =⇒ IO IO∗ =

2R

(82)

A

B C

O H

G

D

E

F

L K I

X

Z Y

O∗

H∗

S∗

Hình 11.7: Định lý 2

Do ta IO

IO∗ = OK

O∗L Kết hợp vớiOK k O

∗Lsuy ra4IOK ∼ 4IO∗L(c.g.c), nên

[

OIK =\O∗IL Từ ta đượcI, K, Lthẳng hàng.

3.3 Đường thẳng qua trực tâm H và điểm Spieker tam giác tạo ba tiếp tuyến (X(4)X(6759))

Để cho thuận tiện, tác giả sử dụng ký hiệu dùng việc phát biểu chứng minhđịnh lý 2để phát biểuđịnh lý 3

Định lý GọiSplà điểmSpiekercủa4DEF Khi đó, đường thẳngHSpđi qua điểmKosnita

K của4ABC.

ĐiểmSpiekercủa tam giác tạo ba tiếp tuyến Kimberling centerX(6759)của4ABC

Chứng minh định lý. GọiG∗là trọng tâm của4DEF Ta cóG∗nằm trênH∗O∗(đường thẳng

Eulercủa tam giácDEF Theobổ đề 3, ýO tâm nội tiếp của4DEF vàSp điểm

Spiekercủa4DEF, suy raO, G∗, Sp thẳng hàng vàOG∗ = 2G∗Sp ĐặtIG∗cắtOKtạiT

Từ phép chứng minh định lý 2 ta OK k O∗L Áp dụng định lý Thales, ta

có OT

O∗G∗ = IT IG∗ =

T K

G∗L Từ suy OT T K =

O∗G∗

G∗L Mặt khác, O

∗G∗ = G∗L nên

OT = 2T K Từ OI

IH = OT T K =

OG∗ G∗S

p

(= 2) Theo định lýThalesđảo suy raHK k IT

(83)

Tạp chí Epsilon, Số 09, 06/2016

A

B C

O H

G

D

E

F

K I

H∗

Sp

U

V

W G∗

O∗

L T

Hình 11.8: Định lý 3

Nhận xét.Kết đường thẳng qua điểmH, K, Sp thực tế hệ củađịnh lý

2 Hơn nữa, ta thu hệ quả:

Hệ HK

HSp

= 2R

2R+ 3R∗

Chứng minh. Thật vậy, áp dụng định lýThaleschoIG∗ k HSp

HK IT =

OH OI =

HSp

IG∗ Từ

đó suy HK

HSp

= IT

IG∗ = IO IO∗ =

2R

2R+ 3R∗ (doOK kO ∗

H∗)

3.4 Đường thẳng qua đẳng giác điểm De Longchamps đối xứng điểm Exeter qua tâm ngoại tiếp O (X(64)X(378))

Định nghĩa Cho 4ABC với trực tâm H và tâm ngoại tiếpO Điểm De Longchamps của 4ABC được xác định điểm đối xứng trực tâm H qua tâm ngoại tiếpO [8] Điểmde LongchampsKimberlingcenterX(20)trong ETC [1].

Bổ đề Cho4ABC nội tiếp đường tròn(O).Dlà trung điểm củaBCH là trực tâm của 4ABC TiaDH cắt(O)tạiJ.J0 là điểm đường tròn(O)sao choJ J0 kBC Tiếp tuyến của(O)tạiB, Ccắt tạiT Khi đó,∠AJ0T = 90◦.

Chứng minh bổ đề. Dựng đường kínhAE của(O) ĐặtU =AH∩(O) Dễ thấyU E kBC k

J J0 GọiBB0, CC0 đường cao của4ABC Ta có kết tiếng làAJ, BC, B0C0đồng quy điểm, gọi điểm làX Ta cóA(BC, U X) = −1(chùm bản) Từ suy

A(BC, U J) =−1 Do vậy, tứ giácBJ CU tứ giác điều hòa Vi vậyJ, U, T điểm thẳng hàng Mặt khác, U E k BC k J J0 nên phép đối xứng qua trung trựcBC, ta suy

(84)

A

B D C

H

J J0

T O

E U

B0

C0 X

Hình 11.9: Bổ đề 5

Từ bổ đề ta chứng minh tính chất liên quan đến đẳng giác điểmde Longchamps

dưới

Bổ đề 10 Cho 4ABC nội tiếp đường tròn(O) Điểmde LongchampsLL0 là đẳng giác củaLtrong4ABC Các tiếp tuyến tạiA, B, Ccủa(O)cắt tạo thành4DEF Vẽ đường kínhAA0, BB0, CC0của(O) Khi đó,DA0 kAL0, EB0 kBL0, F C0 kCL0 Hơn nữa, ta biết

DA0, EB0, F C0 đồng quy điểmNagelNacủa4DEF (kết tiếng), thìNaL0 đối xứng quaO.

Chứng minh bổ đề. Đặt A0H cắt đường tròn ngoại tiếp(O)lần thứ hai tạiG LấyG0 ∈ (O)

sao choGG0 k BC Theobổ đề 5thì D, A0, G0 thẳng hàng Mặt khác,L đối xứngH quaO

nên AHA0L hình bình hành Suy A0L k AH A0L ⊥ BC Dặt A0L∩(O) = J

(J 6= A0) thì∠AJ A0 = 90◦, đóAJ k BC k GG0 Suy ra∠G0A0A = ∠HA0L = ∠HAL

(doAHA0Llà hình bình hành) MàL, L0 vàH, O hai cặp đẳn giác trong4ABC nên

∠L0AO = ∠HAL Do ∠L0AO = ∠G0A0A Suy AL k D, A0, G0 ≡ DA0 Tương tự EB0 kBL0, F C0 kCL0

Gọi L∗ đối xứngL quaO Ta đượcAL0A0L∗ hình bình hành A0L∗ k A0L Từ suy

L∗ ∈DA0 Tương tựL∗ ∈EB0 vàL∗ ∈F C0 VậyL∗ =DA0∩EB0∩F C0 nênL∗ ≡Na Do

đó,NavàLđối xứng quaO

Định nghĩa Cho4ABC với đường tròn ngoại tiếp(O) Các trung tuyến tam giácABC

cắt(O)lân lượt tạiX, Y, Z Các tiếp tuyến của(O)tại A, B, C cắt tạo thành tam giác

DEF Khi đó,DX, EY, F Z đồng quy điểm Điểm đồng quy gọi điểmExeter

của4ABC [9] ĐiểmExeterKimberlingcenterX(22)trong ETC [1].

Bổ đề 11 Cho4ABCnội tiếp đường tròn(O) Đường tròn nội tiếp(I)tiếp xúcBC, CA, AB

tạiD, E, F tương ứng Các đường trònMixtilinearứng với đỉnhA, B, C lần lượt tiếp xúc (O)tại X, Y, Z Ta có kết tiếng AD, BE, CF đồng quy điểm Gergonne Ge của 4ABC AX, BY, CZ đồng quy điểmJ là đẳng giác củaGetrong4ABC Hơn nữa,

J còn nằm trênOI J I

J O = r

(85)

Tạp chí Epsilon, Số 09, 06/2016 A B C D E F O A0 B0

C0 Na

L H L0 G0 G J

Hình 11.10: Bổ đề 6

Bình luận.Bổ đề kết tiếng nên tác giả xin phép khơng trình bày lại chứng minh Dựa vào bổ đề ta chứng minh tính chất quan trọng sau điểmExeter

Bổ đề 12 ĐiểmExeterEx nằm đường thẳngEulercủa4ABC Hơn nữa, ta có tỉ số

ExO

ExO∗ = R

R∗ vớiO, O ∗

là tâm ngoại tiếp của4ABC,4DEF R, R∗ là bán kính đường

trịn ngoại tiếp4ABC4DEF.

Chứng minh bổ đề. GọiHlà trực tâm của4ABC.M trung điểmBC vàX =AM ∩(O) Ta có tứ giácBOCDvàABXC nội tiếp nênM O·M D =M B·M C =M A·M X Do đó, tứ giácAOXDlà tứ giác nội tiếp Vậy∠XDO =∠XAO=∠AXO=∠ADOhayDO phân giác của∠ADX Suy raDAvàDX ≡DExlà đường đẳng giác trong∠EDF Tương tự

EB vàEEx;F C vàF Exlà cặp đường đẳng giác trong4DEF VậyEx đẳng giác

điểmGergonneGe (của4DEF) trong4DEF Theobổ đề 7thìEx nằm đường thẳng

OO∗, đường thẳng đồng thời đường thẳng Euler của4ABC, ExO

ExO∗ = R

R∗

Định lý Cho4ABCvới điểmDe LongchampsLvà điểmExeterEx.L∗ là điểm đẳng giác củaLtrong4ABC.Ex0 đối xứng vớiExquaO Khi đó,L∗Ex0 đi qua điểmKosnitacủa4ABC.

Chứng minh định lý. GọiHlà trực tâm của4ABC.O0 đối xứngO quaL Các tiếp tuyến

A, B, C của(O)cắt tạo thành tam giácDEF.Sp, Na điểmSpiekervà điểm

Nagelcủa 4DEF.O∗ tâm ngoại tiếp của4DEF.R, R∗ bán kính đường trịn ngoại tiếp của4ABC và4DEF

Theobổ đề 8thì ExO

ExO∗ = R

R∗ hay OEx

OO∗ = R

R+R∗ Mặt khác, lại theobổ đề 4, thìO

∗ ∈OH

và HO

OO∗ = R

R∗ Từ suy OEx0

HO = OEx

HO = OEx

OO∗ · OO∗

HO = R R+R∗ ·

R∗ R =

R∗

(86)

A

B C

D M

X O

E

F

H

Ex

O∗

Hình 11.11: Bổ đề 8

A

B C

O H

L

L∗

K

D

E

F

A0 B0

C0

Ex

E0 x

Na

J

O0 Sp

(87)

Tạp chí Epsilon, Số 09, 06/2016

HEx0 HO =

R

R+R∗, màHO

0 = 3HOnên ta được

HEx0 HO0 =

R

3(R+R∗) =⇒

HEx0 E0

xO0

= R

2R+ 3R∗ (1)

Chú ý rằngO, Sp, Nalần lượt tâm nội tiếp, điểm Spieker điểmNagelcủa 4DEF nên

ta Sp trung điểm ONa (xem bổ đề 3) Kết hợp với L∗ đối xứng Na qua O, nên ta

có OL∗ = ONa = 2OSp Từ theo định lý Thales,

HO OO0 =

OSp OL∗ =

nên ta

HK k L∗O0 L∗O0 = 2HSp Mặt khác, ta lại có

HK HSp

= 2R

2R+ 3R∗ (hệ 4) Suy ra, HK

L∗O0 = HK

2HSp

= R

2R+ 3R∗ (2)

Từ (1) (2) suy HE

0

x

E0

xO0

= HK

L∗O0 Mà ∠KHE

x = ∠L

∗O0E0

x (do HK k L

∗O0) Do đó,

4HKEx0 ∼ 4OL∗Ex0 Suy ra∠HEx0K = ∠O0Ex0L∗ = 180◦ −∠HEx0L∗ hay ∠KEx0L∗ = ∠HEx0K+∠HEx0L∗ = 180◦ Do vậy,L∗, Ex0, K thẳng hàng

3.5 Đường thẳng qua trực tâm tam giác Pedal của K và tâm ngoại tiếp O A B C K O D E F H

Hình 11.13: Định lý 6

Định lý Cho tam giácABCvới điểmKosnitaKvà tâm ngoại tiếpO.Hlà trực tâm tam giác pedal củaKứng với tam giácABC Khi đó, đường thẳngOH đi qua điểmKosnitaK.

Định lý tác giả đề xuất diễn đànAoPStrong [2] Để chứng minhđịnh lý 6, ta cần có bổ đề:

Bổ đề 13 Cho4ABC vớiP là điểm Các đường thẳng quaP lần lượt vng góc vớiAP, BP, CP cắtBC, CA, AB tương ứng tạiX, Y, Z Khi đó,X, Y, Z thẳng hàng.

(88)

Trở lại chứng minh định lý. Ta gọiN tâm đường trònEulercủa4ABC Các đường thẳng quaN vng góc vớiAN, BN, CN cắtBC, CA, AB tương ứng tạiX, Y, Z Theo bổ đề 1.1 thìX, Y, Z nằm đường thẳng Sau đây, để chứng minh định lý, tác giả chia chứng minh hai toán nhỏ sau đây:

Bài toán OK ⊥X, Y, Z.

A

B C

N K

O X

Z Y

X∗ L

Q R

I

Oa

P

Hình 11.14: Bài tốn 1

Chứng minh. GọiP, Q, R, Llần lượt trung điểmBC, CA, AB, OA.I, Oatương ứng tâm

ngoại tiếp của4AN Lvà4OBC Khi đó, ta cóN, Llà tâm ngoại tiếp của4P QRvà4ARQ MàA, Dđối xứng qua trung điểm quaQRnênL, N đối xứng qua trung điểmQR Mặt khác,LN lại trung trực củaQR Suy raL, N đối xứng quaQR, nói cách khác,QRlà trung trực củaLN Từ suy raI ∈QR

LấyX0 đối xứngAquaI thìX0 ∈ BC vàAX0 đường kính (AN L) Suy ra∠AN X = ∠ALX = 90◦ Từ ta đượcN X0 ⊥AN, X0L ⊥AOvàX0 ≡X Do đó,XL⊥AOnênX

thuộc trung trực củaAO

Xét phép nghịch đảoIqua đường tròn(O) GọiX∗, Y∗, Z∗ ảnh củaX, Y, Z qua phép nghịch đảoI DoX giao điểm trung trựcAOvớiBC nênX∗sẽ giao điểm đường trịn(A, AO)với đường trịn(OBC) Từ suy raAOalà trung trực củaOX∗ MàK ∈AOa

nênKO =KX∗ Chứng minh tương tự suy raKO =KX∗ =KY∗ = KZ∗ hayK tâm ngoại tiếp của4X∗Y∗Z∗ Theo tính chất phép nghịch đảo ta suy raOK ⊥X, Y, Z

(89)

Tạp chí Epsilon, Số 09, 06/2016

A

B

C

N K

D E F

H X

Y

Z

Hình 11.15: Bài toán 2

Chứng minh. Theo kết 1 N, K hai điểm đẳng giác 4ABC Do đó, ta có

AN ⊥ EF, BN ⊥ F D, CN ⊥ DE Suy N Y k F D ⊥ HE, N Z k DE ⊥ HF Xét

4KEF và4N Y Z cóN A⊥ EF, Y E ⊥KE, ZF ⊥ KF vàN A, Y E, ZF đồng quy tạiA Từ suy ra4KEF và4N Y Z trực giao Vậy đường thẳng quaK, E, F vng góc vớiY Z, N Y, N Zđồng quy MàEH⊥N Y vàF H ⊥N Z Từ ta đượcKH ⊥Y Z Ta giải xong hai toán nhỏ, áp dụng suy raH, K, O thẳng hàng đường thẳng vng góc vớiX, Y, Z.Định lý 6đã chứng minh xong

Nhận xét.Cả hai tốn nhỏ ngồi chứng minh cịn có cách tiếp cận chứng minh khác [2] [3] Cũng [2],Telv Cohlđã phát biểu chứng minh thêm tính chất liên quan đếnđịnh lý 6như sau:

Định lý HO = 3HK

Tác giả xin dẫn lại chứng minh củaTelv Cohlchođịnh lý 7trong [2] đây:

Chứng minh định lý 7. Gọi T trực tâm của4ABC.Ka, Halần lượt tâm ngoại tiếp

trực tâm của4AEF (Xác định điểmKb, Kc, Hb, Hctương tự) ĐặtJ ≡ AN ∩ (ABC)

(90)

A

B

C

K O

D E F

H

T

Ka

Ha

N J K

∗ a

OA

M Hb

Hc

P Q Qa

D∗

L

Hình 11.16: Định lý 7

và ∠J CB = ∠N AB = ∠CAK = ∠EF K, ta 4J BC ∼ 4− KEF, kết hợp với ∠COAB = ∠BOC = 2∠BAC = ∠F KaE = ∠EKa∗F (chú ý OAB = OAC

Ka∗E =Ka∗F)=⇒ 4J BC∪OA

∼ 4KEF ∪Ka∗, vậy∠EKKa∗ =∠OAJ B =∠ACB = ∠EKD =⇒ Ka∗ ∈ DK VìHa đối xứngK qua trung điểmEF, nênKKa∗HaKalà hình

bình hành, dó từKKa∗ ⊥BC ta suy raKaHa ⊥BC =⇒ KaHađi qua trung điểmM

KT (vìKalà trung điểm củaKA) Tương tự, ta chứng minh đượcKbHb ⊥CA, KcHc⊥AB

vàM ∈KbHb, M ∈KcHc

Vì Ha, Hb, Hc tương ứng đối xứng K qua trung điểm EF, F D, DE, nên 4DEF 4HaHbHcbằng (và vị tự), đo kết hợp vớiHaM kDK, HbM kEK, HcM kF K =⇒ 4DEF ∪K,và4HaHbHc∪M (và vị tự)

Gọi P, Qlà tâm đường tròn Euler tâm ngoại tiếp 4DEF Gọi Qa đối xứng

Q qua EF D∗ đối xứng D qua K Từ KD∗ =k DK =k M Ha =⇒ HaM KD∗

một hình bình hành, nênD∗ đối xứngM qua trung điểm củaEF, đóP trung điểm

DQa =⇒ 2P K

k

= QaD∗

k

= QM Mà M, Qlà trung điểm KT, KN, ta suy ra2M Q =k

T N =⇒4P K = 2k M Q=k T N, nếuL≡HP∩OT ta đượcP K =k N L=⇒3P K =k LO

(vìN O =N T)=⇒HO :HK =LO:P K = :

Tài liệu tham khảo

(91)

Tạp chí Epsilon, Số 09, 06/2016

http://faculty.evansville.edu/ck6/encyclopedia/ETC.html

[2] HK passes through the circumcenter

http://www.artofproblemsolving.com/community/c6h1125044

[3] Geometry Problem (23)

http://www.artofproblemsolving.com/community/c6h373509

[4] Euler Lines Reflected

http://www.artofproblemsolving.com/community/c6h610293

[5] Taylor Circle

http://mathworld.wolfram.com/TaylorCircle.html

[6] Honsberger, R , Episodes in Nineteenth and Twentieth Century Euclidean Geometry; Washington, DC: Math Assoc Amer., 1995

[7] The Taylor Circle

http://www.cut-the-knot.org/triangle/Taylor.shtml

[8] De Longchamps point

http://mathworld.wolfram.com/deLongchampsPoint.html

[9] Exeter point

(92)(93)

ĐỊNH LÝ SAWAYAMA VÀ THESBAULT TRONG CÁC BÀI TỐN HÌNH HỌC THI OLYMPIC

Trần Quang Hùng (Tổ toán, trường THPT Chuyên KHTN, Hà Nội) Dương Ánh Ngọc (Lớp 10 A1 Toán, trường THPT Chuyên KHTN, Hà Nội)

Bài viết tập trung vào việc phát triển ứng dụng định lý tiếng Sawayama Thébault tốn hình học thi Olympic

1 Mở đầu

Định lý Thébault định lý đẹp bậc hình học phẳng Nguyên liệu chủ yếu chứng minh định lý bổ đề Sawayama Vì chúng gộp chung thành tên gọi định lý Sawayama Thébault xem [1,2,3] Bổ đề Sawayama phát biểu sau, tham khảo [2,3]

Bài toán (Bổ đề Sawayama) Cho tam giácABC nội tiếp đường tròn (O).Dlà điểm thuộc đoạnBC Đường tròn(K)tiếp xúcDA, DC lần lượt tại M, N và tiếp xúc trong(O) Chứng minh rằngM N đi qua tâm nội tiếp tam giácABC.

A

B C

O

D

K M

N I

F

(94)

Chứng minh. Gọi phân giác góc∠BACcắt(O)tạiE khácA.AEcắtM N tạiI.(K)tiếp xúc

(O)tạiF Dễ cóE, N, F thẳng hàng vàEB2 =EC2 =EN.EF

Ta lại có ∠F M N = 12∠F KN = 12∠F OE = ∠F AE suy tứ giácAF IM nội tiếp Suy ∠IF N = ∠M F N −∠M F I = ∠DM N −∠M F I = ∠AF I −∠M F I = ∠AF M = ∠AIM =∠EIN Từ đó4EIN ∼ 4EF Isuy EI2 =EN.EF =EC2 =EB2 Suy raI

là tâm nội tiếp tam giácABC Ta có điều phải chứng minh

Định lý Thebault hệ trực tiếp bổ đề

Bài toán (Định lý Thébault) Cho tam giácABC nội tiếp đường tròn (O).Dlà điểm thuộc đoạnBC Đường tròn(K)tiếp xúcDA, DCvà tiếp xúc trong(O) Đường tròn(L)tiếp xúcDA, DBvà tiếp xúc trong(O) Chứng minh rằngKLđi qua tâm nội tiếp tam giácABC.

Chứng minh xem chi tiết [1,4] Sau phát biểu khác bổ đề Sawayama tứ giác nội tiếp

Bài toán Cho tứ giácABCDnội tiếp đường tròn(O) Hai đường chéoACBDcắt nhau tạiE Đường tròn(K)tiếp xúc với đoạnEC, EDtạiM, N và tiếp xúc trong(O) Chứng minh rằngM N đi qua tâm nội tiếp tam giácACDvà tam giácBCD.

Bài toán (Định lý Sawayama Thébault mở rộng với tâm bàng tiếp) Cho tam giácABC

nội tiếp đường tròn(O).Dlà điểm thuộc tia đối tiaCB Đường tròn(K)tiếp xúcDA, DC

lần lượt tạiM, N và tiếp xúc ngoài(O) Chứng minh rằngM N đi qua tâm bàng tiếp ứng với đỉnhB của tam giácABC.

Chứng minh. Gọi phân giác đỉnhAcắt(O)tạiE khácA.AE cắtM N tạiJ.(K)tiếp xúc (O) tạiF Dễ cóE, N, F thẳng hàng vàEB2 = EC2 = EN.EF Ta lại có ∠F M N =

1

2∠F KN =

2∠F OE =∠F BE = ∠F AJ suy tứ giácAF M J nội tiếp Suy ra∠EF J = 180◦ −∠N F J = 180◦−∠N F M −∠M F J = 180◦−∠J M A−∠M AJ =∠M J A Từ

4EF J ∼ 4EJ N suy raEJ2 =EN.EF =EC2 =EB2 Suy raJlà tâm bàng tiếp ứng với

đỉnhB tam giácABC Ta có điều phải chứng minh

Cả hai phát biểu hay dùng tốn hình học khác Trên vài toán điểm qua định lý Sawayama Thébault số mở rộng

2 Một số toán ứng dụng

Định lý Sawayama Thébault coi bổ đề thơng dụng tốn Olympic khó, đơi việc dùng thơng dụng hiển nhiên tới mức khó nhận vai trị Chúng ta bắt đầu toán chọn đội tuyển Việt Nam 2014, tham khảo [4]

Bài toán (VNTST 2014) Cho tam giácABC nội tiếp đường trịn(O).Trên cungBCd

khơng chứaAcủa (O)lấy điểmD Giả sửCD cắtAB E BDcắtAC F Gọi(K) đường tròn nằm tam giácEBD, tiếp xúc vớiEB, EDvà tiếp xúc với đường tròn(O).Gọi (L)là đường tròn nằm tam giácF CD, tiếp xúc vớiF C, F Dvà tiếp xúc với đường tròn (O) GọiM là tiếp điểm của(K)vớiBE N là tiếp điểm của(L)vớiCF Chứng minh rằng

(95)

Tạp chí Epsilon, Số 09, 06/2016

B

C A

O

D

K

J N

F

E

M

Chứng minh. Gọi(K)tiếp xúcEDtạiGvà(L)tiếp xúcF D tạiH Theo Định lý Sawayama Thébault mở rộng cho tâm bàng tiếp thìM G, N H qua tâm bàng tiếpJ ứng với đỉnh A Từ theo tính chất góc phân giác góc tạo bởi(EK, LF) = 12(∠A+∠D) = 90◦

EK ⊥ LF Lại dễ cóM G ⊥EK ⊥ LF ⊥ N H từ đóM Gvng gócN H tạiJ nên đường trịn đường kínhM N quaJ cố định

Bài tốn mở rộng thú vị sau, tham khảo [4]

Bài tốn Cho tam giácABC Một đường trịn(O)bất kỳ cố định quaB, C.Dlà điểm di chuyển trên(O)sao choA, Dkhác phíaBC Giả sửCDcắtABE BDcắtACF Gọi (K)là đường trịn tiếp xúcEB, EDlần lượt tạiM, N và tiếp xúc trong(O) Gọi(L)là đường tròn tiếp xúcF C, F Dlần lượt tạiP, Qvà tiếp xúc trong(O) Chứng minh giao điểm

M N, P Qln nằm đường trịn cố định khiDdi chuyển.

Chứng minh. GọiAB, AC cắt(O)tạiG, H khácB, C Áp dụng định lý Sawayama Thébault mở rộng vào tam giácGBC, HBC thìM N qua tâm bàng tiếpS ứng với đỉnhG

của tam giácBGC cố định vàP Qđi qua tâm bàng tiếpT ứng với đỉnhHcủa tam giácHBC

(96)

A

B C

O

D

E

K

F L

J M

N

G

H

Các bạn làm hai toán sau để luyện tập thêm dạng

Bài tốn Cho tam giácABCnội tiếp đường trịn(O).P là điểm di chuyển cungBCdchứa Acủa(O).P B, P C cắtCA, ABlần lượt tạiE, F Đường tròn(K)tiếp xúc đoạnEA, EB tiếp xúc trong(O) Đường tròn(L)tiếp xúc đoạnF B, F C và tiếp xúcBCdkhông chứaAcủa

(O).(K)tiếp xúcAC tạiM (L)tiếp xúcABtại N Chứng minh đường trịn đường kínhM N ln điểm cố định khiP di chuyển.

Bài toán Cho tam giácABC và đường tròn(O)cố định quaB, C.Dlà điểm di chuyển trên cungBCd của(O)sao choD, Acùng phíaBC.DB, DC cắt CA, AB lần lượt tạiE, F.

Đường tròn(K)tiếp xúc đoạnEA, EB tạiM, N và tiếp xúc trong (O) Đường tròn(L)tiếp xúc đoạnF B, F C tạiP, Qvà tiếp xúc(O)tại điểm khơng phíaAso vớiBC Chứng minh giao điểm củaM N P Qluôn thuộc đường trịn cố định khiP di chuyển.

Bài tốn tiếp sau tham khảo [1,5]

Bài toán Cho tứ giácABCDnội tiếp đường trịn(O)ACcắtBDtạiE Đường trịn(K) tiếp xúc đoạnEA, EDvà tiếp xúc trong(O) Đường tròn(L)tiếp xúc đoạnEB, EC và tiếp xúc trong(O) Chứng minh trục đẳng phương của(K)(L)chia đôi cungABd

d

(97)

Tạp chí Epsilon, Số 09, 06/2016

A

B C

O

D

E

K

F L

M

N Q

P G

S H

T R

Chứng minh. Gọi(K)tiếp xúcAC, BDtạiM, N.(L)tiếp xúcAC, BDtạiP, Q Theo định lý Sawayama Thébault thìM N qua tâm nội tiếpI tam giácDABvàP Qđi qua tâm nội tiếpJ tam giácCAB GọiS, T trung điểm cungABdvàCDd của(O) Ta dễ chứng

minhST song song với phân giác∠AEBvà vng góc với phân giác∠BEC Cũng dễ chứng minh tứ giácAIJ Bnội tiếp nênIJcắtEA, EB tạo thành tam giác cân nênIJ ⊥ST Hơn nữaDI, CJ quaS vàSI = SA = SB = SJ Từ tam giácSIJ cân vàST ⊥ IJ nên

ST chia đơiIJ Trong hình thangM N P QcóST kM N kP QvàST chia đơiIJ nênST đường trung bình nênST qua trung điểm củaM P, N Q, vậyST trục đẳng phương của(K)và(L) Đó điều phải chứng minh

Bài toán tiếp sau hệ

Bài toán 10 Cho tứ giácABCDnội tiếp đường trịn(O)AC cắtBDtại E Đường trịn (K)tiếp xúc đoạnEA, EDvà tiếp xúc trong(O) Đường tròn(L)tiếp xúc đoạnEB, EC tiếp xúc trong(O) Chứng minh có tiếp tuyến chung của(K)(L)song song

AB.

Chứng minh. Gọi tiếp tuyến chung của(K)và(L)gầnABhơn cắt(O)tạiM, N.(K),(L)

(98)

O D

A

B

C E

K

L J I

S

M

N

Q

P

T

O

D A

B

C E

K

L M

N

G P

S

Q H

Thébault thìP Q, QH qua trung điểm Scủa cungM Nd Lại cóSG.SP =SM2 =SN2 = SH.SQ Từ S thuộc trục đẳng phương (K) (L) Theo trước S trung điểm cungABd Vậy cungM N ,d dABcó chung trung điểmS nênM N kAB Ta có điều phải chứng

minh

(99)

Tạp chí Epsilon, Số 09, 06/2016

Bài toán 11 Cho tam giác ABC nội tiếp đường tròn (O) D là điểm thuộc đoạn BC. Đường tròn(K)tiếp xúcDA, DC và tiếp xúc trong(O) Đường tròn(L)tiếp xúcDA, DB tiếp xúc trong(O) Tiếp tuyến chung khácADcủa(K)(L)cắt cungBCd không chứa Acủa(O)tạiE Chứng minh rằng∠DAB =∠EAC.

Chứng minh. GọiADcắt(O)tạiF khácA Tiếp tuyến chung khácADcủa(K)và(L)

cắt(O)tạiGkhácE Áp dụng trước vào tứ giác AEF G, ta thấyBC tiếp tuyến chung của(K)và(L)gầnEF nênBC kEF Từ dễ thấy∠DAB=∠EAC Ta có điều phải chứng minh

A

B C

O

D L

K

F E

G

Chúng ta thu hệ đơn giản toán sau

Bài toán 12 Cho tam giácABCnội tiếp đường tròn(O)với đường caoAH Đường tròn(K) tiếp xúcHA, HC và tiếp xúc trong(O) Đường tròn(L)tiếp xúcHA, HB và tiếp xúc trong (O) Chứng minh tiếp tuyến chung khácAHcủa(K)(L)đi qua điểm đối tâm của

Atrên(O)

Bài tốn sau có phần đề chọn đội Romani năm 2006 bổ sung thêm điều kiện tham khảo [6,7,8], lời giải sau dựa vào ý tưởng Telv Cohl [6]

Bài toán 13 Cho tam giácABCnội tiếp đường tròn(O)với đường caoAH Đường tròn(K) tiếp xúcHA, HC và tiếp xúc trong(O) Đường tròn(L)tiếp xúcHA, HB và tiếp xúc trong (O) Chứng minh điều kiện sau tương đương

1)AB+AC = 2BC.

(100)

A

B C

O

H L

K I

Q

P M

N

R

E D

Chứng minh. Nhờ toán ta thấy điều kiện 2) 3) tương đương Ta chứng minh 1) 3) tương đương xong, Gọi(K)tiếp xúcHC, HAtạiP, Q Gọi(L)tiếp xúc

HB, HAtạiM, N Theo định lý Sawayama Thébault thìM N, P Q, KLđồng quy tâm nội tiếpI tam giácABC Dễ thấy tam giácHM N vàHP Qvuông cân nênM Ilà phân giác

∠LM H vàP Ilà phân giác∠KP H Từ dễ có đoạn thẳng nhauIL =IH =IK

suy raIlà trung điểmKL GọiDlà hình chiếu củaI lênBCthìDlà trung điểmM P Gọi tiếp tuyến chung khácAH của(K)và(L)cắtBC tạiR Theo tính chất tiếp tuyến cắt hai đường trịn ngồi dễ cóHM =RP nênDlà trung điểmHR GọiAI cắt(O)tại

E khácA Ta thấyRlà trung điểmBCkhi khiER ⊥BCkhi khiIlà trung điểm

AE Theo hệ định lý Ptolemy điều tương đươngAB+AC = 2BC Ta có điều phải chứng minh

Nhận xét 2.1 Với ý tưởng tương tự lời giải bạn đọc đề xuất số vấn đề tổng quát hơn.

Bài toán tiếp tục sau xuất kỳ thi Olympic chuyên KHTN năm 2015 tham khảo [9]

Bài toán 14 Cho tứ giácABCDnội tiếp đường tròn(O) GọiI, J là tâm nội tiếp tam giác

BAD, CAD Gọi DI, AJ lần lượt cắt(O)tại S, T khácD, A Đường thẳngIJ lần lượt cắt

AB, CDtạiM, N.

a) Chứng minh rằngSM T N cắt đường tròn(O)

b) Gọi đường tròn ngoại tiếp tam giácABN cắtCD tạiP khácN Đường tròn ngoại tiếp tam giácCDM cắtABtạiQkhácM Chứng minh rằngP Qđi qua tâm nội tiếp hai tam giácABC

DBC.

(101)

Tạp chí Epsilon, Số 09, 06/2016

O

D A

B

C G

J

I L

K

N P

S

M

T Q

E

F U

V

∠N DJ =∠J DA=∠AIM Từ hai tam giácM AIvàN J Dđồng dạng Từ suy raSM A

vàT N J đồng dạng Vậy∠ASM =∠N T J đóSM vàT N cắt Etrên đường tròn

(O)

b) GọiABcắtCDtạiG.GEcắt(O)tạiF khácE Ta thấyGC.GD =GE.GF =GM.GQ Từ tứ giácM QF Enội tiếp nên∠QF E =∠AM E =∠M AS+∠M SA=∠M BS+∠AF E = ∠SF A+∠ASE =∠EF S Từ đóS, Q, F thẳng hàng Tương tựT, P, F thẳng hàng Từ chứng minh SM A T N J đồng dạng nên tam giác GM N cân suy GM = GN Lại có

GM.GQ=GN.GP nênGP =GQsuy raP QkM N kST Từ đường trịn ngoại tiếp tam giácF P Qtiếp xúc(O) Vậy theo định lý Poncelet nếuP QcắtDB, AC tạiU, V đường trịn ngoại tiếp tam giácF U V tiếp xúc(O)và tiếp xúcDB, AC Từ theo định lý Sawayama Thebaut thìP Qđi qua tâm nội tiếp hai tam giácABCvàDBC

Nhận xét.Đây toán sử dụng hai bổ đề quan trọng định lý Sawayama Thebaut định lý Poncelet Chú ý với định lý Sawayama Thebaut hiểu cách chặt chẽ phải phát biểu đường chéo tứ giác nội tiếp, việc sử dụng định lý Poncelet để đưa đường trònF U V tiếp xúc vớiCA, BDlà cần thiết Chúng xin nhắc lại định lý Poncelet

Bài toán 15 (Định lý Poncelet) Cho tứ giácABCDnội tiếp đường tròn(O) Đường tròn(K) tiếp xúcAB, CDtại M, N Đường tròn(L)tiếp xúc vớiAC, BDtạiP, Q Chứng minh rằng nếuM, N, P, Qthẳng hàng thì(K),(L)(O)đồng trục.

Chứng minh chi tiết mở rộng định lý xem [10] Bài tốn gốc phát biểu gọn lại ý sau

Bài tốn 16 Cho tứ giácABCDnội tiếp đường trịn(O) GọiI, J là tâm nội tiếp tam giác

BAD, CAD Đường thẳngIJ lần lượt cắtAB, CDtạiM, N Gọi đường tròn ngoại tiếp tam giácABN cắtCD tạiP khácN Đường tròn ngoại tiếp tam giácCDM cắt ABtạiQkhác

M Chứng minh rằngP Qđi qua tâm nội tiếp hai tam giácABC DBC.

(102)

Bài toán 17 Cho tứ giácABCDnội tiếp đường tròn(O) GọiI, J là tâm nội tiếp tam giác

BAD, CAD Đường thẳngIJ lần lượt cắtAC, BDtạiM, N Gọi đường tròn ngoại tiếp tam giácACN cắtBDtạiP khácN Đường tròn ngoại tiếp tam giácBDM cắtAC tạiQkhácM. Chứng minh rằngP Qđi qua tâm bàng tiếp góc∠A∠Dcủa hai tam giácABC DBC.

Bài tốn có mở rộng khác đơn giản thú vị sau

Bài toán 18 Cho tứ giácABCDnội tiếp đường tròn(O) GọiS, T là trung điểm cung nhỏ

d

AB,CDd.M, N lần lượt thuộcAB, CD Gọi đường tròn ngoại tiếp tam giácABN cắtCDtại P khácN Đường tròn ngoại tiếp tam giácCDM cắtABtạiQkhácM Chứng minh rằngSM

T N cắt trên(O)khi khiSQT P cắt trên(O)

Chúng ta chuyển sang vấn đề tiếp theo, toán quan trọng liên quan tới nhiều toán khác, tham khảo [21,22,23]

Bài toán 19 Cho tứ giácABCDnội tiếp đường tròn(O) Hai đường chéoAC BDcắt nhau tạiE Đường tròn(K)tiếp xúc với đoạnEC, EDvà tiếp xúc trong(O)tạiP Chứng minh rằng phân giác∠CP Dđi qua tâm nội tiếp tam giácECD và phân giác∠AP B đi qua tâm nội tiếp tam giácEAB.

A

D C

O B

E

K

P I

J L

N

M T

Chứng minh. Gọi(K)tiếp xúc đoạnEC, EDtạiM, N GọiJ, Llà tâm nội tiếp tam giácACD

vàBCD Theo định lý Sawayama Thebaut thìM N quaJ, L Hơn tứ giácP CM L

và P DN J nội tiếp Gọi đường tròn ngoại tiếp hai tứ giác cắt tạiI khácP Ta có

(103)

Tạp chí Epsilon, Số 09, 06/2016

nênP Ilà phân giác∠CP D Xét tam giácEABcó đường trịn(O)quaA, B và(K)tiếp xúc

EA, EB tiếp xúc trong(O)tạiP Theo phần vừa chứng minh phân giác∠AP Bđi qua tâm nội tiếp tam giácEAB

Nhận xét 2.2 Bài tốn có nhiều biến thể khác ví dụ sau

Bài tốn 20 Cho tam giácABC một đường tròn(K)quaB, C và cắt cạnhCA, AB Đường trịn(L)tiếp xúc đoạnAB, AC và tiếp xúc ngồi(K)tạiP Chứng minh phân giác∠BP C

đi qua tâm nội tiếp tam giácABC.

Cách chứng minh biến thể bạn áp dụng cách hồn tồn tương tự cách chứng minh toán với mở rộng định lý Sawayama Thebaut cho tâm bàng tiếp tiếp xúc ngồi Bài tốn dẫn tới toán ứng dụng đẹp định lý Sawayama Thebaut đề nghị bởi, Vladimir Zajic tham khảo [11]

Bài toán 21 Cho tam giácABCnội tiếp đường tròn(O).P là điểm cạnhBC,AP cắt (O)tạiQkhácA Gọi(O1)là đường tròn tiếp xúc với(O)và tiếp xúc với hai cạnhP A,

P B.(O2)là đường tròn tiếp xúc với(O)và tiếp xúcP C,P A GọiI1,I2 là tâm đường tròn nội tiếp tam giácP BQ,P CQ Chứng minh rằngO1O2,I1I2,BCđồng quy.

A

B

C O

P

O1

O2

Q

I1 I

2

T1

T2

S

X1

X2

Chứng minh. Gọi(O1),(O2)tiếp xúc(O)tạiT1, T2 Theo chứng minh trước phân giác ∠CT1Qđi qua tâmI2 tâm nội tiếp tam giácP QC quaX2là điểm cungQCd

Tương tự phân giác∠BT2Qđi qua tâmI1 tâm nội tiếp tam giácP QBvà quaX1là điểm

chính cungQBd Từ áp dụng Pascal cho sáu điểm

T1B X1

C T2 X2

thu đượcBCcắtT1T2

tạiS thuộcI1I2 Mặt khác dễ thấySthuộcO1O2, nên ta có điều phải chứng minh

Nhận xét.Sử dụng hàng điều hòa, ta dễ chứng minh đượcO1I1, O2I2 vàAP đồng quy

(104)

Bài toán 22 Cho tam giác ABC nội tiếp đường tròn (O) với P, Q là hai điểm đoạn

BC.AP, AQcắt(O)tạiM, N khácA Gọi(I1),(I2),(I3),(I4)là đường tròn nội tiếp tam giác P AB, QAC, QAB, P AC Gọi (J1),(J2),(J3),(J4) là đường tròn tiếp xúc với đoạnP B, P M;QC, QN;QB, QN;P C, P M và tiếp xúc trong(O) Chứng minh

I1I2, I3I4, J1J2, J3J4 BCđồng quy.

Lời giải chi tiết bạn tham khảo [12] Bài tốn sau tham khảo [28]

Bài toán 23 Cho tam giácABC nhọn nội tiếp đường tròn(O) Đường tròn(I)tiếp xúc với cung nhỏABvà tiếp xúc với đoạnABtạiQ Đường tròn(J)tiếp xúc với cung nhỏAC và tiếp xúc với đoạnAC tạiN Tiếp tuyến chung ngồi khơng cắt đoạnAQ, AN của(I)(J)tiếp xúc(I),(J)lần lượt tạiM, P Chứng minh rằngM N, P Qcắt phân giác∠BAC khi và khiM P kBC.

A

B C

O F

E G

I

H J

P Q

N

M R

K

Chứng minh. GọiM P cắt(O)tạiG, H vàAK đường cao tam giácAGH GọiM N cắt

P QtạiR Theo định lý định lý Sawayama Thébault thìM N, P Qcùng qua tâm nội tiếp tam giácAGH nênRlà tâm nội tiếp tam giácAGH Từ đóARlà phân giác∠GAHhay phân giác∠OAK doAO, AK đẳng giác trong∠GAH.Từ đóARlà phân giác∠BAC

khi khiAO, AK đẳng giác trong∠BAC hayAK đường cao tam giácAB Từ

Rnằm phân giác∠BAC khiM P kBC

Qua số ví dụ bạn phần thấy ứng dụng quan trọng định lý Sawayama Thebaut toán thi Olympic

3 Một số tốn liên quan

Phần tơi xin đề nghị sưu tập lại số toán hay liên quan đến nội dung có dẫn link tham khảo, bạn tự vẽ hình minh họa

(105)

Tạp chí Epsilon, Số 09, 06/2016

Bài tốn 24 Cho tam giácABC nội tiếp(O)P di chuyển cạnhBC Đường tròn(K) tiếp xúcP C tạiM tiếp xúcP Avà tiếp xúc trong(O) Đường tròn(L)tiếp xúcP B tạiN tiếp xúcP Avà tiếp xúc trong(O).AP cắt(O)tạiQkhácA Chứng minh đường trịn ngoại tiếp tam giácQM N ln qua điểm cố định.

Lời giải sau sử dụng ý tưởng Lym [13]

O

B C

P A

I

N M

K

L

Q Y

X

E F

D

J J'

D'

A'

S

T

Lời giải thứ nhất. GọiX, Y tiếp điểm của(K)và(L)với(O).E, F tiếp điểm của(K)và

(L)vớiAP.(I)nội tiếp tam giácABC tiếp xúcBCtạiD Gọi đường tròn A-Mixtilinear tam giácABC tiếp xúc(O)tạiJ,D0 tiếp điểm đường tròn bàng tiếp∠AvớiBC,J0 giao điểm củaAD0 với(O) Theo tính chất quen thuộc đường trịn Mixtilinear thìJ J0 kBC, theo tính chất đối xứng thìJ Dđi quaA0 đối xứngAqua trung trựcBC, nênP DJ Qnội tiếp Mặt khác vìY N, XM qua điểm cungBCdkhông chứaAnênY N M X

nội tiếp.HạP S ⊥LKtạiS, áp dụng định lí Sawayama Thébault ta cóK, L, Ithẳng hàng

∠N IM =∠N SM = 90◦ Theo tính chất hai đường trịn tiếp xúc đường trịn thứ ba, ta cóBC, XY, KLđồng quy tạiT, đóT P.T D=T S.T I =T N.T M =T Y.T X =T B.T C

suy raP Y XDnội tiếp Kết hợp vớiXY QJ, P DJ Qnội tiếp, ta chứng minh đượcQJđi quaT, thu đượcT Q.T J =T Y.T X =T N.T M suy ra(QM N)đi quaJ cố định

(106)

Bổ đề 14 GọiK là điểm đường trịn ngoại tiếp tam giácABC,X bất kì thuộc đường thẳngAK TừXkẻ hai tiếp tuyến tới đường tròn nội tiếp(I)của tam giácABC, cắtBC

lần lượt tạiY, Z Khi đó(KY Z)đi qua tiếp điểmJ của đường tròn A-Mixtilinear tam giác

ABC với(O)

Lời giải sau dựa ý tưởng Jean-Louis tham khảo [27]

B

C A

I

J E

Ia

K

X Z

Y T

F

D

S L

Chứng minh. Giả sửK nằm cungBCdkhông chứaAcủa(O) GọiIalà tâm bàng tiếp∠A

của tam giácABC,Elà điểm giữaBCdkhơng chứaAcủa(O),T giao điểm củaAX

vàBC.IaT giao(BIC)tạiF,IX giao IaF tạiD TừT K.T A= T B.T C =T F.IaT suy

A, F, K, Iacùng thuộc đường tròn Kết hợp với tính chất quen thuộc đường trịn

Mixtilinear∠IJ E = 90◦, ta có∠IJ K +∠IF K = ∠IJ E−∠EJ K +∠IF Ia+∠KF Ia = 90◦−∠EAK+ 90◦+∠KAIa = 180◦nênJ KF I nội tiếp MàIF CB, BCKJ nội tiếp, ta thu

(107)

Tạp chí Epsilon, Số 09, 06/2016

tại T, A, X, T thẳng hàng nênLI, BIa, Y Dđồng quy tâm bàng tiếp ∠L tam giác

LBY Do đó∠IY D = 90◦ Chứng minh tương tự ta có∠IZD = 90◦, dễ thấyI, F, Y, Z, D

thuộc đường trịn Theo tính chất phương tích thìSY.SZ =SI.SF =SK.SJ,

Z, Y, K, J thuộc đường tròn

O

B C

P A

I

N M

K

L

Q E

F

J

D Z

Y

Lời giải thứ hai. GọiDlà tiếp điểm đường tròn nội tiếp(I)vớiBC Kẻ tiếp tuyếnN Z,

M Y tới(I) Vì tam giácP N F cân nên dễ thấy∠ZIN +∠N F P =∠N ID+∠P N F = 90◦, suy raN Zsong song vớiAP Tương tự ta cóM Y kAP kN Z coi giao điểm

M Y vàN Z nằm trênAP Áp dụng bổ đề thì(QM N)đi qua điểm cố định tiếp điểm đường trịn A-Mixtilinear tam giácABCvới(O)

Bài tốn sau tham khảo [14]

Bài toán 25 Cho tam giácABCnội tiếp đường tròn(O).P là điểm thuộc đoạnBC Đường tròn(K)tiếp xúcP A, P C và tiếp xúc trong(O)tạiM Đường tròn(L)tiếp xúcP A, P B tiếp xúc trong(O)tạiN.

a) Chứng minh rằngM N luôn qua điểm cố định khiP thay đổi.

b) Chứng minh đường trịn ngoại tiếp tam giácP M N ln qua điểm cố định khiP

(108)

O

B P C

A

I

K

L

M

N S

J

D

Chứng minh. a) Áp dụng định lí Sawayama Thébault ta cóKL qua tâm nội tiếpI tam giácABC Theo tính chất hai đường trịn tiếp xúc với đường tròn thứ ba, dễ thấy

KL, BC, M N đồng quy tạiS GọiOI cắtM N tạiJ Áp dụng định lí Menelaus cho tam giác

IOLcát tuyếnN J S ta có J I J O =

SI SL

N L N O =

R(I)

R(L)

R(L)

R(O) =

R(I)

R(O) VậyM N quaJ cố định

vớiJ đẳng giác điểm Nagel tâm vị tự ngồi của(O)và(I)

b) GọiDlà hình chiếu củaI lênBC, từ kết cách chứng minh thứ củaBài tốn 23ta có

P DN M nội tiếp, suy ra(P N M)đi quaDcố định

Bài toán sau tham khảo [15,16]

Bài toán 26 (Bulgaria 2010) Cho tam giácABCnội tiếp đường tròn(O).P là điểm thuộc đoạnBC Đường tròn(K)tiếp xúcP A, P C và tiếp xúc trong(O) Đường tròn(L)tiếp xúc

P A, P Bvà tiếp xúc trong(O) Chứng minh bán kính của(K)(L)bằng chỉ khiP là tiếp điểm đường tròn bàng tiếp gócAvớiBC.

Chứng minh. GọiI tâm nội tiếp tam giácABC,AP giao(O)tạiD,S điểm cungBCd khơng chứaAcủa(O) HạIQ ⊥ BC tạiQ.(K),(L)tiếp xúcBC tạiR, T tiếp

xúcO tạiM, N, dễ thấyS, T, N thẳng hàng,S, R, M thẳng hàng Tiếp tuyến chung khác

AP của(K)và(L)cắt(O)tạiE, F cho E, Dcùng phía với BC Theo kết quảBài toán 11dễ thấyAE, AP hai đường đẳng giác Ta cóP tiếp điểm đường trịn bàng tiếp góc

AvớiBC khiE tiếp điểm đường tròn A-Mixtilinear tam giácABC với(O)

hay tương đương vớiT RDE nội tiếp VìDE kBC nên điều tương đương vớiT đối xứng

Rqua trung điểmBC Chú ý đẳng thứcST.SN =SR.SM nên giả thiết tương đương với

ST SN =

SR SM hay

OL ON =

OK

OM Điều xảy khi(K)và(L)có bán kính

(109)

Tạp chí Epsilon, Số 09, 06/2016

O A

B C

S I

E

D P Q

T R

K L

M N

F

Bài toán 27 Cho tam giácABC nội tiếp đường trịn(O) Hai đường trịn ln trực giao qua

AI cắt(O)tạiM, N khácA Chứng minh đường thẳngM N luôn qua điểm cố định khi hai đường trịn thay đổi.

Bổ đề 15 Cho tam giácABC với đường tròn ngoại tiếp(O), tâm nội tiếpI.P là điểm thuộc đoạnBC Đường tròn(K)tiếp xúcP A, P C và tiếp xúc trong(O)tạiM Đường tròn(L) tiếp xúcP A, P Bvà tiếp xúc trong(O)tạiN Khi đó(AIN)trực giao với(AIM)

Chứng minh bổ đề. Gọi(K),(L)tiếp xúcBCtạiX, Y Kẻ tiếp tuyến chung khácAP

của (K) (L) cắt (O) BC A0, Q cho A, A0 phía BC Kẻ

P S ⊥ KL, theo cách chứng minh thứ củaBài tốn 24ta cóLK, N M vàBC đồng quy tạiT thìT S.T I =T N.T M =T B.T C nên đường tròn ngoại tiếp tam giácBIC quaS Mặt khác gọiS0là tâm nội tiếp tam giácA0BC thìS0 thuộc(BIC)và theo định lí Thébault,S0 thuộcKL, suy raS0trùngS VìP S ⊥KLnênIQ⊥KL GọiJlà giao điểm hai tiếp tuyến

(O)tạiM, N Dễ thấyJ thuộc trục đẳng phương của(K)và(L), kết hợp với kiệnIQkSP

vàY P = QX suy đường thẳng quaJ vng gócKLchia đơi đoạn thẳngXY, QP

IS MàT S.T I =T B.T C =T N.T M suy I, S, M, N thuộc đường trịn, nênJ tâm

(N SIM) Từ đó∠AM I+∠AN I =∠M IN−∠M AN = 180◦−1

2∠M J N−∠J M N = 90

, suy ra(AN I)trực giao với(AM I)

Chứng minh. Áp dụng bổ đề dễ thấy tồn tại(K)và(L)cùng đồng thời tiếp xúcBC

và(O)sao cho tiếp tuyến chung chúng quaA Áp dụng kết toán 25 ta cóM N ln qua tâm vị tự ngồi của(I)và(O)cố định

(110)

O

B P C

A

I

Y X

K L

M N

Q S

T

A'

J

Bài toán 28 Cho lục giác lưỡng tiếpABCDEF với tâm nội tiếpI và tâm ngoại tiếp O khi đó đường chép AD, BE, CF đồng quy tại G GọiJ, K, L, M, N, P là tâm nội tiếp các tam giácGAB, GBC, GCD, GDE, GEF, GF A Đường tròn tiếp xúc đoạn GA, GB và tiếp xúc trong(O)tạiX Tương tự có điểmY, Z, T, U, V Chứng minh bẩy đường thẳng

XM, Y N, ZP, T J, U K, V LOI đồng quy.

Bài toán sau tham khảo [19]

Bài tốn 29 Cho tam giác ABCcó tâm ngoại tiếpO và tâm nội tiếpI Đường tròn(K)tiếp xúcAB, ACtạiE, F và tiếp xúc ngồi đường trịn ngoại tiếp tam giácOBC Chứng minh rằng

EF chia đôiAI.

Chứng minh. Dễ thấy(K)được xác định nên, giả sử EF chia đôiAI Ta chứng minh(K)tiếp xúc(BOC) GọiDlà giao điểm thứ hai của(BOC)vớiAB,S điểm cungdACkhơng chứaBcủa(O).SBcắt(BOC)tạiQthì∠BQC =∠BOC = 2∠BSC

nên QS = QC = QD, suy S tâm bàng tiếp ∠B tam giác BDC Dễ thấyEF quaS Do theo dạng đảo bổ đề Sawayama mở rộng cho tâm bàng tiếp thì(K)tiếp xúc

(BOC)

(111)

Tạp chí Epsilon, Số 09, 06/2016

O

B

C A

I

K L

M

N J

O

A

B

C I

E

K F

D S

Q

Bài tốn 30 Cho đường trịn (O1)tiếp xúc ngồi(O2)tạiT.P Qlà dây cung của (O1)

P T, QT cắt(O2)tại R, S Tiếp tuyến quaP của (O1)cắt(O2)tại A, B sao choAnằm giữa

P, B Tiếp tuyến quaQcủa(O1)cắt(O2)tạiD, C sao choDnằm giữaQ, C.SAgiaoP Qtại

(112)

O1 T O2

P

Q

R S

A

B D

C

F E

Chứng minh. Dễ thấyS điểm cungCDd khơng chứa B của(O2) Mà theo bổ đề

Sawayama mở rộng cho tâm bàng tiếp thìP Qđi qua tâm bàng tiếp∠Ccủa tam giácCAD, suy raF tâm bàng tiếp tam giácCAD Tương tự ta cóElà tâm bàng tiếp∠B tam giácBAD Từ đó∠EAF =∠DAE−∠DAF = 90◦−

2∠DAB−90

◦+

2∠DAC =

2∠BAC

Bài toán sau tham khảo [24]

Bài toán 31 Cho tam giácABC với trực tâmH, đường caoAD, BE.K thuộcDE sao cho

DK =DH Đường thẳng quaK vng gócDE cắtADtạiI.M là trung điểmBC Chứng minh rằngBM =M I+IK.

Lời giải Luis González. Kẻ đường cao CF tam giác ABC, ta có H tâm nội tiếp tam giác DEF Đường trịn tâmI bán kínhIK tiếp xúc DF L, dễ thấyLK k BC Gọi

J giao điểm củaKLvàF C, ta có∠F J L =∠F CB =∠F EB =∠HEK suy J HEK

nội tiếp Mặt khác gọi P giao điểm F D với đường trịn đường kính BC, ý

F EM D nội tiếp tam giác DHK cân, ta có ∠F J E = ∠HKE = 90◦ + 12∠HDK = 90◦+14∠F M E = 90◦+ 12∠F P E, vậyJ tâm nội tiếp tam giácP F E Áp dụng dạng đảo bổ đề Sawayama cho tam giácP F E, vìLK qua J nên (I)tiếp xúc(M)tạiT Do

M B =M T =M I+IT =M I+IK

(113)

Tạp chí Epsilon, Số 09, 06/2016

A

B C

H

D

E

F

K I

M

L J

P T

Bài toán 32 Cho tam giácABC nội tiếp đường tròn(O).Dlà điểm đoạnBC Đường tròn (K)tiếp xúcDA, DC và tiếp xúc trong(O) Đường tròn(L)tiếp xúcDA, DBvà tiếp xúc trong (O) Tiếp tuyến chung của(K)(L)khácADcắtBCtạiS Tiếp tuyến chung của (K)(L)khácBCcắtADtạiT Chứng minh rằngST đi qua tâm nội tiếp tam giácABC.

Gợi ý giải. GọiI tâm nội tiếp tam giácABC, áp dụng định lí Thébault ta cóL, K, I thẳng hàng Theo cách chứng minh Bổ đề 27.1 SI ⊥ KL, mà ST ⊥ KL S, I, T thẳng hàng

Lời kết cảm ơn

(114)

B

C D

A

I

K

L

S T

Tài liệu tham khảo

[1] Nguyễn Thị Hường, Lương Ánh Nguyệt, Lương Thị Thanh Mai, Đào Thị Quỳnh Nga Định lý Sawayama Thébault

http://analgeomatica.blogspot.com/2014/02/inh-ly-sawayama-va-thebault.html

[2] Forum Geometricorum, Sawayama and Thebault’s theorem, Jean-Louis Ayme http://forumgeom.fau.edu/FG2003volume3/FG200325.pdf

[3] Sawayama Thebault’s Theorem

http://www.cut-the-knot.org/triangle/SawayamaTheBault.pdf

[4] Xung quanh tốn hình học kỳ thi chọn đội tuyển Việt Nam năm 2014 http://analgeomatica.blogspot.com/2014/03/xung-quanh-mot-bai-toan-hinh-hoc-trong.html

[5] Parallel tangent

http://artofproblemsolving.com/community/c6h15945

[6] Metric relation describing the position of the incenter http://artofproblemsolving.com/community/c6h88823

[7] circles and circles

http://artofproblemsolving.com/community/c6h118385

[8] b+c=2a

http://artofproblemsolving.com/community/c6h210518

[9] Hai hình học thi Olympic chuyên KHTN 2015

(115)

Tạp chí Epsilon, Số 09, 06/2016

[10] Topic circles with common tangency point

http://www.artofproblemsolving.com/community/q1h474157p4809875

[11] Ordinary and Thebault incircles

http://www.artofproblemsolving.com/community/c6h244007p1341565

[12] Five concurrent lines

http://www.artofproblemsolving.com/community/c6h485516p2720041

[13] From mixtilinear incircles to the Thebault circles

http://www.artofproblemsolving.com/community/c6h213098p1176147

[14] Maybe Thebault have ovelooked

http://www.artofproblemsolving.com/community/c6h570686p3635527

[15] Prove that M = D - [Bulgaria NMO 2010]

http://www.artofproblemsolving.com/community/c6h349730

[16] On a particular case of Thebault’s theorem

http://www.artofproblemsolving.com/community/c6h201858

[17] Exsimilar center

http://www.artofproblemsolving.com/community/c6h617532p3683327

[18] Seven concurrent lines

http://www.artofproblemsolving.com/community/c6h623162p3729318

[19] middle

http://www.artofproblemsolving.com/community/c6h517026p2914609

[20] A little hard for me

http://www.artofproblemsolving.com/community/c6h221652

[21] Concyclic points with triangle incenter

http://www.artofproblemsolving.com/community/c6h41667

[22] incenter of triangle

http://www.artofproblemsolving.com/community/c6h407366

[23] Fairly difficult

http://www.artofproblemsolving.com/community/c6h6086

[24] Prove that

http://www.artofproblemsolving.com/community/c6h503957p2840426

[25] Again with Thebault’s circles

http://artofproblemsolving.com/community/q2h1086479p4806266

[26] Cevian and mixtilinear incircle

http://www.artofproblemsolving.com/community/c6h209898p1156398

(116)

[28] Mỗi tuần toán: Tuần tháng năm 2016

(117)

VỀ BÀI TOÁN TAM GIÁC 80-80-20

Lê Phúc Lữ

(Thành phố Hồ Chí Minh)

Ở cấp THCS, bạn học sinh yêu Toán quen thuộc sợ gặp tốn tính góc Hầu hết địi hỏi phải kẻ thêm đường phụ, điểm phụ xử lý

Trong thế, có lẽ toán sau tiếng phổ biến nhất:

Cho tam giácABC có gócA, B, Clần lượt là20◦,80◦,80◦ Trên cạnhAC, AB, lấy điểm

D, Esao cho

∠BCE = 60◦,∠CBD= 50◦

Tính số đo góc∠CED

Bài tốn biết đến lần vào năm 1909 nhà Toán học Langland Mỹ giới thiệu Dưới giới thiệu số cách chứng minh cho toán số vấn đề liên quan

Các lời giải chúng tơi có tham khảo trang web:

www.cut-the-knot.org/triangle/80-80-20/Classical1.shtml

Cách 1.(đây cách phổ biến nhất)

QuaE, dựng đường thẳng song song vớiBC cắtAC tạiF.GọiN giao điểm củaBF, CE

(118)

Do đó, ta cóEF =EN

Tam giácBCDcó∠BCD = 80◦,∠CBD= 50◦ nên ta tính được∠BDC = 50◦hay tam giác

BCDcân tạiC Do đó,CD = CB = CN hay tam giácCN D cân tạiC.Từ đó, ta có

∠CN D =∠CDN = 80◦, suy

∠DN F = 180◦ −(∠EN F +∠CN D) = 40◦

Suy ra∠DF N =∠CDN −∠DN F = 40◦hay tam giácDN F cân tạiF

Do đó, ta cóDF =DN

Suy raDElà trung trực củaN F hayEDlà phân giác của∠N F E

Vậy ta được∠CED = 12∠CEF = 30◦

Cách 2.

(119)

Tạp chí Epsilon, Số 09, 06/2016

Chú ý tam giácBCDcân tạiC nênBC =CD.Do đó,BN =CD.Ta có

∠N BE =∠CBE−∠CBN = 80◦−60◦ = 20◦

DoCH kABnên ta có∠HCF = 20◦

Suy ra∆BN E = ∆CDH(c.g.c)và∠CHD =∠BEN = 40◦

Mặt khác∠CHF = 80◦ nênHDchính phân giác của∠CHF

Rõ ràng∠ECF =∠HCF = 20◦nênCF phân giác góc∠HCE.Do đó,Dchính tâm đường tròn nội tiếp tam giácCHE.Từ suy

∠CED =

2∠CEF = 30

Cách 3.(Đây có lẽ cách đơn giản cho toán này)

TrênAB, lấy điểmF choCB =CF.Khi đó, ta có∠BCF = 20◦và∠F CD = 60◦

Chú ý rằngCB =CDnênCD =CF, tức tam giácCDF cân ởCnên Do đóDF =CF

Bằng biến đổi góc, ta có∠F CE=∠F EC = 40◦nên tam giácF EC cân tạiF

Từ suy raF C =F D =F E nênF tâm đường trịn ngoại tiếp tam giácCDE

Vậy∠DEC = 12∠DF C = 30◦

(120)

GọiO tâm đường tròn ngoại tiếp tam giácBDE.Ta có

∠DBE = 30◦ ⇒∠DOE = 60◦,

màOD =OEnên tam giácODEđều Do đó:EO =ED

Bằng biến đổi góc, ta có∠DCE = 80◦−60◦ = 20◦

Dễ thấyOC trung trực của∠BCDnên∠OCB = 802◦ = 40◦ nên

∠OCE = 60◦−40◦ = 20◦

Hai tam giácCDE vàCOEcó cạnhCEchung,ED=EO và∠DCE =∠OCE

Tuy góc khơng nằm xen cạnh tương ứng, góc∠DEC,∠OECđều nhọn (nằm góc∠DEOnên nhỏ hơn60◦), thế∆CDE = ∆COE dẫn đến

∠CED =∠CEO =

2∠DEO= 30

(121)

Tạp chí Epsilon, Số 09, 06/2016

Trên tia phân giác góc∠BCD, lấy điểmM cho∠ABM = 30◦

Khi đó, ta cóM D =M B và∠M BD =∠ABM +∠ABD = 30◦+ 30◦ = 60◦, đó, tam giácBDM Khi đó, ta cóCE phân giác góc∠DCM

VìBE phân giác góc∠DBM nên trung trực củaDM

Trong tam giácCDM,Elà giao điểm phân giác góc∠DCM trung trực củaDM nênE

thuộc đường tròn ngoại tiếp tam giácDCM

Do đó,

∠CED =∠CM D = 60

2 = 30

(122)(123)

SỬ DỤNG TỔNG TÍCH PHÂN ĐỂ TÍNH GIỚI HẠN DÃY SỐ

Nguyễn Tài Chung

(Trường THPT Chuyên Hùng Vương, Gia Lai)

1 Giới thiệu

Có nhiều phương pháp thống để tính giới hạn dãy số, chẳng hạn sử dụng định nghĩa giới hạn dãy số, sử dụng định lí Weiertrass, sử dụng dãy con, sử dụng định lí Lagrange, Chuyên đề trình bày phương pháp mà biết vận dụng cách linh hoạt giải nhiều toán giới hạn dãy số liên quan đến tổng Đặc biệt kết hợp với nguyên lí kẹp phương pháp có sức cơng phá lớn, giải nhiều tốn hay, khó tổng quát

2 Nội dung

2.1 Một số kiến thức thường dùng

Cơ sở phương pháp sử dụng tổng tích phân định nghĩa tích phân xác định sau

Định nghĩa 2.1 (Định nghĩa tích phân xác định) Cho hàm số y = f(x) xác định đoạn [a;b] Xét phân hoạch(phép chia)T bất kỳ đoạn[a;b], tức chia đoạn[a;b]thànhn

đoạn điểm chia:

a=x0 < x1 < x2 <· · ·< xn−1 < xn=b

Trên đoạn [xi−1;xi] (i= 1,2, , n)ta lấy điểm αi ∈ [xi−1;xi]và gọi∆i =

xi−xi−1 là độ dài đoạn[xi−1;xi] Đặtd= max{∆1,∆2, ,∆n} Khi n

X

i=1

f(αi)∆i =f(α1)∆1+f(α2)∆2+· · ·+f(αn)∆n

gọi tổng tích phân hàm sốf(x)trên đoạn[a;b]ứng với phép phân hoạchT Tổng này phụ thuộc vào phân hoạchT, số khoảng chianvà phụ thuộc vào cách chọn điểmαi Nếu tồn tại lim

d→0 n

P

i=1

f(αi)∆i (là số xác định), không phụ thuộc vào phép phân hoạch đoạn[a;b]và cách chọn điểmαi thì giới hạn gọi tích phân xác định hàm sốf(x)trên đoạn[a;b]và ký hiệu là

b

Z

a

(124)

Lưu ý.

• Như hàm sốf(x)khả tích đoạn[a;b]thì phép phân hoạchT đoạn

[a;b]và cách chọn điểmαk ∈[xk−1;xk](k số nguyên dương), ta ln có: b

Z

a

f(x)dx= lim d→0

n

X

i=1

f(αi)∆i

• Trong thực hành, đặc biệt ý đến số lớp hàm khả tích đơn giản dễ nhận biết sau đây:

Nếu hàm sốy =f(x)liên tục đoạn[a;b]thì khả tích đoạn

Nếu hàm sốy =f(x)bị chặn đoạn[a;b]và có số hữu hạn điểm gián đoạn khả tích đoạn

Nếu hàm sốy =f(x)bị chặn đơn điệu đoạn[a;b]thì khả tích đoạn

Chú ý 2.1 Cho đoạn[a;b]⊂R Khi với mọiα>0, β >0sao choα+β = 1, ta có

αa+βb∈[a;b]

2.2 Tính giới hạn dãy số Sn = n P i=1

xi

Có nhiều tốn giới hạn dãy số mà việc tính tổng trực tiếpSnrất khó khăn khơng thể

thực Tuy nhiên biến đổi tổng dạng tổng tích phân (của hàmf(x)trên đoạn[a;b]ứng với phép phân hoạchT đó) Vậy để tính giới hạn tổng nhờ tích phân xác định ta thường tiến hành theo bước sau:

Bước 1:Biến đổi tổngSnthành biểu thức

Sn =

b−a n

n

X

i=1

f(a+ib−a n )

Bước 2:Chọn hàmf(x)khả tích đoạn[a;b]

• Lập phép phân hoạch trên[a;b]bởi điểm chia

a=x0 < x1 < x2 <· · ·< xn−1 < xn=b • Lập tổng tích phân hàmf(x)trên[a;b]:

Sn =

b−a n

n

X

i=1

f(a+ib−a n )

Bước 3:Khi lim

n→+∞Sn =

b

Z

a

(125)

Tạp chí Epsilon, Số 09, 06/2016

Đặc biệt:Nếua= 0vàb= 1thì bước trở thành:

Bước 1:Biến đổi tổngSnthành:

Sn= n f n +f n

+· · ·+f n n = n n X i=1 f i n

Bước 2:Chỉ hàmf(x)khả tích đoạn[0; 1]

• Lập phép phân hoạch trên[0; 1]bởi điểm chia

0 =x0 < x1 < x2 <· · ·< xn−1 < xn= • Lập tổng tích phân hàmf(x)trên[0; 1]:

Sn=

n

n

X

i=1

f(i

n)

Bước 3:Khi lim

n→+∞Sn =

1

Z

0

f(x)dx

Bài toán 2.1 ChoSn =

n2 +

n2 +· · ·+

n

n2 Tìmn→lim+∞Sn. Lời giải Ta có:

Sn= n n +

n +· · ·+ n n = n n X i=1 i n

Xét hàm số f(x) = x khả tích [0; 1] Chia đoạn [0; 1] điểm chia xi =

i n,i =

0,1, , n, tức chia đoạn[0; 1]thànhnđoạn nhau:

x0 =

n = 0< x1 =

1

n < x2 =

2

n <· · ·< xn−1 = n−1

n < xn= n n =

Chọnαi =

i

n ∈ [xi−1;xi],∀i = 1,2, , n Khi tổng tích phân hàm sốf(x)trên[0; 1]

ứng với phép phân hoạch nói

Sn= n n +

n +· · ·+ n n = n n X i=1 i n Ta có lim

n→+∞Sn = limn→+∞

1 n n X i=1 i n ! = Z

xdx= x 2 =

Lưu ý.Trong toán 2.1 này, ta phải chọnαi =

i

n trùng với đầu mút phải đoạn[xi−1;xi],

để cho vớif(x) =xthìf(αi) =

(126)

Bài toán 2.2 (Olympic toán Sinh viên tồn quốc năm học 2007-2008) Tìm lim

n→+∞Snbiết Sn =

12008+ 22008+· · ·+n2008

n2009 Lời giải Ta có:

Sn= n " n 2008 + n 2008 +· · ·+ n n 2008 # = n n X i=1 i n 2008

Xét hàm sốf(x) = x2008 Khi đóf(x)khả tích đoạn [0; 1] Chia đoạn[0; 1]bởi điểm chiaxi =

i

n (i= 0,1, , n)và chọn

αi =

i

n ∈[xi−1;xi],∀i= 1,2, , n

Khi tổng tích phân hàm sốf(x)trên[0; 1]là

n

n

P

i=1

f(i

n) =Sn Vậy

lim

n→+∞Sn=

1

Z

0

f(x)dx=

Z

0

x2008dx= x 2009 2009 = 2009

Bài toán 2.3 Chứng minh giới hạn

lim n→+∞

 

sin π

n+

1 +

sin 2π

n+

2 +· · ·+

sin nπ

n+

n 

 

là số dương.

Lời giải Xét hàm sốf(x) =

( sinx

x x∈(0;π]

1 x=

Vì lim

x→0+f(x) = limx→0+

sinx

x = =f(0)nên hàm sốf liên tục đoạn[0;π], hàm sốf

khả tích đoạn[0;π] Hơn nữa:

π

Z

0

f(x)dx >0 (do f(x)>0, ∀x∈[0;π])

Như vậy:

lim n→+∞

n

X

i=1

sin iπ

n+

i = limn→+∞ 

 

π n+

n

X

i=1

sin iπ

n+

iπ n+

(127)

Tạp chí Epsilon, Số 09, 06/2016

= lim n→+∞

π n+

n X i=1 f iπ n+

!

= π

Z

0

f(x)dx >0

Ta có điều phải chứng minh

Bài tốn 2.4 Tính lim

n→+∞Sn, với: Sn =

1

n+2

+

n+

+

n+14

+· · ·+

n+6n−4

,∀n= 1,2,

Phân tích Ta biến đổi:

Sn = c.c n X i=1

n+ 6i−4 = c c n n X i=1 1 + 6i−4

3n ,

vớicsẽ độ dài đoạn lấy tích phân Ta dự đốn hàm số dấu tích phân làf(x) = 1 +x

và đoạn lấy tích phân là[d;d+c] Xét phép phân hoạch đoạn[d;d+c]bởi điểm chia

x0 =d, x1 =d+

c.1

n , x2 =d+ c.2

n , , xn=d+ c.n

n =d+c

Khi đó∆i =xi−xi−1 =

c

n,∀i= 1,2, , n Giả sửαi ∈[xi−1;xi], suy raαicó dạng: αi =axi−1 +bxi, với a>0, b>0, a+b =

Ta cần tìmavàbsao cho: 6i−4

3n =axi−1+bxi Tức tìmavàbsao cho

2i n −

4 3n =a

d+c(i−1)

n

+b

d+ ci

n

= (a+b)ci

n −

ac

n +d(a+b)

Suy ra:     

(a+b)c=

ac=

d(a+b) =

doa+b=1 ⇒     

c=

a= 3, b =

1

d=

Bởi ta có lời giải sau

Lời giải Ta có:

Sn= n

X

i=1

1

n+6i−4 =    n n X i=1 1 + 6i−4

(128)

Xét hàm sốf(x) =

1 +x đoạn[0; 2] Rõ ràngf(x)liên tục trên[0; 2]nên khả tích

đoạn Xét phép phân hoạch đoạn[0; 2]bởi điểm chia

x0 = 0, x1 = 2.1

n , x2 =

2.2

n , , xn−1 =

2.(n−1)

n , xn=

2.n n =

Khi đó∆i =xi−xi−1 =

n,∀i= 1,2, Xét: αi =

2

3xi−1+

3xi ∈[xi−1;xi]

Khi đó:

αi =

2 (i−1)

n +

1

2i n =

6i−4

3n , f(αi) =

1 + 6i−4

3n

Suy ra:

Sn=    n n X i=1 1 + 6i−4

3n   = n X i=1

f(αi)∆i

Đây tổng tích phân hàm sốf(x)trên đoạn[0; 2](ứng với phép phân hoạch đoạn

[0; 2]và phép chọn điểmαinhư nói trên) Do theo định nghĩa tích phân ta có lim

n→+∞Sn =

1

2

Z

0

f(x)dx=

2ln = ln √

3

Bài tốn 2.5 Tính lim n→+∞

1k+ 3k+· · ·+ (2n−1)k

nk+1 Lời giải Ta có:

1k+ 3k+· · ·+ (2n−1)k

nk+1 =

n

1k+ 3k+· · ·+ (2n−1)k

nk ! =2 k n 2n k + 2n k +· · ·+

2n−1 2n

k!

Do 2i−1

2n =

1

i−1

n + i n

là trung điểm đoạn

i−1

n ; i n

, i= 1,2, , n

nên k n 2n k + 2n k +· · ·+

2n−3 2n

k

+

2n−1 2n

k!

là tổng tích phân hàm

số f(x) = 2k.xk trên đoạn [0; 1]ứng với phép phân hoạch đoạn [0; 1]bởi điểm chia

xi =

i

n (i= 0,1, , n)và chọnαi =

2i−1

2n trung điểm đoạn

(129)

Tạp chí Epsilon, Số 09, 06/2016

Như vậy:

lim n→+∞

1k+ 3k+· · ·+ (2n−1)k

nk+1 =

Z

0

2k.xkdx= 2k x k+1

k+

= k

k+

Bài toán 2.6 Tính lim

n→+∞Sn, với: Sn = 2n

n−1

X

i=1

1

(2i+ 1)2+ 4n2,∀n= 0,1,2,

Lời giải Xét hàm số f(x) =

1 +x2, ta có

Z

0

f(x)dx =

Z

0

dx

1 +x2 =

π

4 Ta chia đoạn [0; 1]

thành2nphần điểm chia

x0 = 0, x1 =

2n, x2 =

2

2n, , x2n−1 =

2n−1

2n , x2n=

2n

2n =

Khi đó∆i = xi −xi−1 =

2n,∀i = 1,2, ,2n Trên đoạn[x2i;x2i+1]hay[x2i+1;x2i+2]

(i= 0,1, , n−1)ta chọn điểmαi =

2i+

2n Thế tổng tích phân hàmf(x)trên

[0; 1]là

f(α0)∆1+f(α1)∆2+f(α2)∆3+· · ·+f(α2n−2)∆2n−1+f(α2n−1)∆2n = 2n    1 +

4n2

+

1 + 4n2

+

1 + 4n2

+· · ·+ 1 + (2n−1)

2 4n2

   = 2n

4n2

1 + 4n2 + 4n2

32+ 4n2 + 4n2

52+ 4n2 +· · ·+

4n2

(2n−1)2+ 4n2

= 2n

n−1

X

i=0

4n2

(2i+ 1)2 + 4n2 =Sn

Vậy lim n→+∞Sn=

1

Z

0

f(x)dx=

Z

0

dx

1 +x2 =

π

4

Lưu ý.

• Trong cách chọn thìαi chọn đầu mút chung hai đoạn liền kề • Để áp dụng thành cơng phương pháp tổng tích phân, phải biến đổi

Sn =x1+x2+· · ·+xn

(130)

Trên đoạn[0; 1], [0;a]hay[a;b]phù hợp;

Thường với phép chia miền lấy tích phân(rất cần thừa số

n);

Chọn điểm trung gianαi mút trái, mút phải, đầu mút hai đoạn liền kề nhau,

hay trung điểm đoạn thứi, chí phải chứng minhαichính điểm

của đoạn thứi

• Nhiều tổngSn=x1+x2+· · ·+xnkhơng phải tổng tích phân, mà "gần như"

vậy, trường hợp phương án đưa thường đổi tổng; thay số hạng tổng; thêm bớt lượng thích hợp để đưa tổng tích phân hàm số

2.3 Phương pháp đổi tổng

• Trong tổng tích phân hàm sốf có biểu thức

n

P

i=1

f(αi), nhiên số tậpi

không phải chạy từ1đếnn, mà q trình khác Lúc ta phải đổi tổng, tức thayibởi biểu thức

• Cũng có giới hạn cần tính khơng phải tổng tích phân hàm nào, lúc ta cố gắng thay số hạng tổng quát tổng số hạng khác Phương pháp thường sử dụng tính liên tục hàm số (xem tốn 2.9 trang 130) sử dụng định lí Lagrange (xem toán 2.10 trang 130, toán 2.11 trang 130)

Bài tốn 2.7 Tính lim n→+∞ n

2n−1

X

k=n

k2

!

.

Lời giải Ta đổi tổngkchạy từnđến2n−1bằng tổngichạy từ0như sau:

n

2n−1

X

k=n

k2 =n n−1

X

i=0

(i+n)2 (do thaykbởii+n) =

n

n−1

X

i=0

1

1 + i

n =

1

n

n−1

X i=0 f i n ,

vớif hàm sốf(x) =

(1 +x)2 Chia đoạn[0; 1]bởi điểm chia

xi =

i

n, i= 0,1, , n,

tức chia đoạn[0; 1]thànhnđoạn nhau:

x0 =

n = 0< x1 =

1

n < x2 =

2

n <· · ·< xn−1 = n−1

(131)

Tạp chí Epsilon, Số 09, 06/2016

Chọnαi =

i

n ∈[xi;xi+1],∀i= 0,1,2, , n−1 Khi tổng tích phân hàm sốf(x)trên

[0; 1]ứng với phép phân hoạch nói

n

n−1

X i=0 f i n

Như vậy:

lim n→+∞ n

2n−1

X

k=n

k2

!

= lim n→+∞

1

n

n−1

X i=0 f i n = Z

f(x)dx=

Z

0

dx

(1 +x)2 = −

x+

=−1

2 + =

Bài tốn 2.8 Tính lim n→+∞

2n−1

X

k=n 2k+ 1.

Lời giải Trongun= 2n−1

X

k=n

2k+ 1, thayk bởii+n, ta được:

un= 2n−1

X

k=n 2k+ =

n−1

X

i=0

1 2i+ 2n+

Đặtvn= n−1

X

i=0

2i+ 2n, đó:

vn= 2·

1

n

n−1

X

i=0 1 + i

n

= 2·

1

n

n−1

X i=0 f i n ,

vớif hàm sốf(x) =

1 +x Như vậy:

lim n→+∞vn =

1

1

Z

0

f(x)dx=

1

Z

0

dx

1 +x =

1

2ln(1 +x)

= 2ln

Lại có:

|un−vn|=

n−1

X

i=0

1

2i+ 2n+ − 2i+ 2n

=

n−1

X

i=0

1

(2i+ 2n)(2i+ 2n+ 1)

n

2n(2n+ 1), ∀n= 1,2,

Mà lim n→+∞

n

2n(2n+ 1) nên sử dụng nguyên lí kẹp suy ran→lim+∞|un−vn|=

Như vậy:

un− 2ln

=

(un−vn) +

vn−

1 2ln

|un−vn|+

vn− 2ln

Từ lại sử dụng nguyên lí kẹp ta lim n→+∞un =

(132)

Bài tốn 2.9 Tìm giới hạn:

lim n→+∞

 

21n

n+ + 22n

n+

+· · ·+

n n

n+

n 

 

Lời giải Biến đổi tổng dấu giới hạn dạng

Sn= 2n1

n+ + 22n

n+

+· · ·+

n n

n+

n

= n

X

i=1 2ni

n+1

i = n X i=1 n

2ni

1 +

ni

Dường vế phải khơng phải tổng tích phân hàm đoạn[0; 1]với phép chia thànhnđoạn chọn điểmαi mút trái, mút phải trung điểm thông thường

Tuy nhiên dễ chứng minh

2i−n1 = i n

2 2ni

1 +

ni

62ni,∀i= 1,2, , n

do 1<1 +

ni 62

Vì hàm sốg(x) = 2xliên tục trênRnên tồn tạiαi ∈

i−1

n ; i n

để

2αi = i n

1 + ni1

Từ đóSn = n

P

i=1

n.2

αi Mà

n

P

i=1

n.2

αi là tổng tích phân hàm sốg(x) = 2x trên đoạn[0; 1]với

phép chia đoạn[0; 1]thànhnđoạn cách chọn điểmαi Do lim

n→+∞Sn=

1

Z

0

2xdx= x ln = ln

Bài toán 2.10 Giả sử hàm số f(x) có đạo hàm khả tích bị chặn đoạn [a;b] Đặt

Sn=

b−a n n X i=1 f

a+ib−a n

.Chứng minh rằng:

lim n→+∞n

 Sn−

b

Z

a

f(x)dx 

= b−a

2 [f(b)−f(a)]

Bài toán 2.11 Giả sử hàm sốf(x)có đạo hàm cấp hai khả tích bị chặn đoạn[a;b] Đặt

Sn=

b−a n n X i=1 f

a+ (2i−1)b−a 2n

.Chứng minh rằng:

lim n→+∞n

2   b Z a

f(x)dx−Sn

=

(b−a)2 24 [f

0

(133)

Tạp chí Epsilon, Số 09, 06/2016

Lưu ý.Bài tốn 2.10, toán 2.11 kết quan trọng, định hướng cho việc tính giới hạn số dãy số liên quan đến tổng xây dựng hệ thống tập Cịn việc giải tốn 2.10 tốn 2.11 bạn đọc tham khảo tài liệu tham khảo sáchChuyên khảo Dãy số(NXB ĐHQG Hà Nội) xuất năm 2013 tác giả

(134)(135)

SÁNG TẠO VỚI MỘT BÀI TỐN HÌNH HỌC TRUNG HỌC CƠ SỞ

Nguyễn Ngọc Giang (TP Hồ Chí Minh)

Ở Trung học sở, toán đường phân giác tốn đường trung tuyến chiếm vai trị quan trọng Sở dĩ chúng quan trọng chúng có số tính chất đẹp áp dụng nhiều giải tốn Sáng tạo tốn bậc học khó bậc học cấp cơng cụ giải tốn cịn tương đối Đối với hình học, việc sáng tạo tốn ngồi việc tìm cách giải áp dụng định lí quen thuộc Thales, Menelaus, phương pháp vẽ đường phụ phương pháp u thích Thêm vào đó, phương pháp tương tự hóa, khái quát hóa phương pháp ưa dùng Xin giới thiệu với bạn điều vừa nói qua tốn hay sau

Bài toán 1.Cho4ABCđường trung tuyếnBM cắt đường phân giácCD tạiP Chứng minh

P C P D −

AC BC =

Sau cách giải toán 1:

Cách 1VẽDK kBM Ta có

P C P D =

M C M K =

M A M K

= AB

(136)

Cách 2VẽDI kAC Ta có

P C P D =

M C DI =

M A DI

= AB

BD = AB BC +

Cách 3KẻAN kBM Suy raP M đường trung bình của4AN C, tức làP N =P C Ta có

P C P D =

P N P D

Mà N D

DP = AD BD =

AC

BC(doAN kBP)

⇒ N D

DP + = AC BC +

⇒ P C

P D = P N P D =

(137)

Tạp chí Epsilon, Số 09, 06/2016

Cách 4KẻAHkP C Suy P C

P D = AH

DP(do4AM H =4CM P nênP C =AH)

Hay P C

P D = AC

BC + 1(doDP kAH)

Cách 5Trên tia đối tiaM Blấy điểmE choM B =M E DoM A=M C ⇒ABkCEvàAB =CE

Ta có P C

P D = CE BD =

AB BD =

AC BC +

Cách 6VẽCS kBM

(138)

DoBM kCS ⇒ P C

P D = BS BD =

AB BD Hay P C

P D = AB BD =

AC BC +

Cách 7P C

P D = SBP C

SDBP

và AB

BD = SABP

SDBP

MàSABM =SBM C,SAP M =SCP M ⇒SABP =SBP C

Suy P C

P D = AB BD =

AC BC +

Cách 8Kéo dàiAP cắtBCtạiN Áp dụng định lí Ceva ta có M A

M C · N C N B ·

DB

DA = 1(1)

(139)

Tạp chí Epsilon, Số 09, 06/2016

Vậy N C

N B = DA DB(3)

Áp dụng định lí van Aubel ta có P C

P D = M C M A +

N C N B(4)

Từ (1), (2), (3), (4)⇒ P C

P D = + AC BC

Suy điều phải chứng minh

Bài toán tương tự toán toán sau

Bài toán 2Cho 4ABC,M điểm nằm AC cho AM

M C =

1

3 Đường thẳngBM cắt

đường phân giácCD của4ABC tạiP Chứng minh

P C P D −

3AC BC =

Cách 1VẽDK kBM Ta có

P C P D =

M C M K =

3M A M K

= 3AB

BD =

(140)

Cách 2VẽDI kAC Ta có

P C P D =

M C DI =

3M A DI

= 3AB

BD =

3AC BC +

Cách 3KẻAN kBM

(141)

Tạp chí Epsilon, Số 09, 06/2016

Vậy P C

P D =

3P N P D

Mà N D

DP = AD BD =

AC

BC(doAN kBP)

Suy 3N D

DP + =

3AC BC +

Suy P C

P D =

3P N P D =

3AC BC +

Cách 4KẻAHkP C Theo định lí Thales, P C

AH = M C M A =

Suy P C

P D =

3AH DP

Hay P C

P D =

3AC

BC + 3(doDP kAH)

Cách 5Trên tia đối tiaM Blấy điểmE cho M A

M C = M B M E =

AB CE =

(142)

Vậy theo định lí Thales đảo, ta cóABkCE

⇒ P C

P D = CE BD =

3AB BD =

3AC BC +

Cách 6VẽCS kBM

Theo định lí Thales, ta cóBM đường thẳng song song với CSnên BS

AB = M C

M A = Mà BM kCS⇒ P C

P D = BS BD =

3AB BD (1)

Ta có 3AB

BD =

3AC

BC + 3(2)

Từ (1), (2), ta có P C

P D =

3AC BC +

Cách 7 P C

P D = SBP C

SDBP

, AB

BD = SABP

(143)

Tạp chí Epsilon, Số 09, 06/2016

màSABM =

3SBM C,SAP M =

3SCP M ⇒SABP = 3SBP C

Suy P C

P D =

3AB BD =

3AC BC +

Cách 8Kéo dàiAP cắtBCtạiN Áp dụng định lí Ceva ta có M A

M C · N C N B ·

DB

DA = 1(1)

Theo giả thiếtM A= 3M C

Vậy N C

N B =

3DA DB (2)

Áp dụng định lí van Aubel, suy P C

P D = M C M A +

N C N B(3)

Từ (1), (2), (3)⇒ P C

P D = +

3AC

(144)

Bài toán 3Cho4ABC,M điểm nằm trênACsao cho M C

M A =n(n >0)Đường thẳngBM

cắt đường phân giácCDcủa4ABC tạiP Chứng minh

P C P D =

nAC BC +n

Cách 1VẽDK kBM Ta có

P C P D =

M C M K =

nM A M K

= nAB

BD = nAC

BC +n

Cách 2VẽDI kAC Ta có

P C P D =

M C DI =

nM A DI

= nAB

BD = nAC

(145)

Tạp chí Epsilon, Số 09, 06/2016

Cách 3KẻAN kBM

Suy P M đường thẳng song song với cạnh AN 4AN C Theo định lí Thales ta có

P C =nP N

Vậy P C

P D = nP N

P D

Mà N D

DP = AD BD =

AC

(146)

⇒ nN D

DP +n= nAC

BC +n

⇒ P C

P D = nP N

P D = nAC

BC +n

Cách 4KẻAHkP C Theo định lí Thales ta có P C

AH = M C

M A =n Suy raP C =nAH

⇒ P C

P D = nAH

DP

Hay P C

P D = nAB

BD = nAC

BC +n

Cách 5Trên tia đối củaM B lấy điểmEsao cho M C

M A = M E M B =

CE AB =n

Theo định lí Thales đảo, ta cóABkCE

⇒ P C

P D = CE BD =

nAB BD =

(147)

Tạp chí Epsilon, Số 09, 06/2016

Cách 6VẽCS kBM

Theo định lí Thales, ta cóBM đường thẳng song song vớiCSnên BS

AB = M C

M A =nmàBM

kCS

⇒ P C

P D = BS BD =

nAB BD (1)

Ta có nAB

BD = nAC

BC +n(2)

Từ (2), (3), ta có P C

P D = nAC

BC +n

Cách 7 P C

P D = SBP C

SDBP

, AB

BD = SABP

SDBP

Mà SABM =

nSBM C,SAP M =

1

nSCP M ⇒SABP =

1

(148)

⇒ P C

P D = nAB

BD = nAC

BC +n

Cách 8Kéo dàiAP cắtBCtạiN Áp dụng định lí Ceva ta có

M A M C ·

N C N B ·

DB

DA = 1(1)

Theo giả thiếtM C =nM A Vậy N C

N B = nDA

DB

Áp dụng định lí van Aubel ta có

P C P D =

M C M A +

N C N B (3)

Từ (1),(2),(3), P C

P D =n+ nAC

BC

Chúng ta vừa có số khám phá thú vị xoay quanh toán giao điểm đường trung tuyến đường phân giác Các cách giải khác nhau, toán tương tự mở rộng đem đến cho nhiều điều thú vị Bài viết cần trao đổi thêm? Mong chia sẻ bạn

Sau số tốn luyện tập

Bài tốn 4Cho4ABCcó trung tuyếnBM.Dlà điểm trênABsao cho DB

DA =

2CB

CA GọiP

là giao điểm củaCDvàBM Tính tỉ số P C

P D theo CA CB

Bài tốn 5Cho4ABC có trung tuyếnBM.Dlà điểm trênABsao cho DB

DA = nCB

CA GọiP

là giao điểm củaCDvàBM Tính tỉ số P C

P D theo CA CB

Tài liệu tham khảo

(149)

Tạp chí Epsilon, Số 09, 06/2016

[2]Phan Đức Chính, Tơn Thân, Nguyễn Huy Đoan, Lê Văn Hồng, Trương Công Thành, Nguyễn Hữu Thảo (2011), Toán tập 2, Nhà xuất Giáo dục Việt Nam

(150)(151)

“NẾU BẠN KHÔNG NUÔI DƯỠNG, ĐAM MÊ SẼ TỪ BỎ BẠN" - TRÒ CHUYỆN VỀ CON ĐƯỜNG ĐẾN VỚI CMU CỦA PHẠM HY HIẾU

(Trung Dũng - Tạp chí Người thị)

Được Google ba lần gửi lời mời làm việc Nghiên cứu lĩnh vực thời cơng nghệ cao: Trí thơng minh nhân tạo Xác định làm Google năm, sau dành thời gian cho chương trình tiến sĩ Đại học Carnegie Mellon (CMU) Phạm Hy Hiếu trường hợp người Việt trẻ nỗ lực khẳng định thân tìm vị trí giới ngày phẳng xích lại gần nhờ bước tiến vũ bão công nghệ Thế người biết rằng, để có hoạch định ấy, người bạn trẻ khơng lần bị thách thức tốn hóc búa đặt từ sống

Báo chí nước dẫn nguồn tin từ du học sinh Việt Nam Mỹ, giai thoại ba lần bạn nhận lời mời Google (trong hai lần đầu từ chối) Lựa chọn đường đến với “gã khổng lồ” Google bạn thực nào? Đó định đầy cân nhắc bởi bạn có hội đến từ Apple, Microsoft?

Con đường đến với Google dài Tôi thay đổi từ chỗ muốn làm cho Google danh tiếng cơng ty đến chỗ đồng ý làm cho Google nghĩ học nhiều kinh nghiệm có ích từ họ

Nhớ hồi năm đại học, bỡ ngỡ thiếu thông tin công ty, đánh liều gửi hồ sơ xin thực tập Google Họ nói “khơng vấn” tơi chưa có kinh nghiệm lập trình Thấy vậy, tơi học vài lớp lập trình Stanford năm hai xin việc lại Năm thứ hai đại học, vấn vượt qua tất câu hỏi họ, cuối khơng nhận họ nói tơi “khơng hợp với đề án thực tập” Lúc thật khơng hiểu Google muốn tuyển ứng viên

Cũng thời gian này, suy nghĩ việc học dần thay đổi Tôi nhận phải học mà thích cố gắng để trở thành người có lực, học để kiếm công việc hay để gây ấn tượng với công ty Nghĩ thế, tơi khơng quan tâm đến Google Tơi bắt đầu học trí tuệ nhân tạo, thích học đó, cố gắng để tiến

(152)

Sau tốt nghiệp, lại Google nhận làm việc thức Lần này, tơi cảm thấy câu hỏi vấn họ bắt đầu làm khó Nhưng tơi hỏi họ cịn giữ sách “hợp với đề án” khơng biết họ cịn giữ, tơi nói với họ tơi không cảm thấy thoải mái cống hiến cho cơng ty đối xử với ứng viên vậy, lại từ chối họ

Phạm Hy Hiếu màu áo đội Cardinal Đại học Stanford tham dự vịng chung kết Cuộc thi lập trình ACM dành cho sinh viên đại học toàn giới tổ chức Nga năm2014.Việt Nam có số sinh viên tham gia thi tin học quốc tế (IOI) tham gia vòng chung kết giới màu áo đại học MIT (Mỹ), NUS (Singapore), chuyển từ thi toán quốc tế (IMO) sang thi ACM Phạm Hy

Hiếu trường hợp đặc biệt - Ảnh CTV

Đến tháng 3.2016 vừa rồi, Google lần mời làm việc Tôi lại hỏi sách tuyển dụng họ, biết họ thay đổi, bắt đầu cân nhắc việc làm vào làm cho Google Lúc này, so sánh Google với Facebook, Apple, Microsoft việc học tiến sĩ Đại học Carnegie Mellon (CMU) Đây định khó khăn Tơi hỏi xin lời khun từ giáo sư hướng dẫn Stanford nhiều bậc đàn anh giáo sư hướng dẫn CMU Cuối cùng, sau suy nghĩ nghe lời khuyên, định làm việc Google Brain năm vào CMU làm tiến sĩ

Bí để “lọt vào mắt xanh” công ty hàng đầu gì? Bởi thực tế việc xét hồ sơ trao học bổng trường đại học nhắm đến học sinh giỏi tài khơng cịn quan niệm nhà nghèo học giỏi, cơng ty nhắm đến ứng viên có năng lực thực sự, ranh giới vùng miền, chủng tộc hay quốc gia mờ Bạn có đồng tình quan điểm đó?

(153)

Tạp chí Epsilon, Số 09, 06/2016

hướng học tập tơi Tơi học mà tơi thích học theo đam mê Tơi học tơi muốn nâng cao lực thân, để cống hiến cho lĩnh vực u thích Tơi nghĩ bạn có lực, Apple, Facebook, Google, Microsoft, hay tương tự, tự tìm đến Ngược lại, bạn học điều hội bạn giảm nhiều

Tơi khơng đồng tình với quan điểm “ranh giới vùng miền, chủng tộc mờ” Tơi tin giới ngày phẳng gần hơn, chừng mục đó, ranh giới cịn Tơi hy vọng tương lai, ranh giới mờ dần tiến tới chỗ hội mở cho tất người

Hãy kể chút công việc bạn?

Tôi tham gia nghiên cứu sau đại học trí tuệ nhân tạo Đại học Stanford Hướng nghiên cứu ứng dụng mạng nơ-rơn giúp máy tính hiểu ngơn ngữ người Tôi tập trung vào

nghiên cứu đa ngơn ngữ, nhằm giúp máy tính hiểu giao tiếp với người dùng nhiều ngôn ngữ khác Hiện nay, nhiều trí tuệ nhân tạo tương tác với người tiếng Anh tốt so với ngôn ngữ khác Nguyên nhân phổ biến tiếng Anh, khiến người ta dễ dàng thu thập liệu cho máy tính học Tuy nhiên, máy tính sử dụng ngơn ngữ khác, tốt tiếng Anh, người dân nhiều nước giới tiếp cận công nghệ

Khi cơng nghệ đa ngơn ngữ trở nên hồn thiện hơn, tin người không thiết phải học tiếng Anh để nhằm mục đích tiếp cận cơng nghệ Thay vào đó, họ đầu tư thời gian rèn luyện thật giỏi kỹ đó, sau dựa vào máy tính để tiếp nhận truyền đạt kiến thức cho tồn giới

Từ tháng6.2016,tôi bắt đầu làm việc Google sau năm bước vào chương trình tiến sĩ CMU Có thể hướng nghiên cứu tơi thay đổi Tơi khơng nói trước được, phải nhờ vào duyên số

Vừa qua người quan tâm kiện trí thơng minh nhân tạo chiến thắng người Bạn có thể cho người hình dung chút AlphaGo? Theo bạn, chiến thắng giới nghiên cứu trí tuệ nhân tạo đánh giá cao?

(154)

Chiến thắng AlphaGo đánh giá cao Go (cờ vây) trị chơi cho khó trí tuệ nhân tạo Trước AlphaGo, chuyên gia trí tuệ nhân tạo tin sớm muộn máy tính chiến thắng người cờ vây, nhiên họ dự đốn điều xảy vòng10năm, tức khoảng năm2025

Thành công AlphaGo cho thấy tốc độ phát triển trí tuệ nhân tạo vượt qua dự đốn họ, đánh giá cao AlphaGo mở nhiều hướng nghiên cứu trí tuệ nhân tạo Các thuật tốn dùng trí tuệ nhân tạo ứng dụng để giúp máy tính hiểu ngơn ngữ tự nhiên hay nhận diện giọng nói hình ảnh Ngồi ra, thành công AlphaGo khiến nghiên cứu trí tuệ nhân tạo nhận nhiều quan tâm cộng đồng Thơng qua đó, nghiên cứu thu hút nhiều nguồn đầu tư, từ tạo tiềm phát triển thêm

Là người góp phần giúp cho cỗ máy trở nên thơng minh, biết tư cảm xúc, có bao bạn lo ngại người bị máy móc điều kiển lại? Bởi, theo cách hiểu đó, con người làm biếng suy nghĩ mà phụ thuộc vào công nghệ; chứng nghiện smartphone hay mạng xã hội làm người giao tiếp với nhau, tưởng gần lại xa?

Trí tuệ nhân tạo phát triển nhanh, chúng cịn xa đến chỗ điều khiển người Lo lắng điều lúc khiến nghiên cứu trí tuệ nhân tạo tập trung cách khơng cần thiết Bản thân không cho người làm biếng suy nghĩ hay phụ thuộc vào công nghệ Trước có máy tính, người phải tự thực nhiều phép toán Sau ứng dụng tính tốn phát triển, sử dụng ứng dụng thay mình, đâu có nói phụ thuộc vào ứng dụng này? Trí tuệ nhân tạo Tơi nghĩ chúng phần mềm giúp làm việc hiệu hơn, tiết kiệm công sức thời gian để tập trung cho công việc quan trọng

Bạn có dõi theo hoạt động đời sống tốn học nước? Những chuyển động mang lại cho bạn suy nhĩ hệ đàn em tiếp bước?

Tơi khơng có chun mơn tốn cao cấp nên khơng hiểu nhiều nghiên cứu hoạt động chuyên sâu toán, ngồi nước Hoạt động tốn học nước mà tơi cịn theo dõi phong trào olympic tốn Tơi thấy việc dạy học toán olympic nước cởi mở Các em học sinh có nhiều hội để tiếp xúc với tài liệu tham gia hoạt động giao lưu học hỏi Do đó, chất lượng dạy học tăng lên Tơi nghĩ em biết kiên trì với đam mê định hướng mình, em xa hệ chúng tơi

Tốn học hỗ trợ cho bạn công việc – nghiên cứu trí tuệ nhân tạo? Thuật tốn học sâu - Deep Learning hướng thời thượng cho việc nghiên cứu trí tuệ nhân tạo? Cơ hội có tầm cho bạn trẻ Việt Nam?

Việc bạn bước ra khỏi trường đại học như quan trọng nhiều

so với việc bạn

bước vào trường

đại học nào.

(155)

Tạp chí Epsilon, Số 09, 06/2016

Deep learning hướng thời thượng nhiều năm nữa.Thuật tốn chuyển khó khăn trí tuệ nhân tạo từ việc thu thập liệu, vốn thời gian tốn kém, thành việc lập trình mạng nơ-rơn lớn, có khả xử lý nhiều liệu đơn giản cách nhanh hiệu quả.Với phát triển phần cứng máy tính, việc giải nhìn chung khả thi Vì deep learning đạt nhiều thành tựu

Deep learning không tầm so với bạn trẻ Việt Nam Ngược lại, trở nên phổ biến dễ tiếp cận nhiều Lấy ví dụ, năm2012,TS Lê Viết Quốc Google công bố phương pháp dùng deep learning để xây dựng khái niệm mèo từ video YouTube cách kết hợp16.000CPU nhiều máy tính Giá mơ hình thời điểm khoảng

1triệu USD Tuy nhiên, vài tháng sau đó, nhóm nghiên cứu Stanford cơng bố mơ hình deep learning khác, cho kết tương tự có giá khoảng20nghìn USD Đó năm2012,cịn thiết bị mua với vài nghìn USD Chi phí này, theo tơi, tầm nhiều sở nghiên cứu nước ta Một có thiết bị, làm hay khơng hồn tồn phụ thuộc vào người

Đâu lựa chọn khó bạn cơng việc sống?

Điều khó tơi lựa chọn từ chối học bổng Đại học NUS (Singapore) đồng thời nhà năm để học tiếng Anh tìm học bổng để Mỹ Thật lựa chọn mà định hướng bố mẹ Càng lâu dài, tơi gia đình nhận định sai lầm Nó để lại nhiều hậu cho thân tơi mà đến phải khắc phục

Và bạn dung hoà giữ tự lập, thích nghi với mơi trường nào? Nếu được lựa chọn lại, bạn có phủ nhận ước mơ trước Singapore lãng mạn, mang tính háo hức tuổi trẻ?

Bây nghĩ lại chuyện Singapore, chưa vượt qua cảm giác tiếc nuối Để du học đại học Mỹ, phải trả giá đắt tôi: năm không học làm điều thích Thời cấp ba, tơi đầu tư cho thi học sinh giỏi nên trình độ tiếng Anh khơng đủ để vượt qua kỳ thi chuẩn hóa vào đại học Mỹ TOEFL SAT Tơi khơng hiểu biết phong tục văn hóa người Mỹ để viết essay tuyển sinh cho họ đọc Cái giá tất điều năm bỏ để học điều chưa quan tâm trước

Một năm tơi tai hại vô Tôi phải học điều cho vô nghĩa Tai hại hơn, để học tiếng Anh văn hóa Mỹ, tơi phải cấm thân khơng học tốn, cần đụng vào tốn, tơi dễ dàng bỏ vài để tìm hiểu vấn đề thích mà qn nhiệm vụ chính: học tiếng Anh Những thi tốn olympiad hiểu, cần vài tuần không luyện tập, kỹ rèn luyện nhiều năm dần đi, đừng nói đến năm tơi

(156)

Phạm Hy Hiếu (áo trắng) nhận giải thưởng Ben Wegbreit dành cho luận văn tốt nghiệp xuất sắc khoa Khoa học máy tính, Đại học Stanford2015- Ảnh tư liệu gia đình

Tuy nhiên điểm SAT không cao Tôi thi lần 2.090, điểm số nằm trong25%

thấp khóa học Stanford tơi Khi tơi vào Stanford, điểm SAT khơng cao mà kỹ tốn tơi giảm sút Hồi năm nhất, có lần giáo sư nói chuyện với tơi có đề cập đến bổ đề Cauchy-Davenport Tôi nhớ tên bổ đề này, khơng thể nhớ nội dung Vị giáo sư ngạc nhiên tơi đại diện Việt Nam IMO mà bổ đề quan trọng Tôi xấu hổ vô Tôi kể điều để thấy rằng, phải bỏ lại điều nhiều thời gian cơng sức để học Tơi cịn gặp nhiều rắc rối khác với Stanford Từ ngôn ngữ, văn hố, cách sinh hoạt, cách hồ nhập Tơi sống qua thời đại học với nhiều buồn vui

Đổi lại, tin Singapore, điều năm để học SAT, TOEFL, Xung quanh tơi có nhiều bạn bè mà quen từ cấp Những người bạn đó, tơi tin họ hiểu tơi giúp tơi học tập, tiến bộ, tốt nhiều so với thực tế mà phải trải qua

Tôi xin gửi đến thân của6năm trước, bạn trẻ nghĩ đại học, du học, v.v lời nhắn nhủ: Hãy hỏi thân bạn thích điều cố gắng làm điều thật tốt Đừng tên Stanford, MIT hay Harvard hay mà “gác lại” đam mê mình, để làm điều khơng thích Bởi bạn khơng ni dưỡng, đam mê từ bỏ bạn Cuộc đời bạn có khoảng4năm trường đại học phần sau dài nhiều Vì thế, việc bạn bước khỏi trường đại học quan trọng nhiều so với việc bạn bước vào trường đại học Dù bạn thích tốn, tin học, văn học, piano, nhảy đại hay hoạt động xã hội, đam mê bạn đáng trân trọng Hãy đầu tư vào đam mê Chính điều chìa khố để bạn thành cơng

(157)

Tạp chí Epsilon, Số 09, 06/2016

Tôi nhận nhiều ảnh hưởng việc định hình tính cách từ bố mẹ Tơi học từ mẹ tơi cá tính phải bảo vệ quan điểm biết sai Thật cứng nhắc khiến mẹ nhiều lần bất đồng quan điểm Càng lớn, bảo vệ quan điểm trước mẹ Ngược lại, tơi học từ bố đức tính coi trọng lực người phần quan niệm - đời Khi làm việc nhóm đạt phần thưởng, tơi thường tự đóng góp phần vào liên hoan vào quỹ chung Bởi lẽ quan điểm quý nhất, kỹ để đạt phần thưởng kia, học

Là người có trải nghiệm mơi trường học đặc biệt, nước bạn học những trường Top đầu thành phố, du học với học bổng toàn phần đại học thạc sĩ của Stanford Dành nhiều phần thưởng giá trị thời gian học, đặc biệt danh dự Stanford Những phần thưởng ngẫu nhiên đến với bạn mục tiêu bạn ngắm đến từ trước? Bạn có thấy thân may mắn? Bí sống học nơi xứ người, trì được “phong độ” gì?

Khi tơi làm việc trong

một nhóm đạt

được phần thưởng,

tôi thường tự đóng góp phần vào một cuộc liên hoan hoặc vào quỹ chung Bởi lẽ tôi quan điểm cái quý nhất, kỹ để đạt phần thưởng kia, học được rồi.

Tơi nghĩ gặp nhiều may mắn Người ngồi nhìn vào tơi thấy giải thưởng, lời mời làm việc học bổng Nhưng thân với hiểu tơi trải qua thời khắc khó khăn Tơi chưa dám nghĩ đạt tơi có Tơi cảm thấy điều đến với tơi cách ngẫu nhiên, nụ cười số phận, khuyến khích tơi tiếp tục theo đuổi đường chọn

Hiện tại, tơi tìm bí để sống vui vẻ học tập, làm việc hiệu xứ người Tôi nghĩ điều quan trọng xác định đam mê, mục tiêu gắn liền với đam mê định hướng để đạt mục tiêu Nhìn vào tương lai, tơi thấy cần bổ sung nhiều hiểu biết trí tuệ nhân tạo, trau dồi kỹ lập trình phát triển kỹ thuật nghiên cứu khoa học thân Tơi khơng thể biết đến đâu, nghĩ đời thú vị

Nhiều người đánh giá bạn người tài đầy cá tính, khơng ngại bộc lộ quan điểm cá nhân Tính cách mạnh mẽ hình thành nhờ yếu tố gì?

Cảm ơn người đánh giá cao Tôi chưa nghĩ người tài năng, cịn cá tính tơi hình thành tơi suy nghĩ hậu lần im lặng Ví dụ, trả lời việc không Singapore gây cho hậu Một lý tơi im lặng, khơng nói lên quan điểm bố mẹ tơi nói tơi phải Mỹ Tơi nghĩ nghe điều tơi nên nói, bố mẹ tơi nghĩ lại giúp tơi có định tốt cho sống Vì vậy, tơi ln phải nói lên nghĩ Như thế, dù tơi có sai, tơi có hội bảo vệ quan điểm tơi nhận quan điểm sai

Phương châm, hay triết lý sống bạn gì? Bạn có lựa chọn câu châm ngơn nào cho riêng mình?

(158)

cho mình, từ định cá nhân vấn đề nghiên cứu, học tập Hiện nay, nghĩ điều tốt nên làm bước lên phía trước Có thể bạn vấp ngã từ sai lầm mình, bạn tìm đường tốt cho thân Vì thế, tơi thích câu Lỗ Tấn “Trên đời làm có đường Người ta thành đường thơi” Tơi thích phiên “văn học mạng” câu nói này, nghe “thơ” hơn: “Cứ Phía trước có đường.”

Dành khơng vinh quang có bạn thấy thất bại hay bế tắc? Bạn đã vượt qua điều nào?

Tơi thất bại nhiều thành công nhiều lần Đối với tơi, thất bại khó chịu, khơng dấu chấm hết Thất bại sửa chữa Tơi làm thí nghiệm huấn luyện cho máy tính học thuật toán mới, để nhận thuật tốn hồn tồn sai máy chẳng học Đó loại thất bại khơng làm q nhiều thời gian để sửa Nó làm tăng thêm kinh nghiệm tôi, cần ghi lại tự nói với sau đừng dùng thuật tốn

Giáo dục gia đình ảnh hưởng nhiều đến tính cách Hy Hiếu nghiên cứu Trong ảnh Hy Hiếu mẹ em trai - Ảnh Tư liệu gia đình

(159)

Tạp chí Epsilon, Số 09, 06/2016

Tôi nghĩ điểm mấu chốt để vượt qua thất bại kiểm sốt cảm xúc thân, tìm điều làm chưa để dẫn đến thất bại sau đừng làm Nói dễ, thân tơi nhiều khơng làm

Cịn cơng việc, mục tiêu tối thượng mà bạn hướng tới gì? Hãy chia sẻ lựa chọn khi đến với chương trình Tiến sĩ Đại học Carnegie Mellon (CMU) Hiếu? Ước mơ mà bạn đang tiếp tục đeo đuổi?

Có nhiều lý khiến CMU mơi trường nghiên cứu lý tưởng Thứ nhất, CMU có nhiều chuyên gia hàng đầu lĩnh vực hẹp xử lý ngôn ngữ tự nhiên Khi lựa chọn trường để làm tiến sĩ, phải nghĩ đến danh tiếng người giáo sư hướng dẫn mà phải nghĩ đến việc vị giáo sư có thời gian dành cho khơng, quan tâm đến đề tài khơng, Tơi thấy CMU có đủ điều Thứ hai, CMU nằm thành phố Pittsburgh, nơi tương đối cách biệt với thành phố lớn khác Mỹ lại không cô lập Tôi hy vọng cách biệt giúp tơi tập trung vào việc nghiên cứu mà khơng bị phân tâm yếu tố bên

(160)(161)

VỀ PHONG TRÀO OLYMPIC TOÁN Ở SAUDI ARABIA

Võ Quốc Bá Cẩn (Archimedes Academy)

Hai năm trở lại đây, chúng tơi có may mắn sang thăm làm việc đội tuyển Toán Saudi Arabia thời gian Những ngày tháng làm việc cho chúng tơi hiểu thêm phong trào học Tốn phổ thông xứ Hồi giáo

Bài viết xin chia sẻ cảm nhận cách thành viên đội tuyển Olympic nước bạn học Tốn Những chia sẻ có phần chủ quan hy vọng bạn đọc có nhìn khái qt phong trào nước bạn, thấy hay, chưa hay để rút kinh nghiệm cho

Trước hết, xin nói qua cách thức tổ chức tuyển chọn bồi dưỡng học sinh giỏi Saudi Arabia Trước đây, Saudi Arabia tổ chức tuyển chọn em học sinh có khiếu Tốn học bồi dưỡng Riyadh Sau đó, trước kỳ thi Quốc tế quan trọng, em làm thi loại, bạn có kết làm tốt chọn vào đội tuyển thi với hy vọng mang vinh quang cho nước nhà

Từ năm 2014 đến nay, Saudi Arabia có chương trình hợp tác huấn luyện với thầy Lê Anh Vinh, khóa học tập trung chuyển trường Đại học Khoa học Công nghệ mang tên nhà vua Abdullah (gọi tắt KAUST) Các học sinh tập huấn trực tiếp với huấn luyện viên từ Việt Nam sang chủ đề liên quan đến Olympic Toán Và ngồi khóa học tập trung này, học sinh rèn luyện trực tuyến nhà với tập thầy Vinh trực tiếp biên soạn đưa lên website hàng tuần

(162)

Tiếp theo, chúng tơi phân tích điểm tốt điểm chưa tốt phong trào học Toán

Điểm tốt:Các em học sinh thân thiện, động, thích đào sâu suy nghĩ vấn đề mà giáo viên giao thường xin thêm thời gian để tìm cách giải tốn giáo viên yêu cầu chữa để tiếp tục giảng

Sự tương tác em với giáo viên thực tốt Chúng bất ngờ buổi giảng mình, em muốn chia sẻ ý kiến vấn đề mà chúng tơi giảng Nhờ mà vấn đề thêm sáng tỏ, lớp học trở nên sôi động hẳn, khiến người thầy trở nên hào hứng giảng

Điều trái ngược với Việt Nam, học sinh thường rụt rè trao đổi với giáo viên Nhiều em tiếp nhận giảng thầy cô cách thụ động mà không dám đặt vấn đề hay đưa ý kiến cá nhân Có lẽ học sinh Việt Nam ta nên thay đổi dần cách học để trở nên tích cực

Các em học sinh level 4+ huấn luyện viên

Điểm chưa tốt:Tương tác tốt lớp vậy, khả tự học học sinh chưa tốt Học sinh lười ghi chép học lớp, nhà thường không xem lại để hiểu rõ vấn đề hơn, em hay quên Sẽ khó để nhớ hiểu lý thuyết khó khơng nghiền ngẫm thường xun Chúng thường “thử” em cách cho lại toán dạy buổi trước vào buổi cách buổi đầu xa chút, thật có em nhận tốn

Một điều đáng nói em khơng tự giác cao nhà, khơng tìm kiếm tài liệu để đọc thêm Cũng đề cập trên, thầy Vinh có đưa lên website tập để học sinh rèn luyện thêm, có em chịu làm nộp (dù điều gần bắt buộc) Rõ ràng phụ thuộc vào điều giáo viên giảng lớp khó có tiến đáng kể

(163)

Tạp chí Epsilon, Số 09, 06/2016

tốn thầy cho mà “dáng dấp” khác Olympic Toán em khơng cố gắng làm, có bạn biện minh không xuất kỳ thi Toán

Điều trái ngược với cách học Toán khoa học phải thấm nhuần ý tưởng lý thuyết Toán học “luyện” Olympic để tiếp cận ý tưởng Một đứng vững phần gốc bị sâu mọt

Tuy nhiên, điều đáng mừng sau hai năm làm việc bạn đội tuyển Saudi, dần tác động cải thiện tư tưởng em Nhiều em bắt đầu ghi chép cẩn thận để xem lại, có em tìm đến diễn đàn Tốn mà chúng tơi giới thiệu để tìm kiếm thêm tư liệu tự trau dồi, rèn luyện

Có điều hạn chế thuộc vấn đề tôn giáo, Saudi Arabia quốc gia Hồi giáo với 100% dân số theo đạo Hồi, mà tín đồ Hồi giáo ngày phải cầu nguyện lần từ sáng sớm tối Do đó, thời gian học học sinh bị ảnh hưởng lớn, gần phần ba thời gian buổi học dành cho việc cầu nguyện

Ngoài ra, tôn giáo nên Saudi, nam nữ huyết thống có quan hệ vợ chồng khơng nói chuyện tự với nhau, khơng ngồi gần Có đội ngũ cảnh sát tơn giáo chun kiểm sốt việc Do đó, nam nữ không học chung với Ở KAUST việc thống chút học viên đa phần du học sinh quốc tế, nam nữ học chung với Nhưng tư tưởng tôn giáo thấm vào em nên học sinh nam học sinh nữ thường không trao đổi ý kiến với nhau, chí có toán em nữ làm được, giáo viên yêu cầu lên bảng trình bày lại cho bạn khác nghe em nam thường khơng tập trung lắng nghe bạn trình bày Điều gây ảnh hưởng nhiều đến hiệu học tập lớp học

Đó điều chúng tơi nhận thấy làm việc đây, nhiên vấn đề thuộc tơn giáo, văn hóa nước bạn nên tơn trọng mà khơng thể góp ý

Mặc dù cịn nhiều điểm chưa tốt, việc chuyển biến theo chiều hướng tích cực Điều thể rõ ràng qua kết đội tuyển Saudi kỳ thi Quốc tế Đội tuyển đạt kết tốt so với năm trước, chí có huy chương vàng BMO, JBMO, APMO, điều chưa có năm trước

(164)(165)

BÀI TOÁN HAY LỜI GIẢI ĐẸP

Trần Nam Dũng

(Đại học Khoa học Tự nhiên - ĐHQG TP.HCM)

LỜI GIỚI THIỆU

Chuyên mục lấy cảm hứng từ viết thầy Nguyễn Duy Liên số báo trước toán số6trong kỳ thi IMO2001với5cách giải khác Mục để dành viết toán hay, lời giải đẹp câu chuyện thú vị xung quanh tốn lời giải

Tên chun mục mượn từ tên nhóm người yêu toán Facebook anh Nguyễn Văn Lợi sáng lập “Bài toán hay – Lời giải đẹp – Đam mê toán học” Chuyên mục ghi nhận đề cử bạn đọc chọn đăng kỳ1,

bài tốn

Số chúng tơi giới thiệu với bạn đọc toán số6trong đề thi toán quốc tế năm1995với lời giải sau đưa vào sách giáo khoa phương pháp giải toán

Bài toán.(IMO 1995, toán 6) Choplà số nguyên tố lẻ Có tập conAgồmpphần tử tập hợp{1, 2, , 2p}có tổng phần tử chia hết chop?

Chứng minh. Đặt X = {1, 2, , p}, Y = {p+ 1, p+ 2, , 2p}.Với tập A ⊂ X ta định nghĩa phép tịnh tiến theo sốklàk+A={y ∈X :y ≡k+x (mod p),vớix ∈Anào đó}.Ta định nghĩa điều tương tự trongY

Bây ta ý P ⊂ {1, 2, , 2p} |P| = p P = X, P = Y <

|P ∩X|,|P ∩Y|

Tiếp theo ta xétP 6=X, Y định nghĩa

k+P = (k+ (P ∩X))∪(P ∩Y), k ∈X

Chú ý vì0<|P ∩X|< pnên tổng phần tử củak+P lấy thặng dư modulo

p,trong có thặng dư thỏa mãn tốn Ta định nghĩa quan hệ tương đương tập hợpP ⊂ {1, 2, , 2p}vớipphần tử,P 6=X, Y sau:

P ∼Q⇔(P ∩Y =Q∩Y & k+P ∩X =Q∩X, k∈X)

Quan hệ tương đương xác định phân hoạch họ tập conpphần tử của{1,2, , 2p}

khác vớiXvàY.Mỗi lớp tương đương phân hoạch cópphần tử từ lớp có phần tử thỏa mãn điều kiện toán

Suy đáp số toán C

p

2p−2

(166)

Chứng minh. Gọiωlà nguyên thủy bậcpcủa đơn vị Khi

2p

Y

i=1

(x−ωi) = (xp −1)2 =x2p−2xp+

So sánh hệ số củaxp ta có

2 =Xωi1+i2+···+ip =

p−1

X

j=0

njωj

Trong tổng thứ tính theo tất tập conpphần tử{i1, i2, , ip}của{1, 2, , 2p}

vànj tổng thứ hai số tập cho

i1+i2+· · ·+in ≡j (mod p)

Từ suy rằngωlà nghiệm

G(x) = (n0−2) + p−1

X

j=1

njxj,

là đa thức có bậc p−1.Vì đa thức tối tiểu ω trường số hữu tỷ F(x) = +x+· · ·+xp−1,cũng có bậcp−1.Từ suy raG(x)phải có dạngk·F(x)vớiklà số Như thến0−2 = n1 =n2 =· · ·=np−1.Vì

n0+n1+· · ·+np−1 =C2pp,

nên từ ta suy ran0 = C2pp−2

p +

Lời giải thứ nhất, sử dụng phân hoạch song ánh, thuộc tác giả tốn, Marcin Kuczma, trưởng đồn Ba Lan Lời giải thứ hai, sử dụng số phức, cụ thể đơn vị, thuộc Roberto Dvornicich, trưởng đoàn Ý Nikolay Nikolov, học sinh người Bulgaria trao giải đặc biệt cho lời giải mình, giống với lời giải thứ hai trình bày đọng nữa, trong2

(167)

ĐỒNG NHẤT THỨC BRAHMAGUPTA -FIBONACCI VÀ ỨNG DỤNG

Trần Nam Dũng

(Đại học Khoa học Tự nhiên - ĐHQG TP.HCM)

Một học sinh chuyển từ lớp7lên lớp8ắt yêu cầu chứng minh đẳng thức:

(a2+b2)(c2+d2) = (ac+bd)2+ (ad−bc)2,

và chẳng gặp khó khăn Bạn không ngờ ngày xưa, để chứng minh nó, nhà tốn học tiếng Fibonacci phải đến5trang giấy Và không ngờ rằng, đẳng thức này, mà dạng tổng quát có tên gọi đồng thức Brahmagupta - Fibonacci, có nhiều ứng dụng đẹp đẽ, hiệu bất ngờ Chúng ta vào chuyến tham quan đến với lịch sử đồng thức đẹp đẽ này, ứng dụng kinh điển toán cịn nóng hổi Điều đặc biệt bạn học sinh THCS hiểu đa số tốn ví dụ viết

1 Đơi dịng lịch sử

Phần dẫn dắt lịch sử chủ yếu lấy từ [4]

Đồng thức mà ngày thường gọi làđồng thức Brahmagupta - Fibonaccirất đơn giản có lịch sử thú vị có ứng dụng bất ngờ

(a2+b2)(c2+d2) = (ac+bd)2+ (ad−bc)2 = (ac−bd)2 + (ad+bc)2

Người ta tin người nhắc đến đồng thức Diophantus (thế kỷ thứ III trước công nguyên) người viết Số học (Quyển III, Bài tốn19) [Stillwell, trang174]: “65một cách tự nhiên chia thành hai bình phương hai cách vì65 tích của135,mỗi số tổng hai bình phương

John Stillwell đề cập năm 950 sau cơng ngun, nhà tốn học vùng Ba Tư al-Khazin (900−971SCN) giải thích khẳng định Diophantus dạng tổng quát gần giống với cơng thức nêu Một giải thích tương tự xuất sách FibonacciLiber Quadratorumvào năm1225.Lúc ngơn ngữ tốn học cịn vào thời thơ ấu nên Fibonacci phải mất5trang giấy để chứng minh công thức

Ý tưởng Diophantus tổng qt hóa trước nhà tốn học Ấn Độ Brahmagupta (597−668SCN), người chứng minh

(a2+N b2)(c2+N d2) = (ac+bd)2+N(ad−bc)2 = (ac−bd)2+N(ad+bc)2

Và sử dụng để giải phương trình Pellx2−N y2 = 1.

Cơng thức, dạng trường hợp riêng (khi n = 2) đồng thức Lagrange

n

X

i=1

a2i ! n

X

i=1

b2i !

= n

X

i=1

aibi

!2

+ X

16i<j6n

(akbj−ajbk)

(168)

Đúng với mọin >2,ví dụ, vớin= 3có dạng sau

(a2+b2+c2)(x2+y2+z2) = (ax+by+cz)2+ (ay−bx)2+ (az−cx)2+ (bz−cy)2,

và sử dụng để chứng minh tính chất điểm đối trung

Tuy nhiên, đồng thức Brahmagupta - Fibonacci (n = 2) sở hữu tính chất đặc biệt khơng với đồng thức Lagrange, trường hợpn= 3của Đó cách mà Diophantus phát biểu tổng qt hóa tốn 19 ơng:

Tích hai số ngun, số tổng hai bình phương, tổng hai bình phương.

Ta khơng thể nói điều cho tổng ba bình phương Nhưng vào năm1748,Euler thơng báo đồng thức bốn bình phương:

(a21+a22+a23+a24)(b21+b22+b23 +b24) = (a1b1−a2b2−a3b3−a4b4) + (a1b2+a2b1+a3b4−a4b3)

2

+ (a1b3 −a2b4+a3b1+a4b2)

+ (a1b4+a2b3−a3b2+a4b1)

Trường hợp n = liên quan đến lý thuyết số phức mà diễn đạt “chuẩn (mô-đun, giá trị tuyệt đối) hai số phức tích chuẩn chúng.” Vai trị tương tự dành cho đồng thức bốn bình phương lý thuyết quaternion đồng thức tám bình phương Degen lý thuyết octonion [Conway and Smith]

Bài tập Hãy chứng minh đồng thức Brahmangupta - Fibonacci

(a2+N b2)(c2+N d2) = (ac+bd)2+N(ad−bc)2 = (ac−bd)2+N(ad+bc)2

2 Ứng dụng toán biểu diễn số nguyên

Bài toán biểu diễn số nguyên tốn tìm điều kiện để số ngun biểu diễn dạng cho trước, ví dụ dạng tổng hai bình phương (số0, 1, 2, 4, 5biểu diễn nhưng3thì khơng), tổng ba bình phương (số0,1, 2, 3, 4, 5, 6biểu diễn được,

7thì khơng) Định lý Langrange lại nói số tự nhiên biểu diễn dạng tổng bốn bình phương

Tuy nhiên, tạm thời không xa vậy, kết nêu kết kinh điển sức học sinh THCS, đối tượng viết Ta vào số vấn đề cụ thể có giới hạn

Bài tốn Ta nói số nguyên dươngnlà biểu diễn tồn số nguyêna, bsao cho

n =a2+b2

a) Chứng minh nếum, nlà biểu diễn thìmncũng biểu diễn được.

b) Chứng minh nếu2mbiểu diễn thìmcũng biểu diễn được.

(169)

Tạp chí Epsilon, Số 09, 06/2016

Chứng minh. a)Kết luận kết mà Diophantus phát biểu chứng minh trực tiếp nhờ đồng thức Brahmagupta - Fibonacci

b)Giả sử2m=a2+b2thìa, bcùng tính chẵn lẻ, suy raa+bvàa−blà số chẵn ta có

m=

a+b

2

2

+

a−b

2

2 ,

từ suy điều phải chứng minh

Đẳng thức đồng thức Brahmagupta - Fibonacci áp dụng cho 12 = 122 + 122

và2m=a2+b2.Ta lưu ý thủ thuật nhỏ hữu ích cho tình sau.

c)Ta có số phương chia3dư0hoặc1và chia4dư0hoặc1.Do nếua2+b2 chia hết cho3thìavàbđều phải chia hết cho3,do đóa2+b2 chia hết cho9.Cũng từ số có dạng 4k+ 3khơng thể biểu diễn dạng tổng hai bình phương Áp dụng tính chất ta suy số sau không biểu diễn được:

Số2013(chia hết cho3mà khơng chia hết cho9), 2015(có dạng4k+ 3)

Số2014 = 2·1007khơng biểu diễn do1007có dạng4k+ 3khơng biểu diễn (áp dụng tính chấtb)) Số2016 = 27·63khơng biểu diễn do63có dạng4k+ 3khơng biểu

diễn (áp dụng nhiều lần tính chấtb)

Số 2017 biểu diễn (ta dùng thuật toán ăn tham để thử từ xuống tìm

2017 = 442+ 92).

Bài toán Chomnlà số nguyên dương phân biệt Hãy biểu diễnm6+n6 dưới dạng tổng hai bình phương khác vớim6n6.

Chứng minh. Ta có

m6+n6 = (m2)3+ (n2)3 = (m2+n2)(m4−m2n2+n4) = (m2+n2)h(m2−n2)2+ (mn)2i

Bây giời áp dụng đồng thức Brahmagupta - Fibonacci choa=m, b=n, c=m2−n2

d=mnta thu

m6 +n6 = (m3−mn2+mn2)2+ (m2n−m2n+n3)2

Đây điều mà ta mong muốn biểu diễn trùng với biểu diễn ban đầu Nhưng khơng có mát ta để ý đẳng thức Brahmagupta - Fibonacci có dạng khác chút ta thaydbằng−d

(a2+b2)(c2+d2) = (ac−bd)2+ (ad+bc)2

Lúc ta viết

m6+n6 = (m3−mn2−mn2)2+ (−m2n−m2n+n3)2 = (m3−2mn2)2+ (n3−2m2n)2

(170)

Ta thấy yêu cầumvànnguyên dương bỏ qua: Đồng thức mà ta thu đồng thức đại số với biến số (hay ẩn số)mvàn

Bài tốn đề thi vơ địch tốn Moscow đặt vấn đề khó khăn chút:

Bài toán (Moscow MO) Chứng minh với mọin >3,tồn số lẻx, y sao cho 7x2+y2 = 2n

Chứng minh. Nếu khơng có điều kiệnx, y lẻ giải đơn giản nhờ vào quy nạp Chú ý vớin = 3ta có biểu diễn 7·12 + 12 = 23 và với n = 4thì 7·12 + 32 = 24 và nếu 7x2+y2 = 2nthì7(2x)2+ (2y)2 = 2n+2.Tuy nhiên, với điều kiệnx, ylẻ bước chuyển từn

lênn+ 2khơng dùng Ta dùng đến giải pháp gần giống với giải pháp 1, với ý

7· 2 + 2

=

Từ đó, giả sử2n = 7x2+y2vớix, ylẻ, áp dụng đồng thức Brahmagupta - Fibonacci ta có

2n+1 = 2·2n =

" 7· 2 + 2#

(7x2+y2)

=

7x+y

2

2

+

x−y

2

2

=

7x−y

2

2

+

x+y

2

2

Do x, y lẻ nên số ngoặc nguyên Tuy nhiên, theo yêu cầu đề bài, ta cần chúng lẻ Đến ta cần xét2trường hợp

• Nếux, ycùng số dư mod 4thì ta chọn biểu diễn thứ2

• Nếux, ykhác số dư mod 4thì ta chọn biểu diễn thứ1

Trong trường hợp phương trình2n+1 = 7x2+y2 có nghiệmx, y lẻ Theo ngun lý quy

nạp tốn học ta có điều phải chứng minh

Bài tập (Saudi Arabia TST 2016, Juniors) Choklà số nguyên dương Chứng minh tồn tại số nguyênx, y không số chia hết cho3sao chox2+ 2y2 = 3k

Bài tập (VMO 2010) Chứng minh với số nguyên dươngn,phương trình

x2+ 15y2 = 4n,

có nhấtnnghiệm ngun khơng âm.

Bài tập (Saudi Arabia Training Camp 2016) ChoSlà tập hợp tất số tự nhiên biểu diễn được dạngx2+ 3y2vớix, y là số nguyên Chứng minh tính chất sau củaS

i) Nếum∈S, n∈Sthìmn∈S

ii) NếuN ∈S2|N thì N4 ∈S

(171)

Tạp chí Epsilon, Số 09, 06/2016

3 Ứng dụng phương trình Pell

Theo số nhà nghiên cứu lịch sử tốn học phương trình Pell, tức phương trình nghiệm nguyên dạng

x2−N y2 = 1, (3.1)

vớiN số nguyên dương khơng phương đáng phải gọi phương trình Brah-magupta - Bhaskara Tuy nhiên khơng sâu khía cạnh lịch sử để phân tích xem hai ơng tiến xa việc giải phương trình (3.1) mà ứng dụng đồng thức Brahmagupta-Fibonacci việc giải phương trình Pell phương trình dạng Pell

Bài tốn Chứng minh phương trình

x2 −2y2 = 1, (3.2)

có vơ số nghiệm nguyên không âm.

Chứng minh. Ta thấy a2 −2b2 = 1 và x2 −2y2 = 1 thì áp dụng đồng thức

Brahmagupta - Fibonacci ta có

1 = (a2−2b2)(x2−2y2) = (ax+ 2by)2−2(ay+bx)2 (3.3) Từ dễ dàng suy (3.2) có nghiệm (3.2) có vơ số nghiệm Dễ thấyx= 3, y = 2là nghiệm (3.2) (3.2) có vơ số nghiệm Hơn nữa, áp dụng (3.3) choa= 3, b= 2và

x, y nghiệm nguyên không âm (3.2) ta thấy: Nếu(x, y)là nghiệm (3.2)

(3x+ 4y,2x+ 3y)cũng nghiệm (3.2)

Ví dụ từ cặp(3,2)theo cách ta có nghiệm(17,12),(99,70),(577,408),

Điều thú vị cách xây dựng vét hết tất nghiệm ngun khơng âm phương trình (3.2) (tất nhiên phải bổ sung nghiệm hiển nhiênx = 1, y = 0) Tuy nhiên không chứng minh kiện

Cũng từ cách chứng minh trên, ta thấy phương trình (3.1) có nghiệm ngun dương có vơ số nghiệm ngun dương Hơn nữa, (3.1) có nghiệm ngun dương phương trình

x2−N y2 =a, (3.4)

có nghiệm ngun dương (3.4) có vơ số nghiệm ngun dương

Người ta chứng minh nếuN số nguyên dương khơng phương trình phương trình (3.1) ln có nghiệm nguyên dương, theo lập luận trên, có vơ số nghiệm ngun dương Hơn nữa, ta tìm tất nghiệm Kết trọn vẹn thuộc Langrange(1767−1838)dựa vào lý thuyết liên phân số Tuy nhiên, ta không sâu mà xem xét ví dụ áp dụng tồn vơ số nghiệm phương trình dạng Pell

(172)

Chứng minh. Mấu chốt lời giải nhận xét

n2+k|n4 ⇔n2+k|n4−k2+k2 ⇔n2+k|k2

Tiếp theo, sử dụng điều kiệnkthuộc[n2+ 1, n2+ 2n]dễ dàng suy ra k2

n2+k ∈ {2,3}

Cuối ta chứng minh phương trìnhk2 = 2(n2+k), k2 = 3(n2+k)khơng có nghiệm

ncó dạng7m+ 3,7m+ 4(dùng mơ-đun) có vơ số nghiệm dạng7k,7k+ 1,7k+ 2,7k+

(phương trình thứ nhất) dạng7k,7k+ 6(phương trình thứ hai) cách sử dụng công thức Brahmagupta - Fibonacci

Bài tập (VMO 1999) Cho hai dãy số(xn), (yn)xác định sau

x1 = 1, x2 = 4, xn+2 = 3xn+1−xn, với mọin >1,

y1 = 1, y2 = 2, yn+2 = 3yn+1−yn, với mọin>1

Chứng minh số nguyên dươnga, bthỏa mãn phương trìnha2 −5b2 =−4khi chỉ khi tồn số nguyên dươngksao choa=xk, b=yk

4 Một số ứng dụng khác

Đồng thức Lagrange, trường hợp tổng quát đồng thức Brahmagupta - Fibonacci cho ta hệ bất đẳng thức Cauchy - Schwarz, bất đẳng thức cổ điển có nhiều ứng dụng quan trọng

Bất đẳng thức Cauchy-Schwarz trường hợpn = 2:Nếua, b, c, dlà số thực ta có bất đẳng thức

(a2+b2)(c2+d2)>(ac+bd)2

Dấu xảy khia:b=c:d

Cũng theo hướng bất đẳng thức, thiên biểu diễn, định lý sau kết tương tự với kết lý thuyết số (về điều kiện cần đủ để số nguyên dương biểu diễn dạng tổng bình phương hai số nguyên) dành cho đa thức Hóa điều kiện cần đủ để đa thức với hệ số thực biểu diễn dạng tổng bình phương hai đa thức với hệ số thực làP(x)>0với mọix.Tuy nhiên, kết chứng minh đơn giản kết lý thuyết số nhiều Chứng minh thực tế dùng đến đồng thức Brahmagupta -Fibonacci

Bài toán Cho P(x)là đa thức với hệ số thực choP(x) >0với mọixthuộc R.Chứng minh tồn đa thứcQ1(x)Q2(x)với hệ số thực cho

P(x) = Q21(x) +Q22(x)

Chứng minh. VìP(x)khơng âm nên hoặcP(x)khơng có nghiệm thực, nghiệm thực có bội chẵn Từ suy

P(x) = c

n

Y

k=1

(x2 +pkx+qk),

(173)

Tạp chí Epsilon, Số 09, 06/2016

Nhưng với mỗik,phương pháp gom bình phương chứng tỏ

x2+pkx+qk=

x+pk

2

2

+

p

4qk−p2k

!2 ,

do

P(x) = c

n

Y

k=1

(Uk2(x) +Vk2(x))

Bây giờ, áp dụng đồng thức Brahmagupta - Fibonacci nhiều lần, ta biến số thừa số từntrở về1

Cuối cùng, ta nêu ứng dụng thú vị đồng thức bốn bình phương Euler

Bài tốn Chứng minh rằng:

a) Phương trìnhx2+y2+z2 = 7khơng có nghiệm hữu tỷ. b) Phương trình sau khơng có nghiệm hữu tỷ

x2+y2+z2+t2 = 7, a2+b2+c2+d2 = 1, ax+by+cz+d=

Chứng minh. a) Điều cần chứng minh tương đương với “chứng minh không tồn nghiệm

(x, y, z, t)khác(0,0,0,0)của phương trình

x2 +y2+z2 = 7t2 (4.1)

Giả sử tồn nghiệm nguyên không âm không đồng thời bằng0.Chọnx, y, z, tlà nghiệm (4.1) cóx+y+z+tnhỏ

Nếutlẻ vế trái≡7 (mod 8)trong khix2 +y2+z2 chỉ có số dư là0,1,2,3,4,5,6 (mod 8), mâu thuẫn Vậy t chẵn, suy vế phải đồng dư (mod 8) Suy x, y, z

phải chẵn Nhưng lúc x2,y2,z2,2t nghiệm (4.1) mâu thuẫn với cách chọn

x, y, z, t.Vậy điều giả sử sai, suy điều phải chứng minh

b)Giả sử hệ có nghiệm hữu tỷ từ đồng thức Euler ta suy

7 = (a2 +b2+c2+d2)(x2 +y2 +z2+t2)

= (ax+by+cz+dt)2+ (ay−bx+ct−dz)2 + (az −bt−cx+dy)2+ (at+bz−cy−dx)2 = (ay−bx+ct−dz)2+ (az −bt−cx+dy)2+ (at+bz−cy−dx)2

Suy phương trìnhX2+Y2+Z2= có nghiệm hữu tỷ, mâu thuẫn với a).

Tài liệu tham khảo

[1] A Shenitzer, J Stillwell,Mathematical Evolutions, MAA,2002

[2] J H Conway, D A Smith,On Quoternions and Octonions, A K Peters,2003

[3] S Savchev, T Andreescu,Mathematical Miniatures, MAA,2003, p.6−7

[4] Alexander Bogomolny:

(174)(175)

MỘT SỐ BÀI TOÁN TRONG ĐỀ THI VÀO CÁC TRƯỜNG CHUYÊN

(Ban biên tập)

Trong viết này, giới thiệu số tốn trích đề thi vào trường Chuyên toàn Quốc vừa diễn cuối tháng đầu tháng

Bài 0.1 (Trích đề thi vào 10 Chuyên Phan Bội Châu-Nghệ an)

Giải phương trình

5−3x+√x+ =√3x2−4x+ 4. Lời giải Điều kiện:−16x6

3

Bình phương hai vế phương trình ta

2p(5−3x) (x+ 1) = 3x2−2x−2

Đặtt =p(5−3x) (x+ 1) =√−3x2+ 2x+ 5, t>0ta có phương trình

t2 + 2t−3 = 0⇔t = (dot >0)

Suy

−3x2 + 2x+ = 1⇔3x2 −

2x−4 = 0⇔x= 1± √

13

6

Kết hợp điều kiện ta có:x= 1± √

13

6 nghiệm phương trình Bài 0.2 (Trích đề thi vào 10 Chun Tốn PTNK)

Tìma>1để phương trình

ax2+ (1−2a)x+ 1−a= có hai nghiệm phân biệtx1, x2 thỏa mãn

x22−ax1 =a2−a−1 (1) Lời giải Với tốn liên quan đến nghiệm phương trình bậc hai cơng cụ ta thường sử dụng định lí Vi-et Với điều kiện x2

2 −ax1 = a2 −a −1thì ta kết hợp với điều kiện

ax22+ (1−2a)x2+ 1−a= 0(2) để xử lí Dựa vào định lí Vi-et, ta thayx1 = 2aa−1 −x2

để biến đổi (1) đẳng thức chưax2vàa.Cụ thể:

x22−ax1 =a2−a−1 ⇔x22−a

2a−1

a −x2

(176)

Kết hợp (2) với (3) ta

a2+ 2a−1x2 =a3 +a2−3a+ = (a−1) a2+ 2a−1

⇔x2 =a−1

(doa >1nêna2+ 2a−1>0)

Thayx2 =a−1vào (2) ta tìm đượca= 1, a=

Bài 0.3 (Trích đề thi vào Trường Chuyên Lê Qúy Đơn Khánh Hịa)

Giải hệ phương trình

(x2−xy)(xy−y2) = 25 (1)

p

x2−xy+pxy−y2 = 3(x−y) (2)

Lời giải Từ phương trình (2) ta cóx−y >0nênx, y > Hơn x= ykhơng nghiệm hệ, (2) tương đương với

x+√y= 3√x−y

⇔x+y+ 2√xy= 9(x−y) ⇔4x−√xy−5y=

⇔(√x+√y)(4√x−5√y) = ⇔y= 16

25x(dox=y= 0không nghiệm)

Thay vào (1) ta tìm nghiệm hệ:(x;y) =

25 ;

8

Bài 0.4 (Trích đề thi vào Trường Chuyên Lê Qúy Đôn Vũng Tàu)

Cho đa thứcf(x) = x2+bx+c.Biếtb, clà hệ số dương vàf(x)có nghiệm Chứng minh

f(2)>9√3c.

Lời giải Từ giả thiếtf(x)có nghiệm ta có:

∆ =b2−4c>0⇒b2 >4c⇔b>2√c

Vàf(2) = 2b+c+ 4>4√c+c+ Do đó, để chứng minh toán ta cần chứng minh được:

c+ 4√c+ 4>9√3

c (1)

Để chứng minh (1) ta sử dụng bất đẳng thức Cơ si biến đổi tương đương Chẳng hạn ta dụng bất đẳng thức Cô si sau

c+ 4√c+ = √c+√c+

+ (c+ + 1)>2.3√3

c+ 3√3

c= 9√3 c

Bài 0.5 (Trích đề thi vào 10 Trường Chuyên Lý Tự Trọng - Cần Thơ )

Choa, b, clà độ dài ba cạnh tam giác thỏa mãn2ab+ 3bc+ 4ca= 5abc Tìm GTNN của biểu thức

P =

a+b−c+

6

b+c−a +

5

(177)

Tạp chí Epsilon, Số 09, 06/2016

Lời giải Từ giả thiết ta có

a+

4

b+

2

c = Nhận thấy(a+b−c)+(b+c−a) = 2b,(b+c−a)+

(c+a−b) = 2c, (c+a−b) + (a+b−c) = 2avà7 = + 3, = + 2, = + 2nên ta phân tích

P =

1

a+b−c +

1

b+c−a

+3

1

a+b−c+

1

c+a−b

+2

1

b+c−a +

1

c+a−b

Áp dụng bất đẳng thức

x +

1

y >

4

x+y ∀x, y >0,ta có

P > 16

2b +

12 2a +

8 2c =

a + b + c

= 10

Đẳng thức xảy khia =b =c=

5 ⇔x=y=z =

Vậy GTNN củaP bằng10

Bài 0.6 (Trích đề thi vào Trường Chuyên Lê Hồng Phong, Nam Định)

Chox, y, z là số thực thỏa mãn(x−y)(x−z) = 1y6=z Chứng minh:

(x−y)2 + (y−z)2 +

1

(z−x)2 >4 Lời giải Ta có

(x−y) + (y−z) + (z−x) = 0,

nên đặta=x−y, b=x−zta cóy−z =a−bvàab= Khi

P =

a2 +

b2 + (a−b)2 =ab

1

a2 +

b2 +

1

a2+b2−2ab

= a b + b a + 1 b + b a −2

=t+

t−2 (vớit=

a b +

b a)

= (t−3)

t−2 + >4

Đẳng thức xảy chẳng hạn

  

 

y=x− √

5−1

z =x− √

5 +

Vậy GTNN củaP bằng4

Bài 0.7 (Trích đề thi vào Trường Chun KHTN Vịng 2)

1) Vớix, ylà số nguyên thỏa mãn đẳng thức x 2−1

2 =

y2−1

3 Chứng minh rằng:

x2−y2 40

(178)

Lời giải 1) Đề yêu cầu chứng minhx2−y2 40 = 8.5nên ta cần chứng minhx2−y2 5và

x2−y2 .8.

Ta biết: Nếux, y lẻ thìx2−y2 8nên ta chứng minhx, y lẻ Sử dụng tính chất tỉ lệ thức ta có:

x2−1

2 =

y2−1 =y

2−

x2

nên x

2−1

y2−1

3 số nguyên

Do đóxlà số lẻ, suy x

2−1

2 số chẵn

y2 −1

3 số chẵn đóy

2 là số lẻ.

Suy rax2−y2 .8.

Để chứng minhx2−y2 .5ta chứngx2vày2có số dư chia cho5.

Từ giả thiết ta suy x

2−1

2 =

y2−1

3 =

x2+y2−2

5 , từ ta cóx

2+y2−2 .5.

Doa2 ≡0,1,−1 (mod5)nên ta có

x2 ≡y2 ≡1 (mod5)⇒x2−y2

2) Ta có

x4 + 2x2 =y3 ⇔ x2+ 12 = (y+ 1) y2−y+

Gọi

d= y+ 1, y2−y+ 1= (y+ 1,(y+ 1)(y−2) + 3) = (y+ 1,3)⇒d|3

Dod|x2 + 1và không tồn tạixđểx2 + 1 .3nên ta cód= 1.

Từ suy

y+ =m2

y2−y+ =n2

Mặt khác, ta cóy+ >0⇒y>0 +) Vớiy = 0ta cóx=

+) Xéty >0, ta có

m2 −22 =y2−2y+ < y2−y+ =n2 <(y+ 1)2 =m4

Suy ran=m2−1 = y⇒y2−y+ =y2 ⇒y = 1(không thỏa).

Vậyx=y= 0là cặp nghiệm cần tìm

Bài 0.8 (Trích đề thi vào Trường Chuyên Lê Hồng Phong Nam Định)Trên bảng ban đầu ta ghi số2và số4.Ta thực cách viết thêm số lên bảng sau: bảng có hai số, giả sử làa, b;vớia6=b,ta viết thêm lên bảng số có giá trị làa+b+ab.Hỏi với cách thực hiện vậy, bảng xuất số2016được hay khơng? Giải thích.

Lời giải Ta cóa+b+ab+ = (a+ 1) (b+ 1), số ghi bảng ngồi hai số2và4

(179)

Tạp chí Epsilon, Số 09, 06/2016

Bài 0.9 (Trích đề thi vào Trường Chuyên Toán PTNK)

Với số nguyên dươngm >1, kí hiệus(m)là ước nguyên dương lớn củamvà khácm. Cho số tự nhiênn >1, đặtn0 =nvà tính số

n1 =n0−s(n0), n2 =n1−s(n1), , ni+1=ni−s(ni), Chứng minh tồn số nguyên dươngkđểnk= 1và tínhkkhin = 216.1417.

Lời giải Ta thấy dãyn0, n1, dãy giảm, đến lúc giá trị dãy

Nên để chứng minh ý thứ tốn ta dùng phương pháp phản chứng

Nếu khơng tồn tạik đểnk= 1thì dãy sốn1, n2, , ni+1, dãy giảm chứa vô hạn số

nguyên dương không vượt quán Điều vơ lí Suy tồn tạikđểnk =

Ta phân tíchn = 216.1417 = 233.717

Bằng cách thử số trường hợp đặc biệt ta rút nhận xét sau:

NX 1:Vớin0 = 2tthìn1 =t Suy ra: Nếun0 = 2α thìnt = 2α−tnênnα =

NX 2:Nếun0 = 3t,(t,2) = 1thìn2 =t

NX 3:Nếun0 = 7t, tkhơng có ước ngun tố nhỏ hơn7thìn4 =t Suy ran = 7αthìn4α =

Từ nhận xét ta có:

Vớin= 233.717thìn33= 717nênn4.17+33 = 1hayk = 4.17 + 33 Bài 0.10 (Trích đề thi vào Trường Chuyên KHTN Vòng 2)

Cho tập hợpA={1,2,3, , n} Chứng minh tồn hoán vị A

B ={a1, a2, a3, , an} sao cho với mọi16i < j < k6nthìai+ak6= 2aj.

Lời giải Ta chứng minh toán quy nạp kết mạnh :

Cho tập hợpA={1,2, , n} Chứng minh tồn hoán vị A làB ={a1, a2, , an} được xếp đường trịn cho trung bình cộng hai số khơng đặt nằm giữa hai số đó.

Rõ ràng yêu cầu16i < j < k6nthìai+ak 6= 2aj thể tính chất trung bình cộng

hai số khơng nằm hai số trường hợp xét số đặt đường thẳng ; toán vừa phát biểu rõ ràng mạnh

Ta chứng minh với mọin= 2m, m = 0,1,2, thì xếp số{1,2,3, , n}lên

đường tròn cho điều kiện thoả mãn quy nạp theom - Vớim = 0, hiển nhiên kết

- Giả sử(a1, a2, a3, , a2m)là cách xếp thoả điều kiện Xét số

1,2,3, ,2m+1 Ta có cách xếp sau

(b1, b2, , b2m, , b2m+1) = (2a1−1,2a2−1, ,2a2m −1,2a1, ,2a2m)

cũng thoả điều kiện

Thậy vậy, xét hai sốbi, bj Nếubi, bj tính chẵn lẻ theo giả thiết quy nạp, trung

bình cộng củabi, bj khơng nằm chúng Cịn nếubi, bj khác tính chẵn lẻ trung bình

cộng chúng khơng phải số ngun tất nhiên không thuộc dãy cho

Do đó, trường hợpn = 2m+1, ta điền thỏa mãn đề nên theo nguyên lí quy nạp

thì xếp với số tự nhiênncó dạngn= 2m

Rõ ràng điền số với sốathì vớib < a, ta xóa bớt số số quy tắc đề thỏa mãn

(180)

Bài 0.11 (Trích đề thi vào 10 Chun Tốn khu vực TP Hà nội)

Cho2017số hữu tỉ dương viết đường tròn Chứng minh tồn hai số cạnh nhau được viết đường tròn cho bỏ hai số thì2015số cịn lại khơng thể chia thành hai nhóm cho tổng số nhóm nhau.

Lời giải Ta thấy bộx1, x2, , x2017 thỏa u cầu tốn bộkx1, kx2, , kx2017

thỏa yêu cầu tốn vớiklà số hữu tỉ dương Do đó, ta cần chứng minh toán với

2017số ngun dương có số lẻ (vì số chẵn ta chia số cho2)

+) Nếu2017số lẻ ta bỏ hai số liên tiếp và2015số cịn lại khơng thể chia thành hai nhóm có tổng phần tử (vì tổng của2015số số lẻ)

+) Nếu trong2017có số chẵn tồn hai số liên tiếp khác tính chẵn lẻ hai số liên tiếp tính chẵn lẻ Do đó, tổng của2017số cho số chẵn ta xóa hai số liên tiếp mà khác tính chẵn lẻ, tổng số lẻ ta xóa hai số liên tiếp tính chẵn lẻ Khi tổng của2015số cịn lại số lẻ, nên khơng thể chia thành hai nhóm có tổng

Bài 0.12 (THPT chuyên KHTN 2016 vòng 1) Cho tam giácABC nhọn nội tiếp đường tròn (O)vớiAB < AC Phân giác của∠BAC cắtBCtạiDvà cắt(O)tạiEkhácA.M là trung điểm đoạn thẳngAD Đường thẳngBM cắt(O)tạiP khácB Giả sử đường thẳng

EP AC cắt tạiN.

1) Chứng minh tứ giácAP N M nội tiếp vàN là trung điểm củaAC.

2) Giả sử đường tròn(K)ngoại tiếp tam giácEM N cắt đường thẳngAC tạiQkhácN Chứng minh rằngB Qđối xứng quaAE.

3) Giả sử(K)cắt đường thẳngBM tạiRkhácM Chứng minh rằngRA⊥RC.

A

B C

O

D

E M

P

N Q R

Hình 20.1

(181)

Tạp chí Epsilon, Số 09, 06/2016

Do P ∈ (O)nên∠AP B = ∠ACB Mặt khác,∠AP M = ∠AN M (do tứ giácAP N M nội tiếp) Suy ra∠ACB =∠AN M Do ta đượcM N kBC MàM lại trung điểm củaAD

nên theo tính chất đường trung bình suy raN trung điểm củaAC

2) DoEM N QvàAP N M tứ giác nội tiếp nên∠EQC=∠EM N =∠AP E = 180◦− ∠ABE Suy ra∠AQE =∠ABE, kết hợp với∠QAE =∠BAE ta được∠AEB = ∠AEQ Ta được4AEB=4AEQ(g.c.g) Từ suy raB, Qđối xứng quaAE

3) DoERM N vàAP N M tứ giác nội tiếp nên∠ERB =∠M N E =∠EAP =∠EBR Suy tam giácEBRcân tạiE, kết hợp vớiEB =EC (doElà trung điểm cungBC không chứaA), ta đượcER = EC Mặt khác ta có∠REN =∠P M N =∠P AC =∠N EC Từ suy ra4REN =4CEN (c.g.c), đóN R =N C =N A Suy ra∠ARC = 90◦hay

RA⊥RC

Nhận xét Đây toán nằm đề chung cho thí sinh, ý 1) nhẹ nhàng ý 2) khó chút ý 3) có tính phân loại Cấu trúc đề chặt chẽ với câu, ý 2) dùng ý 1) và ý 3) dùng ý 1) 2) Kết nằm tốn có ý nghĩa thu gọn lại đề bài sau

Bài 0.13 (THPT chuyên KHTN 2016 vòng 2) Cho tam giácABC nội tiếp đường tròn(O) Phân giác ADcắt (O)tạiE khácA.N là trung điểm AC Đường tròn ngoại tiếp tam giác

EM N cắtM BtạiRkhácM Chứng minh rằngRA⊥RC.

Bài 0.14 (THPT chun KHTN 2016 vịng 2) Cho hình vngABCD nội tiếp đường trịn tâm(O).P là điểm thuộc cung nhỏADcủa đường tròn(O)P khácA, D Các đường thẳng

P B, P C lần lược cắtADtạiM, N Đường trung trực củaAM cắt đường thẳngAC, P Blần lượt tạiE, K Đường trung trựcDN cắt đường thẳngBD, P C lần lượt tạiF, L.

1) Chứng minh ba điểmK, O, Lthẳng hàng.

2) Chứng minh đường thẳngP Ođi qua trung điểm đọạn thẳngEF.

3) Giả sử đường thẳngEKcắt đường thẳngBDtạiS, đường thẳngF LACcắt tại

T, đường thẳngST cắt đường thẳngP B, P C lần lượt tạiU V Chứng minh bốn điểmK, L, U, V cùng thuộc đường tròn.

O

B

D A

C P

M N

E K

F L S

T U

V

G H

Q I

J

(182)

Lời giải 1) Gọi G, H trung điểm AM vàDN Ta có GK k AB(⊥ AM)và

HL k DC(⊥ DN) Do theo tính chất đường trung bình suy raK, L tương ứng trung điểm củaM B vàN C Từ ta cóOK vàOLlà đường trung bình của4BM Dvà4CN A Suy raOK kM DvàOLkAN Do đó,K, O, Lthẳng hàng (tiên đề Euclid)

2) Xét tam giác AM E DN F cân E, F, có ∠EAM = ∠F DN = 45◦

nên 4AM E và4DN F tam giác vuông cân, M E ⊥ OA vàN F ⊥ OD Đặt

M E cắt P O I Áp dụng định lý Thales cho M N k BC vàM I k BD(⊥ OA), ta

P I P O =

P M P B =

P N

P C Suy IN k OC vàIN ⊥ OD Từ đó,I, N, F thẳng hàng Ta OEIF hình chữ nhật Vì vậyIOđi qua trung điểmEF MàI ∈P O, nên ta đượcP Ođi qua trung điểmEF

3) Đặt P O cắtST J DoIM k OB vàIN k OC nên ∠IM P = ∠OBP = ∠OP M

∠IN P =∠OCP =∠IP N, vậyIP =IM =IN hayI tâm ngoại tiếp của4P M N Mặt khác, từM E kSBvàKM =KB, áp dụng định lý Thales suy raKE =KSvà4OESvuông cân tạiO Tương tựF L=LT và4OF T vng cân tạiO Từ ta được4OEF =4OST Suy ra∠OT S = ∠OF E =∠P OF = 90◦−∠P OE = 90◦−∠J OT, vậy∠OJ T = 90◦

hayP O ⊥ST Từ suy ra∠P V U = 90◦−∠IP V =

2∠P IN =∠P M N =∠P KL(doI

là tâm ngoại tiếp4P M N vàM N kKL) Do ta tứ giácKLV U nội tiếp

Nhận xét Đây toán nằm đề thi vào lớp chuyên toán, ý 1) nhẹ nhàng, ý 2) khó chút ý 3) có tính phân loại Cả ý 2) ý 3) địi hỏi phải dựng thêm hình phụ để làm bài, mục đích làm đề thi trở nên khác biệt so với đề trường khác Cấu trúc đề chặt chẽ với câu, ý 2) dùng ý 1) ý 3) dùng ý 2) Kết nằm một bài toán có ý nghĩa thu gọn lại đề sau

Bài 0.15 Cho hình vngABCDnội tiếp đường tròn(O).P là điểm thuộc cung nhỏ

ADcủa(O).P B, P C lần lượt cắt đoạnADtạiM, N Trung trực củaAM, DN lần lượt cắt

BD, AC tạiS, T.ST cắtP C, P Blần lượt tạiU, V Chứng minh đường trịn đường kính

U V tiếp xúc(O)

Bài 0.16 (THPT chuyên ĐHSP 2016 vòng 1) Cho ba điểm phân biệtA, M, B thẳng hàng và

M nằm giữaAB Trên nửa mặt phẳng bờ đường thẳngABdựng hai tam giác đềuAM C BM D GọiP là giao điểm củaADBC.

1) Chứng minhAM P C BM P Dlà tứ giác nội tiếp. 2) Chứng minh rằng√CP.CB+√DP.DA=AB

3) Đường nối tâm hai đường tròn ngoại tiếp tứ giácAM P C BM P DcắtP AP B

tạiE F Chứng minhCDF Elà hình thang.

Lời giải. 1) Ta cóM C =M A, M B =M D và∠CM B =∠AM D= 120◦, đó4M CB = 4M AD(c.g.c) Từ suy ∠M CP = ∠M AP ∠M BP = ∠M DP Vậy AM P C

BM P Dlà tứ giác nội tiếp

2) Do∠CM D = 180◦−∠AM C−∠BM D = 60◦ = ∠M BDnênCM tiếp xúc với đường tròn ngoại tiếp4BM D Ta được∠CM P =∠M BP, suy ra4CP M ∼ 4CM B(g.g), suy

CP CM =

CM

CB hayCM

2 =CP ·CB Chứng minh tương tự ta đượcDM2 =DP ·DA.

Vậy√CP ·CB+√DP ·DA=CM +DM =AM +BM =AB

(183)

Tạp chí Epsilon, Số 09, 06/2016

A M B

C

D

P

I

J

E F

Hình 20.3

ta AE

AP = AM

AB = P F P B hay

AE P F =

AP

P B Mặt khác ta có∠P IA= 2∠P M A= 2∠P DB =

∠P J B hay4P IA ∼ 4BJ P, suy IA

J P = AP P B =

AE

P F, suy 4IAE ∼ 4J P F(c.g.c)

Từ ta IE

J F = IA J P =

IC

J D, kết hợp∠CIE = ∠DJ F (do IC k J D(⊥ AB)), ta

4CIE ∼ 4DJ F(c.g.c), suy ∠CEI = ∠DF J Do vậy,CE k DF hayCDF E hình thang

Nhận xét Cấu hình đề khơng ý 3) phù hợp với phân loại Bài tốn có ba ý theo cấu trúc tốn thi hình học rõ ràng kết hợp với hau ý đầu dễ ý 3) phân loại phù hợp cho đề thi chung Kết nằm tốn có ý nghĩa khi tổng qt sau

Bài 0.17 Cho tam giác ABC với D thuộc đoạn BC. E, F thuộc đoạn CA, AB sao cho

DE kAB, DF kAC Đường tròn(K),(L)ngoại tiếp tam giácDCF DBE cắt tạiG

khácD.KLcắtGB, GC tạiM, N Chứng minh rằngF M kEN.

(184)

nghĩ lời giải viết lại Tất nhiên với tốn phức tạp tưởng tượng chửa đủ mà phải nhìn hình vẽ, điều khơng có nghĩa máy tính thay thế con người, đóng vai trị hỗ trợ Vậy nên việc có máy tính hay khơng có máy tính khi giải hình khơng quan trọng người giải có tư hình học sắc xảo tới đâu.

(185)

Tuyển chọn đề thi Olympic năm 2016 dành cho học sinh THCS

(Ban biên tập)

Đề thi (EGMO 2016) Chon là số nguyên dương lẻ vàx1, x2, , xnlà số thực không âm Chứng minh rằng

min 16i6n(x

2 i +x

2

i+1)6 max

16j6n(2xjxj+1), trong đóxn+1 =x1

Đề thi (Saudi Arabia TST 2016, Junior) Choa, b, clà số thực dương thỏa mãn điều kiện

a+b+c= 3.Chứng minh rằng

1

a2+b2+c2 + √

abc >

4

Đề thi (ĐHSP HN, tuyển sinh 10) Choa, b, clà số thực không âm thỏa mãn điều kiện

a+b+c= 1.Chứng minh ta có bất đẳng thức

5a+ +√5b+ +√5c+ 4>7

Đề thi (AMC 2016, 10B) Giả sửf(x) = 10

P

k=2

([kx]−k[x])trong đó[x]ký hiệu số ngun lớn nhất khơng vượt qx.Nếux>0thìf(x)có thể nhận giá trị?

Đề thi (APMOPS 2016) Biết sốa1 = 1000, a2 = 1016a3 = a1+2a2, a4 = a2+2a3, Tính giá trị củaba15ctrong đóbxclà phần nguyên củax

Đề thi (Tournament of the towns 2016) Tất số nguyên từ1đến1000000được viết trên một băng giấy theo thứ tự Sau băng giấy cắt thành mẩu chứa hai chữ số liên tiếp Chứng minh mẩu giấy chứa tất số có hai chữ số, cho dù thứ tự ban đầu số nào.

Đề thi (SASMO 2016, G8) Từ72số1, 2, 3, , 72có thể chọn nhiều số sao cho số chọn, khơng có số có tích bằng72?

Đề thi (Moscow MO 2016, G8) Tìm số nguyên dương nhỏ bội của99mà cách viết thập phân có chữ số chẵn.

Đề thi (PTNK 2016, tuyển sinh lớp 10) Lớp9A có27học sinh nam và18học sinh nữ Nhân dịp sinh nhật bạnX(là thành viên lớp), bạn lớp có nhiều quà tặng

X.Ngoài bạn nam lớp làm3tấm thiệp bạn nữ xếp2hoặc5con hạc để tặng bạnX.Biết số thiệp số hạc nhau, hỏi bạnXlà nam hay nữ?

(186)

Khoảng cách hai điểm số bước theo chiều ngang chiều dọc theo đường lưới để từ điểm đến điểm Ví dụ hai điểm cạnh có khoảng cách1cịn hai điểm ở hai góc đối lưới có khoảng cách6.Gọi khoảng cách trung bình hai điểm khác nhau lưới là mn vớim, nlà số nguyên dương nguyên tố Hãy tìmm+n

Đề thi 11 (Moscow MO 2016, G7) Một góc tam giác bằng60◦ và cạnh đối diện với góc này phần ba chu vi tam giác Chứng minh tam giác tam giác đều.

Đề thi 12 (Tournament of the towns 2016) Cho tam giác nhọnABC với∠C = 60◦.GọiH trực tâm tam giác Đường trịn tâmH bán kínhHC cắt đường thẳng CACB tại các điểm thứ haiM N tương ứng Chứng minh đường thẳngAN BM song song hoặc trùng nhau.

Đề thi 13 (Moscow MO 2016, G8) ĐiểmO là tâm đường tròn ngoại tiếp tam giác nhọnABC

Đường thẳng vng góc với cạnhAC cắt đoạn BC và đường thẳngBC ở điểmQ P

tương ứng Chứng minh điểmB, Ovà trung điểm đoạnAP CQnằm một đường tròn.

Đề thi 14 (EGMO 2016) Cho tứ giácABCDnội tiếp Hai đường chéoACBDcắt nhau tạiX.Giả sửC1, D1, M lần lượt trung điểm của CX, DX CD.Đường thẳngAD1

BC1 cắt tạiY, M Y cắtAC BDtại điểm thứ haiE, F tương ứng Chứng minh rằng

XY tiếp xúc đường tròn quaE, F, X

(187)

Tạp chí Epsilon, Số 09, 06/2016

Tìm góc∠EAD

Đề thi 16 (APMOPS 2016) Bốn đường tròn qua điểm tiếp xúc với đường trịn lớn Biết bán kính đường trịn lớn bằng14

Lấyπ = 227.Tính diện tích phần tô đậm.

Đề thi 17 (SASMO 2016, G9) ChoABCDlà tờ giấy hình chữ nhật vớiAB= 18AD= 12

ĐiểmE là điểm trênCD sao choDE :CD = : 2.Hình chữ nhậtABCDđược gấp lại sao choAtrùng vớiE

Tính chiều dài đoạn gấp.

Đề thi 18 (Saudi Arabia TST 2016, Junior) Cho tứ giácABCDnội tiếp đường tròn(O) AC

cắtBDtại P.GọiQ, R là hai điểm cungACDkhông chứa điểmA, B.Các đường thẳngRA, RCcắt đường tròn ngoại tiếp tam giácP QRtại điểm thứ haiL, K P K, P L

cắtBC, AD tạiM, N tương ứng Chứng minh rằngBC ADcắt điểm nằm trên đường tròn ngoại tiếp tam giácQM N

Đề thi 19 (Moscow MO 2016, G7) Hãy thay vào dấusáu chữ số phân biệt cho các phân số tối giản đẳng thức đúng

∗ ∗ +

∗ ∗ =

∗ ∗

Đề thi 20 (SASMO 2016, G8) Trong phép tính đây, chữ khác đại diện cho các chữ số khác nhau.

S A S M O

×

(188)

Hãy tìm số có5chữ sốMATHS.

Đề thi 21 (SASMO 2016, G9) Trong phép tính đây, chữ khác đại diện cho các chữ số khác nhau.

S T A Y

× C O O L

S A S M O

Hãy tìm số có5chữ sốSASMO.

Đề thi 22 (AMC 2016, 10A) Phép tốncó tính chấta♦(b♦c) = (a♦b)·ca♦a = với số thực không âma, bc(ở đâyı·biểu thị phép nhân) Nghiệm phương trình 2016♦(6♦x) = 100được viết dạng pq,trong đópqlà số nguyên dương nguyên tố cùng Hãy tínhp+q

Đề thi 23 (IMC 2016, 2nd round, G8)P(x)là đa thức bậc8thỏa mãn điều kiện

P(1) =P(˘1), P(2) = +P(˘2), P(3) = 24 +P(˘3),

P(4) = 60 +P(˘4).Hãy tính giá trị củaP(5)˘P(˘5)

Đề thi 24 (Purple Comet 2016) Đa thức bậc bap(x)q(x)thỏa mãn điều kiện

p(1) =q(2), p(3) =q(4), p(5) =q(6),

p(7) =q(8) + 13.Hãy tìmp(9)−q(10)

Đề thi 25 (Saudi Arabia TST 2016, Junior) Choklà số nguyên dương Chứng minh tồn tại số nguyênx, y không số chia hết cho3sao chox2+ 3y2 = 3k.

Đề thi 26 (Saudi Arabia Training Camp 2016) ChoSlà tập hợp tất số tự nhiên biểu diễn được dạngx2+ 3y2vớix, ylà số nguyên, tức làS ={n∈N|n =x2+ 3y2 trong đó

x, y là số nguyên}.Chứng minh tính chất sau củaS :

i) Nếum∈S, n∈Sthìm·n∈S

ii) NếuN ∈S2|N thì N4 ∈S.NếuN ∈S,số nguyên tốp∈Sp|N thì Np ∈S

iii) Choplà số nguyên tố dạng3k+ 1.Chứng minh tồn số nguyênN sao choN2+ 3 chia hết chop.Từ suy rap∈S

iv) Tìm điều kiện cần đủ để số nguyên dươngN >1thuộcS

Đề thi 27 (SASMO 2016, G7) Có cách xếp chữ từ SASMO cho hai chữ S xếp cạnh nhau?

Đề thi 28 (APMOPS 2016) Có6chữA, P, M, O, P, S.Một máy tính xếp từ theo thứ tự từ điển:AM OP P S, AM OP SP, AM OSP P, , SP P OM A.Hỏi từP OAM SP

(189)

Tạp chí Epsilon, Số 09, 06/2016

Đề thi 29 (Moscow MO2016, G6) Ở thành phố nhỏ có tuyến tàu điện Nó chạy vịng trịn tàu điện chạy theo vòng tròn theo hai phía Trên vịng có3bến Rạp xiếc, Cơng viên Sở thú Từ Cơng viên đến Sở thú đường qua Rạp xiếc dài gấp3lần đường không qua Rạp xiếc Từ Rạp xiếc đến Sở thú đường qua Công viên ngắn nửa đường đi qua đường không qua Công viên Hỏi đường từ Công viên đến Rạp xiếc ngắn – Qua Sở thú hay không qua Sở thú ngắn lần?

Đề thi 30 (Moscow MO2016, G8) Có2nviên sỏi (nnguyên dương cho trước) xếp thành3đống Mỗi lần thực cho phép lấy nửa số viên sỏi từ đống sỏi có chẵn viên sỏi và chuyển sang đống khác Chứng minh rằng, dù đống sỏi ban đầu xếp nào, sau một số hữu hạn bước thực thế, ta tạo đống sỏi có đúngnviên sỏi.

Đề thi 31 (Moscow MO 2016, G8) Xung quanh bàn trịn có10người ngồi, người trong họ hiệp sĩ, người ln nói thật kẻ nói dối người ln nói dối Có hai người trong họ tuyên bố “Cả hai người ngồi cạnh kẻ nói dối” cịn tất người cịn lại đều nói “Cả hai người ngồi cạnh tơi hiệp sĩ” Hỏi có hiệp sĩ số10người này (liệt kê tất phương án chứng minh khơng cịn phương án khác). Đề thi 32 (Gulf MO2016) Giả sử có4ngườiA, B, CDđánh tennis đơi với Họ có thể tổ chức trận đấu sau: Trận đấuAB đấu vớiCD,trận tiếp theoAC đánh vớiB D,cuối cùngADđánh vớiB C.Cái hay cách xếp hai điều kiện sau thỏa mãn:

a) Hai vợt chung đội với đúng1lần.

b) Hai vợt đấu hai đội khác đúng2lần.

Hỏi xếp trận đấu cho điều kiện a) b) thỏa mãn trường hợp sau? Giải thích rõ câu trả lời.

i) Có5người chơi.

ii) Có7người chơi.

iii) Có9người chơi.

Đề thi 33 (IMC 2016, 2nd round, G6) ƠngAcó khoảng hơn1000con gà ƠngAđếm số gà của báo choB.ÔngBlại báo choC.ÔngC báo choDDlại báo choE.Trớ trêu là tất nhầm lẫn cố tính nhầm lẫn:Ađếm thừa9con gà so với thực tế.Btrước khi báo choCđã đổi chỗ hai chữ số cuối cùng.Cđã đổi chỗ chữ số chữ số thứ ba trước khi báo choD Dnhân đôi số nhận báo choE.Kỳ lạ thay, số màEnhận lại đúng số gà màAcó HỏiAcó gà?

(190)

Đề thi 35 (Purple Comet 2016) Có10viên gạch lát thành hàng dọc, viên có thể sơn trong4màu đỏ (Đ), vàng (V), xanh (X), trắng (T) Tìm số cách sơn sao cho blốc5viên gạch liên tiếp chứa đủ4màu Ví dụ cách tơ ĐTXTVĐĐXTV và TTXVĐTVXTĐ ĐTXVVXTTĐV khơng viên gạch liền XVVXT khơng chứa màu đỏ.

Đề thi 36 (AMC 2016, 10B) Một số đội bóng thi đấu vịng trịn1lượt, hai đội đấu với nhau đúng1trận Mỗi đội thắng10trận thua10trận, khơng có hịa Hỏi có bộ3 đội{A, B, C}AthắngB, BthắngA, C thắngA?

Đề thi 37 (SASMO 2016, G6) Một xe mất3giờ41phút để từ thành phốAđến thành phốB và quay từB vềA.Xe xuống dốc với vận tốc6km/h (kilômét/giờ), đường với vận tốc5km/h lên dốc với vận tốc4km/h Đoạn đường từAđếnB 4km đường Tính khoảng cách hai thành phố.

(191)

LỜI GIẢI ĐỀ THI TOÁN QUỐC TẾ FORMULA OF UNITY

THE THIRD MILLENNIUM

(Ban biên tập)

Tiếp theo Epsilon số 6, Ban biên tập xin giới thiệu với bạn đọc lời giải đề thi kỳ thi Formula of Unity Phần gồm đề khối lớp 5, 6, 7, Các khối lại giới thiệu vào kỳ tới

1 Đề thi cho khối lớp R5

Bài 1. Peter, Basil Anatoly góp tiền tiết kiệm để mua bóng giá la Biết rằng, số tiền góp người khơng nhiều nửa tổng số tiền góp hai người cịn lại Hỏi Peter đóng góp tiền?

Lời giải.Nếu số tiền ba bạn không phải có người có có nhiều tiền Giả sử người Peter Khi đó, số tiền Peter nhiều trung bình cộng số tiền hai bạn lại Tuy nhiên, theo đề số tiền Peter lại khơng nhiều nửa tổng số tiền hai bạn cịn lại Do đó, số tiền bạn góp đô la

Bài 2. Pauline viết hai số A B lên bảng Victoria xóa hai số A; B viết tổng C tích D của chúng Sau đó, Pauline lại xố hai số C D; thay tổng E tích F chúng Biết một trong hai số E F số lẻ Hỏi số nào?

Lời giải.Theo đề thì:

C =A+B D=AB

E =A+B+AB F = (A+B)AB

Ta thấyF số chẵn, sốA, B có số chẵn thìF chẵn; cịn nếuA, B lẻ thìA+Bchẵn vàF chẵn

Do đó, hai sốE, F lẻ số làE (chẳng hạn khiAchẵn,B lẻ)

Bài 3. Ta nói học sinh A học tốt học sinh B điểm A cao điểm B trong phần lớn kiểm tra Sau kiểm tra, thầy giáo nhận xét học sinh A học tốt hơn học sinh B, học sinh B học tốt học sinh C học sinh C lại học tốt học sinh A Liệu điều này xảy không?

(192)

2 Học sinhBđược:8,9,10

3 Học sinhCđược:10,8,9

Ghi chú:bài có lẽ lấy ý tưởng từ đề IMO Shortlist 2014 sau: Có có vơ số qn mà qn viết số thực; ra, với số thựcxthì có qn viết sốx

Hai người chơi lựa chọn tùy ý tập hợp rời nhauA, B mà tập gồm 100 quân từ Nhiệm vụ chọn luật chơi để quân chọn, xác định người thắng Đặc biệt, luật lại phải thỏa mãn số ràng buộc sau:

1 Nó dựa việc so sánh số viết hai 100 quân (không quan tâm đến giá trị cụ thể chúng bao nhiêu)

2 Nếu liệt kê tập hợp theo thứ tự tăng dần:A={a1, a2, , a100}vàB ={b1, b2, , b100}

Ngồi ra, ta cóai > bi vớii= 1,2,3, ,100thì đóAphải thắngB

3 Nếu có người chơi chọn ba 100 quân làA, B, Cmà bộAthắng

B, bộBthắng bộCthì bộAcũng phải thắng bộC

Hỏi có tất cách định nghĩa luật chơi thế?

Hai luật chơi khác tồn tập hợpA, B màAthắngB luật này, lại thuaB luật

Bài 4. Nếu Leon bị điểm trường, cậu ta dành buổi tối để nói dối mẹ Cịn ngược lại, cậu ta ln nói thật Leon có em gái ln mẹ cho kẹo hơm bé khơng bị điểm Một buổi tối, Leon nói với mẹ: "Hơm bị nhiều điểm em". Hỏi cô em gái Leon có mẹ cho kẹo hay khơng?

Lời giải.Câu nói Leon khơng thể câu nói thật Suy Leon bị điểm em gái Leon có số điểm nhiều Leon Suy cô gái Leon không mẹ cho kẹo Câu trả lời phủ định

Bài 5. Một tờ lịch ma thuật ngày tất ngày chẵn tháng sai ngày vào tất cả các ngày lẻ Hỏi số lớn ngày liên tiếp ghi ngày lịch bao nhiêu? Và tháng?

Lời giải.Số lớn thỏa mãn là4.Đó ngày31,1,2,3và ghi ngày2

Bài 6. Có số có biểu diễn thập phân gồm 10 chữ số khác có chứa đoạn0123?

Lời giải.Coi 0123 chữ số, còn4,5,6,7,8,9là6chữ số khác Vì 0123 khơng xếp đầu nên có6cách chọn vị trí cho0123.Sau có6!cách xếp6chữ số cịn lại vào chỗ lại Đáp số là6.6! = 4320

Bài 7. Hình vng8×8được vẽ tờ giấy kẻ ô vuông dọc theo đường kẻ Alex cắt hình vng8×8theo đường kẻ thành7phần với chu vi Hãy cách cắt của Alex.

(193)

Tạp chí Epsilon, Số 09, 06/2016

2 Đề thi cho khối lớp R6

Bài 1. Có 14 người ngồi quanh vòng tròn Peter, Victoria, Anatoly Genghis ngồi cạnh theo thứ tự bạn có đồng xu mệnh giá 1, 2, 10 rúp Một người có thể đưa đồng xu cho người bên trái bên phải có người ngồi họ Sau lúc chuyển vậy, bạn Peter, Victoria, Anatoly Gengis lại nhận được đồng xu Hỏi lúc này, bạn cầm đồng xu nào? Hãy tất khả năng và chứng minh khơng cịn trường hợp khác.

Lời giải.Ta thầy người số chuyển tiền cho người thứ 5, 9, 13, 3, tức người vị trí lẻ chuyển tiền cho Tương tự, người vị trí chẵn chuyển tiền cho Suy phương án tất phương án thỏa mãn 1, 2, 5, 10; 1, 10, 5, 2; 5, 2, 1, 10, 5, 10, 1,

Bài 2. Pauline viết hai số A B lên bảng Victoria xóa hai số A; B viết tổng C tích D của chúng Sau đó, Pauline lại xố hai số C D; thay tổng E tích F chúng Biết một trong hai số E F số lẻ Hỏi số nào?

Lời giải Xem lời giải phần trước

Bài 3. Ta nói học sinh A học tốt học sinh B điểm A cao điểm B trong phần lớn kiểm tra Sau kiểm tra, thầy giáo nhận xét học sinh A học tốt hơn học sinh B, học sinh B học tốt học sinh C học sinh C lại học tốt học sinh A: Liệu điều này xảy khơng?

(194)

Bài 4. Nếu Leon bị điểm trường, cậu ta dành buổi tối để nói dối mẹ Cịn ngược lại, cậu ta ln nói thật Leon có em gái ln mẹ cho kẹo hơm bé khơng bị điểm Một buổi tối, Leon nói với mẹ: "Hôm bị nhiều điểm em". Hỏi em gái Leon có mẹ cho kẹo hay không?

Lời giải.Xem lời giải phần trước

Bài 5. Một tờ lịch ma thuật ngày tất ngày chẵn tháng sai ngày vào tất cả các ngày lẻ Hỏi số lớn ngày liên tiếp ghi ngày lịch bao nhiêu? Và tháng?

Lời giải.Xem lời giải phần trước

Bài 6. Có số có biểu diễn thập phân gồm 10 chữ số khác có chứa đoạn 0123 hoặc đoạn 3210?

Lời giải.Kết 6.6! + 7! = 9360 Lập luận tương tự lớp

Bài 7. Hình vng8×8được vẽ tờ giấy kẻ vng dọc theo đường kẻ Alex cắt hình vng8×8theo đường kẻ thành7phần với chu vi Hãy cách cắt của Alex.

Lời giải.

Xem lời giải phần trước

3 Đề thi dành cho khối lớp R7

Bài 1. Một tờ lịch ma thuật ngày tất ngày chẵn tháng sai ngày vào tất cả các ngày lẻ Hỏi số lớn ngày liên tiếp ghi ngày lịch bao nhiêu? Và ngày ngày nào?

Lời giải.Xem lời giải phần trước

(195)

Tạp chí Epsilon, Số 09, 06/2016

Lời giải. Do số phân biệt nên tổng số hàng cột với2015tối thiểu

5+6+· · ·+12 = 68.Suy tổng chung hàng, cột phải bằng2015+34 = 2049

Và ta sau:

Bài 3. Hình vng8×8được vẽ tờ giấy kẻ vng dọc theo đường kẻ Alex cắt hình vng8×8theo đường kẻ thành7phần với chu vi Hãy cách cắt của Alex.

Lời giải.Xem lời giải phần trước

Bài 4. 27con gián tham gia chạy đua Trong vịng có ba gián chạy Mỗi con gián chạy với tốc độ cố định, khơng đổi vịng đua, tốc độ gián đôi một khác Sau vòng, người ta ghi lại thứ tự đích gián tham gia vịng đua Hỏi14vịng đua có đủ để xác định xác theo thứ tự hai gián chạy nhanh nhất không?

Lời giải.Câu trả lời khẳng định

Ta dùng vòng đầu loại chậm vòng chọn nhanh nhanh loại chậm Vòng 13 chọn nhanh (vô địch) nhanh Lúc cịn đứng nhì đứng nhì vịng 13, đứng nhì đua với vơ địch vịng trận đứng nhì đua với vơ địch vịng đầu Dùng trận 14 để tìm thứ nhì từ

Bài 5. Ta nói học sinh A học tốt học sinh B điểm A cao điểm B trong phần lớn kiểm tra Sau kiểm tra, thầy giáo nhận xét học sinh A học tốt hơn học sinh B; học sinh B học tốt học sinh C học sinh C lại học tốt học sinh A: Liệu điều này xảy khơng?

Lời giải.Xem lời giải phần trước

(196)

Lời giải.Trước hết, ta thấy số hữu tỉ dương tốt tích thương chúng tốt

Ngoài ra, số nguyên dươngnđều viết thành (nn−!1)!

Từ đó, ta đưa toán chứng minh số nguyên dương tốt ta thực điều quy nạp

Vớin= 1, ta có1 = 2!2! số tốt Vớin= 2,ta có2 = 2!2!2! số tốt Vớin= 3, ta có3 = 3!2! số tốt

Khi đó, vớin>4, hợp số, ta viết thành tích số nguyên tố nhỏ theo giả thiết quy nạp, số tốt

Nếun >4là số nguyên tố, ta viếtn = (nn−!1)! vàn−1là hợp số, số tốt nên suy ranlà số tốt

Theo nguyên lý quy nạp nhận xét chứng minh Bài toán giải Chẳng hạn5 = 5!4! = 2·5!3·4 = 25!3·3 =

5! (2!)3·3!

2!

= (2!)5!2·3!

Bài 7. Ta gọi số nguyên dương đẹp dãy chữ số tăng thực sự, ví dụ 1589 là số tăng cịn 447 khơng Hãy tìm số nhỏ số nguyên dương đẹp với tổng là2015

Lời giải.Ta cần số là:1789 + 189 + 37 = 2015 Rõ ràng thân số 2015 không đẹp nên dùng số để biểu diễn

Ta chứng minh dùng số để biểu diễn 2015

Giả sửx+y= 2015vàx < y(hai số khơng thể vì2015lẻ) Dễ thấy số đẹp lớn không vượt quá2015là1789.Suy rax < y 61789

Dễ thấy hai số có chữ số trở lên khơng thỏa nên ta đặtx=abc, y = 1def, đó0 < a < b < c,1 < d < e < f.Ta thấyabc+ 1def = 2015 Ta thấyc+f =

hoặcc+f = 15, nếuc+f = thìc, f nhỏ, khơng thỏa Thế nênc+f = 15và dẫn đếnb+e= 9(vì phép tính trước có nhớ, kết phải tận 0) Khi phép tính

a+d= 9(tương tự trên, phép tính trước có nhớ kết phải tận 0) Nhưng đó, ta cób+e=a+d, mâu thuẫn vìa < b, d < e.Do đó, khơng tồn số đẹp có tổng 2015 Đáp số Ghi Số lớn không vượt 2015 thỏa mãn tính chất: tổng số đẹp số 1978 Khi đó:1978 = 189 + 1789

4. Đề thi dành cho Khối lớp R8

Bài 1. Hãy điền vào ô bảng vng5×5các số ngun dương phân biệt cho tổng các số hàng, cột nhỏ Biết rằng, số1,2,3,4,2015 đã điền trước đường chéo.

Lời giải.Xem lời giải phần trước

(197)

Tạp chí Epsilon, Số 09, 06/2016

Lời giải.Xem lời giải phần trước

Bài 3. Hãy tìm số nguyên dương cho tích ước tự nhiên là1090.

Lời giải. Ta tìm số dạng2m5n Các ước số có dạng2a·5b với0 6 a 6 m,0 6

b 6n Từ tích ước bằng10mn(m+1)(4 n+1) nên ta đưa vềmn(m+ 1)(n+ 1) = 360.Chọn m= 3, n= 5thì ta số thỏa mãn yêu cầu toán là23·55 = 25000.

Bài 4. John có 12 que gỗ với độ dài que số nguyên dương không vượt 56 Chứng minh John có que tạo thành tam giác.

Lời giải.Giả sử ngược lại, que tạo thành tam giác Xếp thứ tự chiều dài que gỗa1 6a2 a12 từ giả thiết, ta suy raan+2 >an+1 +an Từ suy

a3 > a2 +a1 > 2, a4 > a3 +a2 > 3, a5 > a4 +a3 > Cứ thếa6 > 8, a7 > 13, a8 > 21, a9 >34, a10>55, a11>89.Điều mâu thuẫn, suy John tạo tam giác

Bài 5. Ta nói số nguyên dương đẹp tích giai thừa số nguyên tố (không thiết phải phân biệt) Ta gọi số hữu tỉ dương tốt tỉ số hai số nguyên dương đẹp Chứng minh tất số hữu tỉ dương tốt.

Lời giải.Xem lời giải phần trước

Bài 6. Cho tam giác ABC với∠B = 30◦,∠C = 105◦và D trung điểm đoạn thẳng BC.

Tìm góc∠BAD?Lời giải.Hạ CH vng góc với AB suy CHD tam giác AHC tam giác vuông cân H Từ suy tam giác AHD cân H

∠BAD =∠HAD = 15◦

Bài 7. Ta nói học sinh A học tốt học sinh B điểm A cao điểm B trong phần lớn kiểm tra Sau kiểm tra, thầy giáo nhận xét học sinh A học tốt hơn học sinh B; học sinh B học tốt học sinh C học sinh C lại học tốt học sinh A Liệu điều này xảy khơng?

(198)(199)

CÁC VẤN ĐỀ CỔ ĐIỂN VÀ HIỆN ĐẠI

Trần Nam Dũng - Đại học Khoa học Tự nhiên - ĐHQG TP.HCM

LỜI GIỚI THIỆU

Chuyên mục dành cho vấn đề cổ điển đại trình bày dạng tốn xâu chuỗi Đó chuỗi để giải toán đẳng chu, chứng minh đẳng thức Euler kỳ diệu1 + 212 +

1

32 +· · · =

π2

6 ,một chuỗi toán

vận trù Cách trình bày xuất phát từ vấn đề đơn giản, dễ hiểu, khái niệm định nghĩa ln để đọc tương đối độc lập Và chuỗi nêu vấn đề định, giải toán kinh điển hay nêu giả thuyết mới, vấn đề Lời giải thảo luận toán đăng sốN +

(200)

Đề thi thức

Chủ đề: Đại số

Mục tiêu thi tìm hiểu số trường hợp riêng định lý Markov: NếuP(x)là đa thức với hệ số thực có bậc khơng vượt qnthì

max

|x|61|P

0

(x)|6n2max

|x|61|P(x)|

Chứng minh định lý Markov vượt q chương trình tốn THPT Ta tìm cách chứng minh trường hợp riêng khin63của định lý khảo sát số tốn xung quanh trường hợp

Trong toán đây, biến sốxchỉ nhận giá trị thực

A - Bất đẳng thức Markov cho đa thức bậc nhất

Bài PT Giả sửa, blà hai số thực cho|ax+b|61khi|x|61.Chứng minh rằng: (i) |a|61

(ii) |bx+a|61khi|x|61

B - Bất đẳng thức Markov cho đa thức bậc hai bậc ba

Bài PT Giả sửa, b, clà ba số thực cho giá trị đa thựcax2+bx+ctại1, 0, −1 đều thuộc đoạn[−1,1]

(i) Chứng minh rằng|2ax+b|64khi|x|61

(ii) Chứng minh rằng|cx2+bx+a|62khi|x|61.

Bài PT Giả sử a, b, c, d là bốn số thực cho giá trị α, β, γ δ của đa thức

ax3+bx2+cx+dtương ứng tại−1,−1 2,

1

2, 1đều thuộc đoạn[−1,1]

(i) Chứng minh với số thựcA, B ta có|A+B|+|A−B|= max{|A|,|B|}

(ii) Bằng cách biểu diễn3ax2+ 2bx+ctheoα, β, γ, δx, chứng minh bất đẳng thức |3ax2+ 2bx+c|69khi|x|61.

(iii) Chứng minh rằng|dx3+cx2+bx+a|64khi|x|61

C - Hai bất đẳng thức khác cho tam thức

Bài PT Cho a, b, c là ba số thực và n là số nguyên dương Giả sử đa thức

https://blogs.ethz.ch/kowalski/smoothing-sums-wiki-page/ http://www.math.tifr.res.in/publ/ln/tifr59.pdf.Zagier http://people.mpim-bonn.mpg.de/zagier/files/tex/MellinTransform/fulltext.pdf blog bài này) https://www.google.com/patents/US4567600 Boy or Girl Paradox. Phép biến đổi Fouriercó ý nghĩa vật lý gì? Steve on Image Processing, Frequency domain http://artofproblemsolving.com www.cut-the-knot.org/triangle/80-80-20/Classical1.shtml http://www.cut-the-knot.org/m/Algebra/BrahmaguptaFibonacci.shtml

Ngày đăng: 08/02/2021, 07:44

Tài liệu cùng người dùng

Tài liệu liên quan